HS 300 Financial Planning: Process and

Transcription

HS 300 Financial Planning: Process and
Horizons Student Workbook
HS 300 Financial Planning: Process and
Environment
For use with Financial Planning: Process and Environment,
3d Edition
300SW-3
This publication is designed to provide accurate and authoritative information about the subject covered. While every precaution
has been taken in the preparation of this material, the authors, and The American College assume no liability for damages
resulting from the use of the information contained in this publication. The American College is not engaged in rendering legal,
accounting, or other professional advice. If legal or other expert advice is required, the services of an appropriate professional
should be sought.
© 2009 The American College Press
All rights reserved
Printed in the United States of America
The American College
The American College® is an independent, nonprofit, accredited institution founded
in 1927 that offers professional certification and graduate-degree distance education to
men and women seeking career growth in financial services.
The Center for Financial Advisor Education at The American College offers both
the LUTCF and the Financial Services Specialist (FSS) professional designations to
introduce students in a classroom environment to the technical side of financial services,
while at the same time providing them with the requisite sales-training skills.
The Solomon S. Huebner School® of The American College administers the
Chartered Life Underwriter (CLU®); the Chartered Financial Consultant (ChFC®);
the Chartered Advisor for Senior Living (CASL®); the Registered Health Underwriter
(RHU®); the Registered Employee Benefits Consultant (REBC®); and the Chartered
Leadership Fellow® (CLF®) professional designation programs. In addition, the
Huebner School also administers The College’s CFPŠ Boardregistered education
program for those individuals interested in pursuing CFP® certification, the CFP®
Certification Curriculum.1
The Richard D. Irwin Graduate School® of The American College offers the master
of science in financial services (MSFS) degree, the Graduate Financial Planning
Track (another CFP Board-registered education program), and several graduate-level
certificates that concentrate on specific subject areas. It also offers the Chartered
Advisor in Philanthropy (CAP)® and the master of science in management (MSM), a
one-year program with an emphasis in leadership. The National Association of Estate
Planners & Councils has named The College as the provider of the education required
to earn its prestigious AEP designation.
The American College is accredited by:
The Middle States Commission on Higher Education
3624 Market Street
Philadelphia, PA 19104
267.284.5000
The Middle States Commission on Higher Education is a regional accrediting agency
recognized by the U.S. Secretary of Education and the Commission on Recognition of
1.
Certified Financial Planner Board of Standards, Inc., owns the certification marks CFP®, CERTIFIED FINANCIAL
PLANNER™, and CFP (with flame logo)®, which it awards to individuals who successfully complete initial and ongoing
certification requirements.
iii
Postsecondary Accreditation. Middle States accreditation is an expression of confidence
in an institution’s mission and goals, performance, and resources. It attests that in the
judgment of the Commission on Higher Education, based on the results of an internal
institutional self-study and an evaluation by a team of outside peer observers assigned
by the Commission, an institution is guided by well-defined and appropriate goals; that
it has established conditions and procedures under which its goals can be realized; that it
is accomplishing them substantially; that it is so organized, staffed, and supported that it
can be expected to continue to do so; and that it meets the standards of the Middle States
Association. The American College has been accredited since 1978.
The American College does not discriminate on the basis of race, religion, sex,
handicap, or national and ethnic origin in its admissions policies, educational programs
and activities, or employment policies.
The American College is located at 270 S. Bryn Mawr Avenue, Bryn Mawr,
PA 19010. The toll-free number of the Office of Professional Education is (888)
AMERCOL (263-7265); the fax number is (610) 526-1465; and the home page address
is theamericancollege.edu.
iv
TheAmericanCollege.edu
Through The American College’s website, theamericancollege.edu, students can
access information on a variety of topics including:
•
General Information on Courses: Course descriptions, chapter or assignment
topics, and required study materials for all Solomon S. Huebner School, Richard
D. Irwin Graduate School, and the Center for Financial Advisor Education
courses are listed.
•
The American College Online: The College provides online study materials
for this course, including an interactive version of the sample exam, designed to
be used in conjunction with the printed study materials to enhance the learning
experience. This material is provided only through password access to students
registered for this course. To obtain a password, students should complete the
e-mail address section of the course registration form; The College will send the
student a password and instructions on how to access these materials.
•
Course Pages/Updates: New developments in the subject area, important study
points, links to other useful governmental and organizational websites, and errata
in course materials are included. All information is accessible with a password
at blackboard.theamericancollege.edu.
•
Course Registration Procedures: Secure online registration plus registration
forms that can be printed and faxed/mailed to The College are available.
•
Examinations on Demand (EOD) Testing Procedures: The policies and
requirements for EOD testing, as well as links to lists of our more than 3,000
test centers, are provided.
•
Educational Policies and Procedures: The education, experience, ethics,
continuing education, and transfer of credit requirements for The College’s
designation and graduate programs are explained.
v
Contents
The American College ............................................................................ iii
TheAmericanCollege.edu..........................................................................v
Table of Contents....................................................................................vii
Introduction ............................................................................................xi
Course Overview ...................................................................................xiv
Important Notes .....................................................................................xv
1
The Financial Planning Process ................................... 1-1
Overview.............................................. 1-1
Preparing for Class 1............................ 1-1
Study Tips for Class 1.......................... 1-2
Post-Class: Test Your
Comprehension.................. 1-2
Quick Quiz ........................................... 1-3
Key Terms and Concepts ..................... 1-4
I FINALLY GET IT—FINANCIAL
PLANNING, THAT
IS........................................ 1-6
ICE BREAKER: HOW TO START
THE CONVERSATION
WITH CLIENTS ............... 1-8
Class 1 Video Graphics.......................1-11
2
Communicating Effectively with Clients..................... 2-1
Overview.............................................. 2-1
Preparing for Class 2............................ 2-1
Study Tips for Class 2.......................... 2-2
Post-Class: Test Your
Comprehension.................. 2-2
Quick Quiz ........................................... 2-3
Key Terms and Concepts ..................... 2-5
Class 2 Video Graphics........................ 2-7
vii
3 Ethics, Professionalism, and Practice
Standards.............................................................. 3-1
Overview.............................................. 3-1
Preparing for Class 3............................ 3-1
Important Note ..................................... 3-1
Study Tips for Class 3.......................... 3-2
Post-Class: Test Your
Comprehension.................. 3-2
Quick Quiz ........................................... 3-3
Key Terms and Concepts ..................... 3-4
Class 3 Video Graphics........................ 3-7
4
Client Attitudes Toward Risk ....................................... 4-1
Overview ............................................. 4-1
Preparing for Class 4............................ 4-1
Important Note ..................................... 4-2
Study Tips for Class 4.......................... 4-2
Post-Class: Test Your
Comprehension.................. 4-3
Quick Quiz ........................................... 4-4
Key Terms and Concepts ..................... 4-6
Class 4 Video Graphics........................ 4-9
5 Gathering Data and Preparing Financial
Statements ............................................................ 5-1
Overview.............................................. 5-1
Preparing for Class 5............................ 5-1
Study Tips for Class 5.......................... 5-2
Post-Class: Test Your
Comprehension.................. 5-2
Quick Quiz ........................................... 5-3
Key Terms and Concepts ..................... 5-4
Class 5 Video Graphics........................ 5-7
6
Time Value of Money: Basic Concepts and
Applications.......................................................... 6-1
Overview.............................................. 6-1
Preparing for Class 6............................ 6-1
Study Tips for Class 6.......................... 6-2
viii
Post-Class: Test Your
Comprehension.................. 6-3
Quick Quiz ........................................... 6-4
Key Terms and Concepts ..................... 6-6
Class 6 Video Graphics........................ 6-9
7
Time Value of Money: Advanced Concepts and
Applications.......................................................... 7-1
Overview.............................................. 7-1
Preparing for Class 7............................ 7-1
Study Tips for Class 7.......................... 7-1
Post-Class: Test Your
Comprehension.................. 7-2
Quick Quiz ........................................... 7-3
Key Terms and Concepts ..................... 7-5
Class 7 Video Graphics........................ 7-7
8
Financial Planning for Special Circumstances ........... 8-1
Overview.............................................. 8-1
Preparing for Class 8............................ 8-1
Study Tips for Class 8.......................... 8-2
Post-Class: Test Your
Comprehension.................. 8-2
Quick Quiz ........................................... 8-3
Key Terms and Concepts ..................... 8-4
Class 8 Video Graphics........................ 8-7
9
The Regulation of Financial Advisors ......................... 9-1
Overview.............................................. 9-1
Preparing for Class 9............................ 9-1
Study Tips for Class 9.......................... 9-2
Post-Class: Test Your
Comprehension.................. 9-2
Quick Quiz ........................................... 9-3
Key Terms and Concepts ..................... 9-4
Class 9 Video Graphics........................ 9-7
10
The Legal and Economic Environment of Financial
Institutions.......................................................... 10-1
ix
Overview............................................ 10-1
Preparing for Class 10........................ 10-1
Study Tips for Class 10...................... 10-1
Post-Class: Test Your
Comprehension................ 10-2
Quick Quiz ......................................... 10-3
Key Terms and Concepts ................... 10-4
Class 10 Video Graphics.................... 10-7
11
Facilitator Instructions: Financial Planning for
Special Circumstances....................................... 11-1
Overview.............................................11-1
Preparing for Class 11.........................11-1
Study Tips for Class 11 .......................11-2
Post-Class: Test Your
Comprehension.................11-2
Quick Quiz ..........................................11-3
Key Terms and Concepts ...................11-4
Class 11 Video Graphics .....................11-7
12
Review......................................................................... 12-1
Preparing for Review Class ............... 12-1
13
HS 300 Practice Examination................................... 13-1
Answer Key for Practice
Examination..................... 13-7
Explanation of Answers to Practice
Exam Questions............... 13-8
14
HS 300 SAMPLE EXAMINATION......................... 14-1
ANSWER KEY FOR SAMPLE
EXAMINATION ........... 14-22
Answers to Sample Examination
Questions ....................... 14-26
x
Introduction
Overview
1. Financial Planning: Process and Environment Video Course
•
This is one of six courses necessary to qualify to take the Certified
Financial Planner (CFP®) comprehensive exam.
•
This course is offered at your workplace, making it easier to attend classes
and receive facilitated support.
•
It combines classroom and interactive multimedia technologies that
promote learning through practical application of concepts, which, in
turn, develop client advisory skills.
Course Design
1. Classroom Instruction
•
Twelve sessions over 12 weeks
•
Dynamic lectures with optimal graphic support to enhance teaching
•
Instructional videos, which include interviews with leading experts, bring
the course content to life
•
Quizzes and exercises given weekly as a means for you to assess your
understanding of the class content
•
The course is interactive and includes ongoing forum communications
with College faculty and other students
2. Course Materials
•
Textbook—Financial Planning: Process and Environment, Third
Edition, published by The American College Press
xi
•
Student workbook designed to be used prior to, during, and after class and
containing weekly assignments and study tips
3. Attendance
•
Class attendance is very important. To obtain the maximum benefit of
this uniquely designed course, you need to attend the regularly scheduled
class with other students. This increases interactivity and enhances the
learning experience.
4. Time Commitment
•
An average of 10 to 12 hours of study per week is required to successfully
complete the course.
– Classroom time—1 to 2 hours each week
– Independent study—8 to 10 hours each week to complete the
following:
♦ Textbook and student workbook readings that average 49
pages per week. In order to learn the material, students
will need to read the text at least twice. That could require
approximately 4 hours of their time, depending on the
difficulty of content and number of pages.
♦ Test taking that prepares students for the course
examination. A multiple-choice sample exam is available
in the Supplement to the 3d edition of Financial Planning:
Process and Environment. There is another sample exam
available online. The sample exams improve students’
familiarity with the course content.
5. Support (to ensure your success)
•
The American College offers a wide variety of online support services.
– Online
community:
The
American
College
forum
for
course-related
questions
at
<http://blackboard.theamericancollege.edu>
– Other information, such as how to use a financial calculator and
how to calculate time value of money (TVM tutorial and practice),
available at The American College’s web site
xii
•
On-site facilitators available to assist with course information, planning,
and content
•
Fellow students and advisors (mentors) available to answer questions
about course content
6. Audio Review (as a supplementary study aid)
•
An audio review provides a conversational overview of course material.
This audio review is intended as an additional study aid to augment regular
course materials.
Welcome to The American College’s Horizons Video Course...
where you, your company, and The College are working together, as partners,
to guarantee your success and professional development.
xiii
Course Overview
HS 300 Financial Planning: Process and Environment
Financial Planning: Process and Environment
Class
Topics
Chapter(s)
Pages
1
The Financial Planning
Process
1
1.1–1.31
I Finally Get
It—Financial Planning,
That Is
Student Workbook
Class 1
Ice Breaker: How to
Start the Conversation
with Clients
Student Workbook
Class 1
2
Communicating
Effectively with Clients
2
2.1–2.38
3
Ethics, Professionalism,
and Practice Standards
3
3.1–3.43
4
Client Attitudes Toward
Risk
4
4.1–4.44
5
Gathering Data and
Preparing Financial
Statements
5
5.1–5.54
6
Time Value of Money:
Basic Concepts and
Applications
6
6.1–6.51
7
Time Value of Money:
Advanced Concepts and
Applications
7
7.1–7.50
8
Financial Planning
Applications
8
8.1–8.64
9
The Regulation of
Financial Advisors
9
9.1–9.38
10
The Legal and
Economic Environment
of Financial Institutions
10
10.1–10.31
11
Financial Planning for
Special Circumstances
11
11.1–11.34
1–11
1.1–11.34
Review
Financial Planning:
Process and
Environment
xiv
Important Notes
Exam Scheduling: For information on testing center locations and to schedule exam
appointments, call or go to
VUE
866-392-6822
vue.com/tac
Participate in an online community by accessing The American College’s "Forum"
at blackboard.theamericancollege.edu. Post and answer course-related questions to gain
valuable subject matter insight from peers and classmates.
xv
The Financial Planning Process
1
Overview
The primary goal of this class is to introduce the six-step financial planning process.
This process is the foundation on which the financial planning profession is built,
and it provides a framework around which financial planning practitioners develop
comprehensive plans for their clients. In support of the primary goal, three different
approaches to financial planning are discussed along with the subjects that should
be included in a comprehensive plan. Also discussed is how a financial plan can be
developed either by using the steps in the financial planning process or by using the
financial planning pyramid.
The last part of the class analyzes the trends that are creating opportunities in the
financial planning marketplace. It then identifies the principal financial goals/concerns
of most consumers along with obstacles that prevent them from being achieved. The nine
assignments that follow this one broadly explore the basic tools and techniques used in
financial planning as well as the environment in which financial planning practitioners
operate.
Preparing for Class 1
√ Read chapter 1 in Financial Planning: Process and Environment, AT LEAST
TWICE.
√ Read student workbook readings "I Finally Get It—Financial Planning, That
Is" and "Ice Breaker: How to Start the Conversation with Clients," beginning
on page 1-6.
√ Answer the review questions and self-test questions at the end of the chapter.
√ Study the key terms and concepts listed in the textbook.
√ Review the Study Tips for Class 1 in the student workbook, and the video
graphics starting on page G1-1.
√ Participate in the online community for this course by accessing AC Online
at <http://blackboard.theamericancollege.edu>. Explore the channels for this
course on AC Online including: Course Info, Modules, Exams, Forums, and
1-2
Tools. Stop by the Student Lounge, on the Forum channel to post a note to other
students in the course, or Office Hours to post a note for the course instructor.
Contact Student Services if you don’t have your user ID or password.
Study Tips for Class 1
•
As pointed out in the assigned reading, financial planning is a process.
– Be sure to understand that financial planning can be done at any of
three levels: with a single purpose, with a multiple purpose, or with a
comprehensive focus.
– It is the process used (not the range of concerns addressed) that
determines whether the activity is really financial planning.
•
•
Know the six steps in the general model of the financial planning process
including the kinds of activities involved in each step
Know the breakdown of life stages and how they can influence the client’s
financial planning needs.
Post-Class: Test Your Comprehension
√ Complete the Quick Quiz for class 1.
√ Complete the Key Terms and Concepts matching exercise for class 1.
Student Workbook for HS 300
1-3
Quick Quiz
Circle your responses:
T F
1. Most advocates of comprehensive financial planning provided
entirely on a fee-for-service basis recognize that this type of
financial planning is practical only for a small, affluent clientele.
T F
2. There now appears to be general agreement among financial
services practitioners on the question of what constitutes financial
planning.
T F
3. A comprehensive financial plan can be developed for a client
incrementally during the course of several appointments with the
client.
T F
4. The final step in the financial planning process is plan
implementation.
T F
5. The advisor’s responsibility in step 4 of the financial planning
process is to see that the plan is implemented.
T F
6. A survey conducted by the CFPŠ Board in 1999 identified estate
planning as the most popular financial planning specialization
engaged in by financial planning advisors.
T F
7. The phase of the financial life cycle that an individual is currently
in strongly influences the priority given to the goals for each of the
planning areas.
T F
8. Every financial plan is developed around information gathered
during a fact-finding process.
T F
9. A trend creating opportunities for financial planning advisors is the
falling median age of Americans.
T F
10. According to a study of baby boomers conducted by The Allstate
Corporation in 2001, only about 10 percent of them will continue
to work during retirement.
Answers to Quick Quiz
1-T, 2-F, 3-T, 4-F, 5-F, 6-F, 7-T, 8-T, 9-F, 10-F
Student Workbook for HS
1-4
Key Terms and Concepts
Match the key term or concept to the most accurate definition.
a. baby-boom
generation
__________ 1. broad-based measurement of changes in stock-market
conditions based on the average performance of 500
widely held common stocks
b. financial life cycle
__________ 2. client’s psychological attitude toward his or
her willingness to expose financial assets to the
possibility of loss for the chance to achieve greater
financial gain
c. financial plan
__________ 3. financial planning process that uses the client’s
age and assets as a reference point in developing a
financial plan
d. financial planning
pyramid
__________ 4. plan designed to carry a client from his or her present
financial position to the attainment of financial goals
e. financial risk
tolerance
__________ 5. a widely accepted approach for developing a
comprehensive financial plan over time that
prioritizes financial goals by categorizing them into
three levels
f. life-cycle financial
planning
__________ 6. generation faced with financing their children’s
education and aiding their parents while trying to
save for their own retirement
g. multiple-purpose
approach
__________ 7. approach to financial planning that occurs when an
advisor follows the financial planning process to
develop a plan that solves two or more financial
problems for a client
h. sandwiched
generation
__________ 8. approach to financial planning that occurs when an
advisor follows the financial planning process to
develop a plan that solves a single financial problem
for a client
i. single-purpose
approach
__________ 9. the five distinct phases in an individual’s financial
life or career
j. Standard & Poor’s
500 Index
__________ 10. generation of people born between 1946 and 1964
Student Workbook for HS 300
1-5
Answers to Key Terms and Concepts Matching
1.
2.
3.
4.
5.
6.
7.
8.
9.
10.
j
e
f
c
d
h
g
i
b
a
Student Workbook for HS 300
1-6
I FINALLY GET IT—FINANCIAL PLANNING, THAT IS
David J. Drucker, MBA, CFP, a financial advisor since 1981, sold his practice 20
years later to write, speak, and consult with other advisors. His newest book—Tools
& Techniques of Practice Management—was released by National Underwriters in
December 2004 and will be soon followed by his other new book—The One Thing...
You Need to Know from Each of Industry’s Most Influential Coaches, Consultants
and Visionaries Please visit www.daviddrucker.com for more information. You can
reach Dave at [email protected]. Please visit www.daviddrucker.com for more
information. You can reach Dave at [email protected]. The views expressed in
these articles are the author’s. They do not necessarily reflect the views of Morningstar.
Feedback about these articles may be sent to [email protected] 11/3/04.
Reprinted with permission from Morningstar, Inc.
Last month, I was traveling all around the country with my associate Laron "D"
Shannon interviewing some of the foremost personalities who make it their business
to advise us advisors. We lived out of suitcases for three weeks gathering material for
our soon-to-be-self-published book, tentatively titled The One Thing...You Need to Know
from Each of Industry’s Most Influential Coaches, Consultants and Visionaries. As I sat
before each industry guru, I heard one theme over and over again, and it’s made me
realize I was doing it all wrong.
As much as I understood the value of having a good relationship with each of
my clients, my entire financial-planning process was geared, if not to suppress the
relationship, at least to delay it unnecessarily. The relationship with one’s client can be
speeded up, and most of today’s processes that go under the general heading of "life
planning" are designed to show us how to do that. But, ironically, the curricula we
follow to become credentialed in the art and science of financial planning miss the mark.
But I’m getting ahead of myself. Let’s start at the beginning, which, for most of
us, is either the CFP curriculum, or a wirehouse training regimen. What does the CFP
curriculum prepare us to do? As my friend Shannon puts it, we’re all taught to stand
like nurses with clipboards taking down the client’s quantitative data. This is the very
first thing we do in the relationship, as if it were the most critical task we could possibly
perform. This is a failure of the system, of course. Wirehouses fail, too, but in a much
more institutional way that we’ll discuss later.
I remember my own process of working with a new client. A couple would come to
my office and receive the usual comfortable chair and favorite beverage. For about an
hour, we would talk about why they’d come in. Sure, we’d get to some of their personal
issues, the kind that people discuss in the beginning of a real relationship, but it was
understood by all parties that the point was to pay lip service to the personal stuff en
route to discussing their money. And I have no doubt that my process is what furthered
my client’s understanding. I could just as easily have taken the process in a different
Student Workbook for HS 300
1-7
direction, but I didn’t know any better. I was learning gradually through experience, but
what a boost it would have been if the CFP curriculum had more effectively prepared
me.
And what was that experience? It was twofold: first, what I was discovering about
my most successful client engagements and how I’d achieved them, and second, what
I was hearing from the profession about this thing called "life planning." My favorite
and best (what many in the industry call "ideal") clients are the Millionaire Next Door
types. These are folks who understand the transitory nature of material things, and for
that reason, they don’t buy elaborate homes or fancy cars. Instead, they save their money
and place greater value on family, church (or synagogue), and community.
Student Workbook for HS 300
1-8
ICE BREAKER: HOW TO START THE
CONVERSATION WITH CLIENTS
by David J. Drucker. 11/3/04 Reprinted with permission from Morningstar, Inc.
In the first part of this series, we discussed how—if at all—we form relationships with
our clients, and what the place of those relationships is in the overall client engagement.
If you accept that a relationship with any client you want to serve for a long, long time
must be based on caring, trust, and honesty, then you need to examine how you are
creating those relationships. Do you have a systematic process? Or is your process
perhaps impeding the relationship, as mine once was?
The notion of having a real relationship with one’s clients isn’t new. Nor are the
means of getting to that place. It’s just that the industry gets so much advice from so
many people that the message—an amazingly common and simple one—gets obscured.
So let’s see if I can clarify it for you.
In Part I, we said most advisors do things backward, and they can at least partially
blame it on the College for Financial Planning. The CFP six-step process teaches us
to get the facts first and hear the client later. Oh sure, we are told to investigate the
client’s goals. But isn’t it interesting how, in the context of the six-step process where
we gleefully move from one box of our data gathering form to the next, the goals almost
always seem to be "I want to retire comfortably" or "I want to be able to send the kids to
a good university."
These are important goals, but the fact that they come up for almost every client
should tell you something. It should tell you that clients aren’t being asked to think very
deeply about the real possibilities available to them in their lives. Bob Veres, publisher
of Inside Information newsletter, says it all the time. Our profession has the opportunity
to be the most influential professional service throughout the entire economy. Who else
deals with peoples’ money as well as their psyches? Their banker doesn’t. Nor does
their psychologist. (And let’s get one thing straight before we go any further, for all of
you out there whining about how we’re not trained to be headshrinkers: That’s not what
anyone’s talking about; we’re just talking about the same relationship you have with a
close friend to whom you listen when he needs to talk.)
So it would seem we need to be employing a different process and asking different
questions. In short, we need to be having a meaningful conversation with our clients as
the first thing we do in the relationship. Why? Because it’s that conversation from which
all meaningful planning emanates.
Student Workbook for HS 300
1-9
Don’t take my word for it. Look at people as diverse as Deena Katz, president of the
Coral Gables, Fla., firm of Evensky, Brown & Katz, and author of Tools and Templates for
Your Practice; Bill Bachrach, creator of the Values-Based Selling Academy and author of
Values-Based Selling: The Art of Building High-Trust Client Relationships for Financial
Professionals; and Mitch Anthony, founder of Advisor Insights, Inc. and author of The
Financial Professional’s Storybook and The New Retirementality.
Katz tells me it’s not even her job to hold the proverbial clipboard and take down
all the client’s particulars. It’s her job to sit down with the client and get to know him
or her. You know the routine. You do it every time you go to a party. A party is full
of people who are all just one or two degrees of separation removed from you—that
is, they all know the host or someone else the host has invited to the party. It’s sort of
like a client who is referred to you by someone each of you knows. You sit down with
no time pressures, perhaps with a relaxing beverage, and you just talk. In most cases,
you seek out commonalities to see if there’s the basis for some kind of relationship going
forward. The commonalities aren’t based on the fact that one of you has money and other
one manages money. They’re based on personal characteristics and values. Interpreting
from my recent interview with Deena, I think that’s pretty close to what she means by
the conversation.
Bachrach’s message takes a few more steps toward formalization of the conversation
concept. Bachrach says you can’t earn a client’s trust without having a conversation
about his or her values. In fact, he would say the values conversation must precede the
goals conversation, and the goals conversation must precede the money conversation.
Instead, most of us pay lip service to the client’s goals and charge full steam ahead toward
the money.
And Anthony takes it even further. According to Anthony, the client relationship
proceeds from an understand of the client’s history, transitions, principles, and goals. In
other words, where has the client been and what has she done; what changes has she
navigated in her lifetime; what are her beliefs; and what does she want to accomplish.
Again, as with Katz’s and Bachrach’s processes, the goals and the money come last.
Whatever strategies or investments you throw at the client without first understanding
his or her history, transitions, principles, and goals are likely to miss the mark.
So how does this change the time-honored planning engagement? Doesn’t it turn it
on its head? We do our talking now, and we put on the green eyeshades later. We talk
much longer than we used to, and we get more information about the client.
Do I hear any objections? Yes, there in the back. What if the client doesn’t want
to do all that stuff? What if he says he just wants an investment manager? That’s easy.
If that’s the line of work you want to be in, then take him and forget all this nonsense.
Student Workbook for HS 300
1-10
If you want real, long-term client relationships, though, tell him to go elsewhere (do it
nicely, though; give him a few referrals).
Did I see another hand raised? Over here. You still don’t quite get the point of looking
at things like history and transitions. Well, when you get this personal with a client,
you’re more likely to find out what his real goals are—the ones that go beyond retirement
and education planning, goals like, "I hate my job and wish I could quit and do such and
such but I never thought I could afford it." You help clients do what they always really
wanted to do with their life, and you’ve got clients you couldn’t get rid of if you pushed
them off a cliff.
We have time for one more question. Didn’t I say there’s a connection here with life
planning? You mean you haven’t figured it out yet? The conversation is life planning. It
goes by many different names, but at its core, it means being real, asking the questions
a caring friend would ask, and helping clients figure out what’s really important to them
(unlike one of my first clients who’s primary goal in life was to figure out how to afford
to give her daughter a $50,000 wedding).
Bottom line: You can hire someone to work the clipboard, but can you work the
relationship?
Student Workbook for HS 300
_________________________G1-1_________________________
HS 300 Financial Planning:
Process and Environment
Chapter 1:
The Financial Planning Process
1
Course Overview
2
Horizons Instructor
• Craig Lemoine, CFP®
– Assistant Professor of Financial Planning
at The American College
– Other professional endeavors
• Academic Journals
• 8 Years of Life Insurance Industry
Experience
• Financial Planner
3
Student Workbook for HS 300
_________________________G1-2_________________________
Course Overview
HS 300 Financial Planning:
Process and Environment
• First course in
– CFP certification curriculum
– ChFC® designation curriculum
• Elective in the CLU® designation curriculum
4
CFP Topics List
General principles of financial planning
Insurance planning and risk management
Employee benefits planning
Investment planning
Income tax planning
Retirement planning
Estate planning
5
Horizons HS 300 Overview
• Facilitator guided
• Ten classes covering ten chapters
• Review class
• Improved success rate
6
Student Workbook for HS 300
_________________________G1-3_________________________
Course Resources
•
•
•
•
•
•
•
Textbook
DVDs
Facilitator’s guide
Supplement
Audio review
Flash cards (optional)
Review book (optional)
7
Other Course Resources
• AC Online
http://blackboard.theamericancollege.edu
– Announcements
– Forum
– Interactive exercises
– Assessment
• Quizzes
• Practice exams
8
Pretest Question
Is financial planning a
product or a process?
9
Student Workbook for HS 300
_________________________G1-4_________________________
Learning Objectives
10
Learning Objectives
1. Explain the six steps in the financial planning
process.
2. Describe three different approaches to
financial planning, and identify several areas
of specialization in which
advisors concentrate their activities.
11
Learning Objectives
3. Identify the subjects that should be included
in a comprehensive financial plan.
4. Describe what is meant by a person’s
financial life cycle, and explain how it relates
to life-cycle financial planning.
12
Student Workbook for HS 300
_________________________G1-5_________________________
Learning Objectives
5. Explain how a financial plan can be
developed around the steps in the financial
planning process.
6. Explain how a financial plan can be
developed using the financial planning
pyramid.
13
Learning Objectives
7. Explain the trends that are creating
opportunities in the financial planning
marketplace.
14
Learning Objectives
8. Identify the principal financial goals/
concerns of most consumers, and
describe three major obstacles that
prevent them from achieving these goals.
15
Student Workbook for HS 300
_________________________G1-6_________________________
Financial Planning Defined
CFP Board definition:
“Financial planning is the process of meeting
your life goals through the
proper management of your finances.”
16
Financial Planning Defined
Text definition:
“Financial planning is a process that
ascertains the client’s financial goals
and develops a plan for achieving the client’s
goals.”
17
Six Steps in the
Financial Planning Process
18
Student Workbook for HS 300
_________________________G1-7_________________________
Six Steps in the
Financial Planning Process
1. Establishing and defining the clientplanner relationship
2. Gathering client data, including goals
19
Six Steps in the
Financial Planning Process
3. Analyzing and evaluating your financial
status
4. Developing and presenting financial
planning recommendations and/or
alternatives
20
Six Steps in the
Financial Planning Process
5. Implementing the financial planning
recommendations
6. Monitoring the financial planning
recommendations
21
Student Workbook for HS 300
_________________________G1-8_________________________
The General Model
• Step 1: Establish and define advisorclient relationship
– Build relationship
– Explain process
– Describe products and/or services
– Clarify responsibilities
– Disclose advisor’s background,
philosophy, method of compensation
22
The General Model
• Step 2: Determine goals and gather data
– Encourage client to determine and
prioritize goals
– Gather client information
• Fact-finding forms
• Questionnaires
• Counseling
• Examination of documents
23
The General Model
• Step 3: Analyze and Evaluate the Data
– Analyze and evaluate client’s financial
condition relative to achieving goals
– Revise goals if necessary
24
Student Workbook for HS 300
_________________________G1-9_________________________
The General Model
• Step 4: Develop and present a plan
– Design recommended strategies to
achieve goals
• Include alternatives
– Use outside experts as needed
– Present plan
– Obtain client approval
25
The General Model
• Step 5: Implement the plan
– Help client acquire products and services
– Establish accounts
– Use outside experts as needed
26
The General Model
• Step 6: Monitor the plan
– Evaluate performance of implementation
vehicles
– Review changes in client’s circumstances
and the financial environment
– Revisit other steps as necessary
27
Student Workbook for HS 300
_________________________G1-10_________________________
Financial Planning in Action
28
Discussion Break
The six steps of financial planning convey
a fairly linear approach to planning.
Discuss how you could adapt this process
to be more free wheeling while still
covering the six steps.
29
Different Approaches to
Planning
30
Student Workbook for HS 300
_________________________G1-11_________________________
Different Approaches to Planning
• Single-purpose
– How do I invest my 401(k)?
• Multi-purpose
– How can I meet my goals to retire at 60
and fund my children’s education?
31
Different Approaches to Planning
• Comprehensive
– How can we manage our wealth to meet
our needs for current and future income
including the income needs of our planned
estate?
32
Tim
e
an
dW
ea
lth
Financial Planning Pyramid
Stage 3
Stage 2
Managing Retirement
and the Estate
Growing Investments
Guarding Against
Uncertainty
Stage 1
33
Student Workbook for HS 300
_________________________G1-12_________________________
Life-Cycle Financial Planning
34
Life-Cycle Financial Planning
•
•
•
•
Early career (ages 25–35)
Career development (ages 35–50)
Peak accumulation (ages 50–62)
Preretirement (3–6 years before planned
retirement)
• Retirement (ages 62–66+)
35
Financial Planning Needs
• Up and coming (ages 20–39)
– 28 percent have a written financial plan
– Most tolerant of risk
• Mid-life (ages 40–54)
– 39 percent have written financial plan
– Low to moderate risk tolerance
36
Student Workbook for HS 300
_________________________G1-13_________________________
Financial Planning Needs
• Retirement cusp (ages 55–69)
– 47 percent have written financial plan
– Most likely to have financial professional as
advisor
37
Top 10 Reasons People Begin
Financial Planning
38
Top 10 Reasons People Begin
Financial Planning
10. Sheltering income from taxes
(25 percent)
9. Generating current income
(26 percent)
39
Student Workbook for HS 300
_________________________G1-14_________________________
Top 10 Reasons People Begin
Financial Planning
8. Providing insurance protection
(29 percent)
7. Accumulating capital (31 percent)
6. Building a college fund (32 percent)
40
Top 10 Reasons People Begin
Financial Planning
5. Planning vacation/travel (34 percent)
4. Managing/reducing current debt
(34 percent)
3. Building an emergency fund
(40 percent)
41
Top 10 Reasons People Begin
Financial Planning
2. Purchasing/renovating home (41 percent)
1. Building a retirement fund (82 percent)
Source: CFP Board of Standards, Inc.
2004 National Consumer Survey
42
Student Workbook for HS 300
_________________________G1-15_________________________
Planning’s Partners
Client
Attorney
Accountant
Financial
Advisor
Insurance
Agent
Registered
Representative
43
Financial Planning in Action
44
Planning Trends
•
•
•
•
•
•
•
Aging population
Increase in dual income households
Market volatility
Technological change
Increasing sophistication of consumers
Financial planning profession still growing
International economy
45
Student Workbook for HS 300
_________________________G1-16_________________________
Building the Plan
46
Building a Plan
•
•
•
•
•
Goals
Budget
Net worth
Risk tolerance
Investments
47
Using the Pyramid Model
• Liquidity and security
• Capital accumulation
• Income and purchasing power
48
Student Workbook for HS 300
_________________________G1-17_________________________
What’s in a Comprehensive Plan?
• Goals, financial statements, risk tolerance,
advisor and client responsibilities
• Insurance and risk management
• Employee benefits
49
What’s in a Comprehensive Plan?
• Investment planning
• Income tax planning
• Retirement planning
• Estate planning
50
Monitoring the Plan
51
Student Workbook for HS 300
_________________________G1-18_________________________
Monitoring the Plan
• Determine level of service
• Perform periodic reviews
– Update financial situation
– Analyze variances
– Review goals and revise as needed
– Track performance of investments
52
Monitoring the Plan
• Establish need for rebalancing
– Time diversification
– Risk modeling
53
Obstacles to Success
54
Student Workbook for HS 300
_________________________G1-19_________________________
Obstacles to Success
• Procrastination
– Works against the miracle of compound
interest
• Deficit spending
– There’s nothing to save for tomorrow
55
Obstacles to Success
• Gaps in understanding
– Keeps client from visualizing planning
solutions
• Unmanaged expectations
– Unrealistic goals meet unrealized returns
56
Goal Tending
• Consumption/investment decisions
– Budgeting allows client to manage cash
flow
– Net worth keeps score (own-owe)
57
Student Workbook for HS 300
_________________________G1-20_________________________
Goal Tending
• Asset allocation balances risk/return tradeoffs
when investing for the future
– Liquidity
– Return
– Risk
58
Where the Industry Is Going
59
Where the Industry Is Going
• Aging baby boomers
• Sandwich generation
• Kids’ college fund / our retirement / aging
parents
• Investing inheritances
60
Student Workbook for HS 300
_________________________G1-21_________________________
Where the Industry Is Going
• Social Security concerns
• Growth in planners
– BLS Occupational Survey expects 21–35
percent growth in field over next 10 years
61
Chapter One Review
• Planning is a six step process
• Single-purpose, multi-purpose, and
comprehensive approaches to plans
– Only comprehensive incorporates all
seven topic areas
62
Review
• Looking back (monitoring) can help you move
forward
• Overcoming obstacles and maintaining focus
on client goals improves odds of success
63
Student Workbook for HS 300
_________________________G1-22_________________________
Review (Continued)
• Population trends
– Demographics: statistical market
characteristics
• age
• sex
• income
• educational level
64
Review (Continued)
• Population trends
– Psychographics: classification of
population groups according to
psychological variables
• attitudes
• values
• fears
Source: Merriam-Webster’s Unabridged Dictionary
65
Review (Continued)
• Projected growth in profession
66
Student Workbook for HS 300
Communicating Effectively with Clients
2
Overview
Class 2 focuses on learning how to communicate effectively with clients throughout
the financial planning process. In step 1 of the process, the advisor’s objective is to
establish a relationship with clients that is conducive to planning. Communicating
effectively with clients is critical to that relationship.
Communication is the single most critical skill that an advisor brings to a financial
planning session. Ineffective communication is an obstacle to a strong advisor-client
relationship. The failure of clients and advisors to communicate fully and clearly with
each other can result in improperly identified financial goals and the formulation of
inappropriate planning strategies. The result for the client is not being able to achieve his
or her financial goals. The communication process is the starting point from which the
advisor helps the client to establish financial goals and then designs a plan to achieve
those goals. Simply put, effective communication between the advisor and client is
crucial to the financial planning process.
As indicated, the purpose of this assignment is to examine the communication process
as it typically exists in an advisor-client relationship. This requires reviewing the types
of structured communication used in financial planning, the attributes of the effective
advisor, basic communication principles, and attending and listening skills. By reviewing
these and other aspects of the communication process, it is hoped that this class will help
financial advisors become better communicators with their clients.
Preparing for Class 2
√ Read chapter 2 in Financial Planning: Process and Environment, AT LEAST
TWICE.
√ Answer the review questions and self-test questions at the end of the chapter.
√ Study the key terms and concepts listed in the textbook.
√ Review the Study Tips for Class 2 in the student workbook, and the video
graphics starting on page G2-1.
2-2
√ Participate in the online community for this course by accessing AC Online
at <http://blackboard.theamericancollege.edu.> Explore the channels for this
course on AC Online including: Course Info, Modules, Exams, Forums, and
Tools. Stop by the Student Lounge, on the Forum channel to post a note to other
students in the course, or Office Hours to post a note for the course instructor.
Contact Student Services if you don’t have your user ID or password.
Study Tips for Class 2
•
•
•
•
•
•
Review the first four key terms as a way of highlighting the differences
between interviewing (both directive/structured and nondirective/unstructured),
counseling (help-giving), and advising (giving an expert opinion; advice should
be tentative, however, because the best advice is self-advice).
Note that client resistance is almost always present in a financial counseling
session.
Learn the qualities of an effective counselor. Be aware that the counselor must
know his or her own values to avoid trying to impose them on the client.
Review the 10 basic principles of communication theory.
Be aware of the key aspects of attending and listening skills. Thinking in terms
examples of practical applications may help in identifying several types of
responses made during active listening.
Be sure you recognize the difference between types of interviews. The
counselor may follow the client’s lead by (a) summarizing, (b) clarifying, or (c)
restating the content, or the counselor may lead the client by (a) explaining, (b)
interpreting, (c) encouraging or reassuring, and (d) advising or suggesting.
Post-Class: Test Your Comprehension
√ Complete the Quick Quiz for class 2.
√ Complete the Key Terms and Concepts matching exercise for class 2.
Student Workbook for HS 300
2-3
Quick Quiz
Circle your responses:
T F
1. Communication is the single most critical skill that a counselor
brings to a counseling session.
T F
2. In the nondirective interview, the interviewer controls the pace and
content to be covered.
T F
3. Help-giving is an integral aspect of the counseling process.
T F
4. In the counseling process, questions are the predominant form of
counselor response.
T F
5. In a communication setting, structuring serves to establish the
format and subject matter of the interaction that is to follow.
T F
6. Qualities of an effective counselor include unconditional positive
regard, accurate empathy, genuineness, and self-awareness.
T F
7. People’s values, while deeply internalized, are not immutable,
because they can and do change.
T F
8. Communication is learned through experience.
T F
9. Nonverbal behavior is rarely as important as the client’s words in
communicating the client’s message to the financial counselor.
T F
10. Effective communication occurs when the receiver interprets the
sender’s message in the same fashion as the sender intended it.
T F
11. In a financial counseling session, the counselor and client can never
not communicate.
T F
12. Facial expressions and gestures are nonverbal clues, whereas voice
tone and voice pitch are verbal signs.
T F
13. Active listening relies most heavily on the use of leading responses
made by the counselor.
Student Workbook for HS 300
2-4
T F
14. Ideally, questions posed by a financial counselor should be
open-ended and should call for more than just a yes or no response
on the part of the client.
T F
15. Communication can often be improved if we convert some of our
questions into statements.
Answers to Quick Quiz
1-T, 2-F, 3-T, 4-F, 5-T, 6-T, 7-T, 8-T, 9-F, 10-T, 11-T, 12-F, 13-F, 14-T, 15-T
Student Workbook for HS 300
2-5
Key Terms and Concepts
Match the key term or concept to the most accurate definition.
a. accurate empathy
__________ 1. using one’s body to communicate
b. active listening
__________ 2. the act of putting together a speaker’s words and
nonverbal behaviors to get the essence of the
communication being sent
c. clarifying response
__________ 3. predictable patterns of behavior that people
display and that can be observed
d. closed-ended question
__________ 4. in an advisor-client relationship, behavior by
the client being counseled that impedes the
counseling process
e. explanatory response
__________ 5. a type of understanding response associated with
active listening that enhances communication
f. nondirective interview
__________ 6. a type of structured communication that allows
both the interviewer (advisor) and interviewee
(client) to discuss a wider range of subject
matters
g. physical attending
__________ 7. a type of question that solicits singular facts or a
yes or no response
h. question bombardment
__________ 8. a type of leading response in which the advisor
explains something to the client in a simple,
concise, and comprehensible way
i. resistance
__________ 9. a faulty questioning technique in which the
advisor asks two or more questions without
giving the client a chance to respond
j. social styles
__________ 10. a bonding that occurs when the financial
advisor’s sense of the client’s world fits the
client’s self-image
Student Workbook for HS 300
2-6
Answers to Key Terms and Concepts Matching
1.
2.
3.
4.
5.
6.
7.
8.
9.
10.
g.
b.
j.
i.
c.
f.
d.
e.
h.
a.
Student Workbook for HS 300
_________________________G2-1_________________________
HS 300 Financial Planning: Process
and Environment
Chapter 2:
Communicating Effectively With
Clients
1
Learning Objectives
2
Learning Objectives
1. Explain the importance of communicating
effectively with clients throughout the
financial planning process.
2. Explain the three main types of structured
communication used in financial planning.
3. Explain the importance of structuring
communications, building rapport, and
handling resistance in communicating with
clients.
3
Student Workbook for HS 300
_________________________G2-2_________________________
Learning Objectives
4.
5.
6.
7.
8.
Explain the attributes of an advisor that facilitate
communicating with clients.
Describe several basic communication principles.
Explain the importance of attending and listening
skills to communicating effectively with clients.
Describe several types of leading responses.
Compare the advantages and disadvantages of
several types of questions used in financial planning.
4
Discussion Break
How conscious of body language are you in
everyday conversations?
Do you manage your own body language...
– with clients?
– with friends?
– with family?
5
Interviewing
6
Student Workbook for HS 300
_________________________G2-3_________________________
Interviewing
Process of communicating with a
predetermined and specific purpose
• Directive interview
• Example: Gathering client data by
interviewing him or her
7
Directive Interviewing
• Formal, structured
• Strengths
– Brevity
– Organization
• Weaknesses
– Inflexibility
– Losing the story for the facts
8
Nondirective Interview
• Client controls pace and depth of response
• Strengths
– Flexibility
– Depth
– Easier to build client-advisor relationship
• Weaknesses
– Time consuming
– Easier to miss something
9
Student Workbook for HS 300
_________________________G2-4_________________________
Counseling and Advising
10
Counseling
• Connotes an offer to help
• Less formal and structured than interviewing
• Promotes understanding of issues faced rather
than solution to problem
11
Financial Counseling Is NOT
• Marital counseling
• Legal advice
• Tax advice
Being empathetic and understanding is
important but client may need professional
services you don’t provide
12
Student Workbook for HS 300
_________________________G2-5_________________________
Advising
• Provide specific guidance or suggestions
– Best used when advisor has
professional knowledge that isn’t readily
transferable to client.
– Offer after relationship developed or
client may not accept
• Does advice foster dependency?
13
Communication Considerations
14
Communication Considerations
• Structuring communications with clients
– Balance need for information against
need to build relationship
– Don’t waste client time gathering facts if
available another way
15
Student Workbook for HS 300
_________________________G2-6_________________________
Communication Considerations
• Developing rapport with clients
– Mutual acceptance
– Alleviating concerns
16
Social Styles
17
Social Styles
• Driver
• Analytical
• Expressive
• Amiable
18
Student Workbook for HS 300
_________________________G2-7_________________________
Driver
• Action-oriented
• Likes to lead
– Control
– Tell versus ask
19
Analytical
• Thinking-oriented
• Facts lead
• Builds to a decision based on information
20
Expressive
• Intuition-oriented
• Emote, tell
21
Student Workbook for HS 300
_________________________G2-8_________________________
Amiable
• Relationship-oriented
• Emote, ask
22
Effective Communication
23
Communicating Effectively
• Listening skills
– As important as speaking skills
• Communications goals
– Keep in mind for
• Relationship/goals data
• Communications structure
24
Student Workbook for HS 300
_________________________G2-9_________________________
Resistance
• Continue communication and relationship
building by
– Dealing with client resistance
– Recognizing client resistance
– Knowing sources of resistance
25
Resistance
• Dealing with client resistance
– Recognize and categorize
– Keeps the communications flowing and
relationship building
26
Sources of Client Resistance
• Privacy issues
• Death/dying
• Marital discord
– Separation/divorce
– Disagreements over the children
– Empty nest issues
– Midlife crises
27
Student Workbook for HS 300
_________________________G2-10_________________________
Discussion Break
How do you feel about TV ads for financial
advisors where the advisor is shown as both
trusted friend and advisor?
If you like the imagery, why do you like it?
If you don’t like the image, what bothers you about
it?
28
Attributes of an Effective Advisor
29
Attributes of an Effective Advisor
•
•
•
•
Unconditional positive regard
Like and respect have a circular effect
Accurate empathy
Identification without and empathy without
transferring
• Genuineness
• Awareness of client’s social style
30
Student Workbook for HS 300
_________________________G2-11_________________________
Attributes of an Effective Advisor
• Self-awareness of
– Limits and strengths
– Comfort zone, social style, avoidance
strategies
– Need to control, need to fix, degree of
openness
– Not using own values to frame client’s
decisions
31
Attributes of an Effective Advisor
• Build on your strengths and work around your
weaknesses
– Will give you more satisfied clients and
a better practice
• Orientation to values
• Respect differences
32
Communication Principles
33
Student Workbook for HS 300
_________________________G2-12_________________________
Basic Communication Principles
•
•
•
•
•
Elements of nonverbal behaviors
Attending and listening skills
Active listening
Questions
Statements
34
Elements of Nonverbal Behaviors
• Body language
– Positions
– Movements
– Gestures
35
Elements of Nonverbal Behaviors
• Facial expressions
• Eye contact
• Voice
36
Student Workbook for HS 300
_________________________G2-13_________________________
Interpreting Nonverbal Behaviors
• Only clues or indicators
– Not infallible
• Two-way nonverbal communication
– Client and advisor
37
Attending and Listening
38
Physical Attending
•
•
•
•
•
Square on
Open posture
Lean in
Good eye contact
Stay relaxed
39
Student Workbook for HS 300
_________________________G2-14_________________________
Active Listening
• Nonverbal communication plus
• Understanding responses
– Continuing
– Restatement of content
– Reflection of feeling
40
Active Listening—Responses
• Clarifying response
– Facilitates client’s self-understanding
– Attends to client’s feelings
– Communicates advisor’s understanding
– Moves client toward clearer definition of
problem
41
Active Listening—Responses
• Summarization response
• Leading responses
– Explanatory
– Interpretive
– Reassuring
– Suggestive
42
Student Workbook for HS 300
_________________________G2-15_________________________
Questions
43
Questions
• Open-ended versus closed-ended questions
• Leading questions
– Either/or
– True/false
44
Questions
• Why questions
• Question bombardment
45
Student Workbook for HS 300
_________________________G2-16_________________________
Concluding Remarks
• Statements
– Advisor is responsible for what is said
• Questions
– Client is responsible for what is said
46
Financial Planning in Action
47
Chapter Two Review
48
Student Workbook for HS 300
_________________________G2-17_________________________
Chapter Two Review
• Interviewing, counseling,
advising
• Structured versus unstructured
communication
• Self-awareness
• Client awareness
49
Chapter Two Review
•
•
•
•
•
Nonverbal communication
Active listening
Steering your responses
Asking appropriate questions
Making appropriate statements
50
Student Workbook for HS 300
Ethics, Professionalism, and Practice
Standards
3
Overview
The public’s confidence in the financial services industry has diminished considerably
during the past few years. This is primarily due to highly publicized ethical transgressions
by practitioners.
This class focuses on the role of ethics, professionalism, and practice standards in the
financial services profession.
Preparing for Class 3
√ Read chapter 3 in Financial Planning: Process and Environment, AT LEAST
TWICE.
√ Answer the review questions and self-test questions at the end of the chapter.
√ Study the key terms and concepts listed in the textbook.
√ Review the Important Note and Study Tips for Class 3 in the student workbook,
and the video graphics starting on page G3-1.
√ Participate in the online community for this course by accessing AC Online
at <http://blackboard.theamericancollege.edu>. Explore the channels for this
course on AC Online including: Course Info, Modules, Exams, Forums, and
Tools. Stop by the Student Lounge, on the Forum channel to post a note to other
students in the course, or Office Hours to post a note for the course instructor.
Contact Student Services if you don’t have your user ID or password.
Important Note
√ The following topic will be briefly covered in class 3:
– CFPŠ Financial Planning Practice Standards—p. 3.16–3.29
You are responsible for studying these materials on your own.
3-2
Study Tips for Class 3
•
•
•
•
Review The American College Code of Ethics.
Review the principles in the Code of Ethics and Professional Responsibility of
the CFPŠ Board of Standards.
Think of examples within your frame of reference of organizational policies or
activities that have the effect of compromising ethical decision making.
Think of five activities in the financial services industry that represent unethical
behavior and rank them (1) in order of their relative importance and (2) in order
of their relative frequency.
Post-Class: Test Your Comprehension
√ Complete the Quick Quiz for class 3.
√ Complete the Key Terms and Concepts matching exercise for class 3.
Student Workbook for HS
3-3
Quick Quiz
Circle your responses:
T F
1. In all cases, compliance with the law ensures ethical behavior.
T F
2. Adherence to the Code of Ethics of The American College is
mandatory.
T F
3. A spirit of altruism is an important characteristic of a financial
services professional.
T F
4. Recommending replacement of a life insurance policy is always
unethical.
T F
5. An ethical problem is more easily defined than a legal problem.
T F
6. There is a concrete body of ethical standards that serves as recourse
in ethical dilemmas.
T F
7. The CFPŠ Board’s Code of Ethics defines standards of professional
conduct of CFPŠ Board designees for purposes of civil liability.
T F
8. A professional is a person engaged in a field that requires, among
other things, a threshold entrance requirement.
T F
9. Practice standards are statements regarding an element of the
financial planning process.
T F
10. A practice standard in the 600 Series is related to defining the scope
of the engagement.
Answers to Quick Quiz
1-F, 2-T, 3-T, 4-F, 5-F, 6-F, 7-F, 8-T, 9-T, 10-F
Student Workbook for HS
3-4
Key Terms and Concepts
Match the CFPŠ Board’s Financial Planning Practice Standards to the related Code
Principle(s). You may use each Code Principle more than once, and the Practice
Standards may relate to more than one Code Principle.
Code Principle
Practice Standards
a.
Integrity
__________ 1. 100-1 Defining the scope of the engagement
b.
Objectivity
__________ 2. 200-1 Determining a client’s personal and
financial goals, needs, and priorities
c.
Competence
__________ 3. 200-2 Obtaining quantitative information and
documents
d.
Fairness
__________ 4. 300-1 Analyzing and evaluating the client’s
information
e.
Confidentiality
__________ 5. 400-1 Identifying and evaluating financial
planning alternative(s)
f.
Professionalism
__________ 6. 400-2 Developing the financial planning
recommendation(s)
g.
Diligence
__________ 7. 400-3 Presenting the financial planning
recommendation(s)
__________ 8. 500-1 Agreeing on implementation
responsibilities
__________ 9. 500-2 Selecting products and services for
implementation
__________ 10. 600-1 Defining monitoring responsibilities
Student Workbook for HS
3-5
Answers to Key Terms and Concepts Matching
1.
2.
3.
4.
5.
6.
7.
8.
9.
10.
Fairness, Diligence
Diligence
Diligence
Objectivity, Competence, Diligence
Objectivity, Competence, Professionalism, Diligence
Objectivity, Competence, Professionalism, Diligence
Integrity, Objectivity, Professionalism
Competence, Fairness, Professionalism, Diligence
Objectivity, Fairness, Professionalism, Diligence
Diligence
Student Workbook for HS 300
_________________________G3-1_________________________
HS 300 Financial Planning: Process
and Environment
Chapter 3:
Ethics, Professionalism, and
Practice Standards
Learning Objectives
Learning Objectives
1. Describe the role of ethics in society.
2. Explain the relationship between law and
ethics.
3. Identify common themes and sentiments found
in almost all ethics codes applicable to financial
advisors.
Student Workbook for HS 300
_________________________G3-2_________________________
Learning Objectives
4. Summarize the content of codes of ethics of
Certified Financial Planner Board of Standards,
Inc. and The American College.
5. Briefly explain what these codes mean in terms
of daily professional practice.
6. Describe the qualities that define a
professional.
Learning Objectives
7. Identify the hallmarks of professional behavior
for financial advisors.
8. Explain why financial planning professionals are
subject to increased risk of legal liability in
business dealings with clients.
Learning Objectives
9. Explain the nature and significance of Certified
Financial Planner Board’s practice standards
for financial planning practitioners.
Student Workbook for HS 300
_________________________G3-3_________________________
Discussion Break
If you had to choose between an advisor who
would operate
(a) above the law, OR
(b) above the call (of duty),
which advisor would you choose?
Would you pay a premium to use either advisor?
Relationship Between
Law and Ethics
Relationship Between
Law and Ethics
Ethics
Law
Student Workbook for HS 300
_________________________G3-4_________________________
Relationship Between
Law and Ethics
• How We Ought to Live:
Managers that define ethics as legal
compliance are implicitly endorsing a
code of moral mediocrity for their
organizations.
Lynn Sharpe Paine
Relationship Between
Law and Ethics
• You cannot legislate morality
– Not quite true
– That is the intersection of ethics and law
Role of Ethics
• Ethical Codes for Financial Advisors
– CFP® Board of Standards
– The American College
Student Workbook for HS 300
_________________________G3-5_________________________
Overview of the CFP Board’s
Code of Ethics and Professional
Responsibility
Certified Financial Planning Board
Overview
• Code of Ethics and Professional Responsibility
(Code of Ethics)
• Rules of Conduct
• Financial Planning Practice Standards (Practice
Standards)
• Disciplinary Rules and Procedures (Disciplinary
Rules)
Certified Financial Planning Board
Code of Ethics
CFP Board adopted the Code of Ethics to
establish the highest principles and standards.
These Principles are general statements
expressing the ethical and professional ideals
certificants and registrants are expected to
display in their professional activities.
Student Workbook for HS 300
_________________________G3-6_________________________
Certified Financial Planning Board
The Principles form the basis of CFP Board’s
Rules of Conduct, Practice Standards and
Disciplinary Rules, and these documents
together reflect CFP Board’s recognition of
certificants’ and registrants’ responsibilities to
the public, clients, colleagues and employers.
Certified Financial Planning Board
Rules of Conduct
The Rules of Conduct establish the high
standards expected of certificants and describe
the level of professionalism required of
certificants.
The Rules of Conduct are binding on all
certificants, regardless of their title, position, type
of employment or method of compensation, and
they govern all those who have the right to use
the CFP® marks, whether or not those marks are
actually used.
Certified Financial Planning Board
The universe of activities engaged in by a
certificant is diverse, and a certificant may
perform all, some or none of the typical
services provided by financial planning
professionals.
Student Workbook for HS 300
_________________________G3-7_________________________
Certified Financial Planning Board
Violations of the Rules of Conduct may subject a
certificant or registrant to discipline.
Because CFP Board is a certifying and standardssetting body for those individuals who have met
and continue to meet CFP Board’s initial and
ongoing certification requirements, discipline
extends to the rights of registrants and certificants
to use the CFP® marks.
Certified Financial Planning Board
Practice Standards
The Practice Standards describe best practices
of financial planning professionals providing
professional services related to the six
elements of the financial planning process.
Certified Financial Planning Board
Disciplinary Rules
The Disciplinary Rules provide a fair process
pursuant to which certificants are given notice
of potential violations and an opportunity to be
heard by a panel of other professionals.
Student Workbook for HS 300
_________________________G3-8_________________________
Code of Ethics Principles
Seven Principles
Principle 1: Integrity
Provide professional services with integrity.
Seven Principles
Principle 2: Objectivity
Provide professional services objectively.
Student Workbook for HS 300
_________________________G3-9_________________________
Seven Principles
Principle 3: Competence
Maintain the knowledge and skill necessary to
provide professional services competently.
Seven Principles
Principle 4: Fairness
Be fair and reasonable in all professional
relationships. Disclose conflicts of interest.
Seven Principles
Principle 5: Confidentiality
Protect the confidentiality of all client
information.
Student Workbook for HS 300
_________________________G3-10_________________________
Seven Principles
Principle 6: Professionalism
Act in a manner that demonstrates exemplary
professional conduct.
Seven Principles
Principle 7: Diligence
Provide professional services diligently.
Rules of Conduct
(Distilled)
Student Workbook for HS 300
_________________________G3-11_________________________
Rules of Conduct
1. Defining the Relationship With the
Prospective Client or Client
1.1
1.2
1.3
1.4
Services provided by certificant.
Obligations of both parties when
financial planning takes place.
Written agreement must include these
items when financial planning takes
place.
Fiduciary responsibility of certificant in
planning relationships.
Rules of Conduct
2. Information Disclosed to Prospective
Clients and Clients
2.1
2.2
Don’t lie, mislead, misstate or omit
in client communications
Required disclosures
Rules of Conduct
3. Prospective Client and Client
Information and Property
3.1 Confidentiality of client information
3.2 Secure information and property
3.3 Obtain all information relative to
decision making process
3.4 I.D. client assets
3.5 Maintain complete records
Student Workbook for HS 300
_________________________G3-12_________________________
Rules of Conduct
3. Prospective Client and Client
Information and Property
3.6 Don’t borrow from client (exceptions)
3.7 Don’t lend to client (exceptions)
3.8 No commingling of client assets with
certificant assets (exceptions)
3.9 No commingling of client assets with
other client assets (exceptions)
3.10 Return client assets when asked or as
agreed.
Rules of Conduct
4. Obligations to Prospective Clients and
Clients
4.1 Fair treatment/professional service
4.2 Advise in areas of competency
4.3 Comply with applicable regulations
4.4 Exercise reasonable and prudent
judgment
4.5 Make or implement suitable
recommendations
Rules of Conduct
4. Obligations to Prospective Clients and
Clients
4.6 Provide reasonable and prudent
professional supervision of
subordinates or third parties.
4.7 Obligation to inform clients of suspension
or revocation action by CFP Board
Student Workbook for HS 300
_________________________G3-13_________________________
Rules of Conduct
5. Obligations to Employers
5.1
5.2
Perform professional services with
dedication to lawful objectives of
employer/principal.
Obligation to inform employer/
principal of suspension or revocation
action by CFP Board
Rules of Conduct
6. Obligations to CFP Board
6.1
6.2
Abide by Board rules
Meet Board requirements including
C.E.
6.3 Keep contact information current
6.4 Notify Board of convictions,
professional suspensions or bar
6.5 Not engage in conduct that reflects
adversely on integrity or fitness as a
certificant
CFP Board’s Financial
Planning Practice Standards
Student Workbook for HS 300
_________________________G3-14_________________________
Practice Standards
These Practice Standards are intended to:
1.
2.
3.
Assure that the practice of financial planning by
CERTIFIED FINANCIAL PLANNER™ professionals is
based on established norms of practice;
Advance professionalism in financial planning; and
Enhance the value of the financial planning process.
Description of Practice Standards
A Practice Standard establishes the level of
professional practice that is expected of
certificants engaged in financial planning.
Description of Practice Standards
The Practice Standards apply to certificants in
performing the tasks of financial planning
regardless of the person’s title, job position,
type of employment or method of
compensation. Compliance with the Practice
Standards is mandatory for certificants whose
services include financial planning or material
elements of the financial planning process.
Student Workbook for HS 300
_________________________G3-15_________________________
Description of Practice Standards
The Practice Standards are designed to provide
certificants with a framework for the
professional practice of financial planning.
Similar to the Rules of Conduct, the Practice
Standards are not designed to be a basis for
legal liability to any third party.
Format of the Practice Standards
Each Practice Standard is a statement
regarding an element of the financial planning
process. It is followed by an explanation of the
Standard, its relationship to the Code of Ethics
and Rules of Conduct, and its expected impact
on the public, the profession and the
practitioner.
Compliance With the Practice Standards
The practice of financial planning consistent
with these Practice Standards is required for
certificants who are financial planning
practitioners.
Student Workbook for HS 300
_________________________G3-16_________________________
Practice Standards
100-1: Defining the Scope of the
Engagement
The financial planning practitioner and the client
shall mutually define the scope of the
engagement before any financial planning
service is provided.
Practice Standards
100-1: Defining the Scope of the
Engagement
The financial planning practitioner and the
client shall mutually define the scope of the
engagement before any financial planning
service is provided.
Gathering Client Data
200-1: Determining a Client’s Personal and
Financial Goals, Needs and Priorities
The financial planning practitioner and the client
shall mutually define the client’s personal and
financial goals, needs and priorities that are
relevant to the scope of the engagement before
any recommendation is made and/or
implemented.
Student Workbook for HS 300
_________________________G3-17_________________________
Gathering Client Data
200-2: Obtaining Quantitative Information and
Documents
The financial planning practitioner shall obtain
sufficient quantitative information and documents
about a client relevant to the scope of the
engagement before any recommendation is made
and/or implemented.
Practice Standards
300-1: Analyzing and Evaluating the
Client’s Information
A financial planning practitioner shall analyze
the information to gain an understanding of
the client’s financial situation and then
evaluate to what extent the client’s goals,
needs and priorities can be met by the client’s
resources and current course of action.
CFP Board’s Financial
Planning Practice Standards
Part 2
Student Workbook for HS 300
_________________________G3-18_________________________
Developing and Presenting the Financial
Planning Recommendation(s)
The 400 Series
- These Practice Standards can be
described as
“What is Possible?”
“What is Recommended?”
“How is it Presented?”
Developing and Presenting the Financial
Planning Recommendation(s)
400-1: Identifying and Evaluating Financial
Planning Alternative(s)
The financial planning practitioner shall
consider sufficient and relevant alternatives to
the client’s current course of action in an effort
to reasonably meet the client’s goals, needs
and priorities.
Developing and Presenting the Financial
Planning Recommendation(s)
400-2: Developing the Financial Planning
Recommendation(s)
The financial planning practitioner shall
develop the recommendation(s) based on the
selected alternative(s) and the current course
of action in an effort to reasonably meet the
client’s goals, needs and priorities.
Student Workbook for HS 300
_________________________G3-19_________________________
Developing and Presenting the Financial
Planning Recommendation(s)
400-3: Presenting the Financial Planning
Recommendation(s)
The financial planning practitioner shall
communicate the recommendation(s) in a
manner and to an extent reasonably
necessary to assist the client in making an
informed decision.
Practice Standards
500-1: Agreeing on Implementation
Responsibilities
The financial planning practitioner and the
client shall mutually agree on the
implementation responsibilities consistent with
the scope of the engagement.
Practice Standards
500-2: Selecting Products and
Services for Implementation
The financial planning practitioner shall select
appropriate products and services that are
consistent with the client’s goals, needs and
priorities.
Student Workbook for HS 300
_________________________G3-20_________________________
Practice Standards
600-1: Defining Monitoring
Responsibilities
The financial planning practitioner and client
shall mutually define monitoring
responsibilities.
The American College
Code of Ethics
The American College
Code of Ethics: Pledge
In all my professional relationships, I pledge
myself to the following rule of ethical conduct: I
shall, in light of all conditions surrounding those I
serve, which I shall make every conscientious
effort to ascertain and understand, render that
service which, in the same circumstances, I
would apply to myself.
Student Workbook for HS 300
_________________________G3-21_________________________
The American College
Code of Ethics: Eight Canons
I.
Conduct yourself at all times with honor and
dignity.
II. Avoid practices that would bring dishonor upon
your profession or The American
College.
III. Publicize your achievement in ways that
enhance the integrity of your profession.
The American College
Code of Ethics: Eight Canons
IV. Continue your studies throughout your working
life so as to maintain a high level of
professional competence.
V. Do your utmost to attain a distinguished record
of professional service.
VI. Support the established institutions and
organizations concerned with the integrity of
your profession.
The American College
Code of Ethics: Eight Canons
VII. Participate in building your profession by
encouraging and providing appropriate
assistance to qualified persons pursuing
professional studies.
VIII. Comply with all laws and regulations,
particularly as they relate to professional and
business activities.
Student Workbook for HS 300
_________________________G3-22_________________________
SEC Reg. IA-2256
Role of Ethics
• Practical application of ethics code
• IA-2256 requires that RIAs adopt code of ethics
Ethics Mandate for
Investment Advisors
• IA-2256 Investment Advisor Codes of Ethics
– Mandates registered advisers to adopt a code
of ethics
– Subject to compliance and records
– Code breach is actionable
Student Workbook for HS 300
_________________________G3-23_________________________
Discussion Break
As ethics and practice standards place increased
demands on your business’s time and budget,
how will you “manage” ethics in your practice?
Characteristics of a Professional
Characteristics Defining Professional
•
•
•
•
Specialized knowledge
Threshold entrance requirement
Sense of altruism
Code of ethics
Student Workbook for HS 300
_________________________G3-24_________________________
Professional Behavior
• Does advisor listen to client?
• Does advisor answer client’s questions?
• Is advisor gathering information sufficient to
provide good advice?
• Has advisor educated client about products
and/or services?
Professional Behavior
• Has advisor considered client’s ability to deal
with risk in recommendations?
• Has advisor disclosed educational
background and affiliations in financial
planning to the client?
• Does advisor provide client with a sense of
steady service?
Professional Behavior
• Does advisor handle money matters
properly?
• Does advisor refer to other experts?
• Does advisor seem up-to-date?
Student Workbook for HS 300
_________________________G3-25_________________________
Characteristics of Professional
• Behaving professionally
– Salesperson’s standard is suitability
– Advisor’s standard is fiduciary
• Increased risk of legal liability
– Lawsuits and complaints damage
reputation and practice
• NASD’s broker check
• CFP® Board reports disciplinary actions
Applying CFP® Practice Standards to
Financial Planning Process
Financial Planning Process Steps and
Corresponding Practice Standard(s)
• Establishing and defining advisor-client
relationship
100-1 Defining the scope of engagement
• Determining goals and gathering data
200-1 Determining goals, needs, priorities
200-2 Obtaining information and documents
Student Workbook for HS 300
_________________________G3-26_________________________
Financial Planning Process Steps and
Corresponding Practice Standard(s)
• Analyzing and evaluating data
300-1 Analyzing and evaluating the client’s
information
Financial Planning Process Steps and
Corresponding Practice Standard(s)
• Developing and presenting a plan
400-1 I.D. alternatives
400-2 Develop recommendation
400-3 Present recommendation
Financial Planning Process Steps and
Corresponding Practice Standard(s)
• Implementing the plan
500-1 Agreeing on implementing
responsibilities
500-2 Selecting products and services for
implementation
Student Workbook for HS 300
_________________________G3-27_________________________
Financial Planning Process Steps and
Corresponding Practice Standard(s)
• Monitoring the plan
600-1 Defining monitoring responsibilities
Chapter Three Review
Chapter Three Review
•
•
Ethics and the law
CFP Code Principles and Rules
–
–
–
–
Integrity
Objectivity
Competence
Fairness
– Confidentiality
– Professionalism
– Diligence
Student Workbook for HS 300
_________________________G3-28_________________________
Chapter Three Review
• The American College Code
– Pledge
– Canons
• SEC Reg. IA-2256
– Code breach is actionable
• What does it mean to be a professional?
• What does it mean to not be a professional?
Chapter Three Review
• CFP Board® steps in financial planning process
and corresponding practice standards
Student Workbook for HS 300
Client Attitudes Toward Risk
4
Overview
Investments are characterized by different levels of risk and potential return. Both
ethical and regulatory principles require that the financial advisor recommend only
those products and investment strategies that are suitable given the client’s investment
objectives, financial capacity to absorb a loss, and psychological propensity for risk
taking. The goal is to deliver the most return for the amount of tolerable risk. Proper
asset allocation requires a determination of the client’s risk tolerance.
Financial advisors of all types need to understand risk tolerance and convey to their
clients its significance in reaching proper investment decisions. Determining a client’s
level of risk tolerance, while extremely important, is also one of the most difficult tasks
facing the financial advisor. Done properly, however, it will improve relationships with
clients and lessen the possibility of litigation by those who have invested beyond their
comfort level.
To help us understand client attitudes toward risk, this class examines how people
view risk and how they process information about risk factors. It explores the reasons
people either minimize or maximize the objective level of risk in a situation. It also
identifies demographic and personality characteristics that have been linked to risk
tolerance. Finally, it considers various approaches to assessing a client’s risk-taking
propensity and notes their relative merits and drawbacks.
Preparing for Class 4
√ Read chapter 4 in Financial Planning: Process and Environment, AT LEAST
TWICE.
√ Answer the review questions and self-test questions at the end of the chapter.
√ Study the key terms and concepts listed in the textbook.
√ Review the Important Note and Study Tips for Class 4 in the student workbook,
and the video graphics starting on page G41.
4-2
√ Participate in the online community for this course by accessing AC Online
at <http://blackboard.theamericancollege.edu>. Explore the channels for this
course on AC Online including: Course Info, Modules, Exams, Forums, and
Tools. Stop by the Student Lounge, on the Forum channel to post a note to other
students in the course, or Office Hours to post a note for the course instructor.
Contact Student Services if you don’t have your user ID or password.
Important Note
√ The following topics will be briefly covered in class 4:
– Communicating Probability Statements Verbally—p. 4.28–4.29
You are responsible for studying these materials on your own.
Study Tips for Class 4
•
•
•
•
•
•
•
Be prepared to tell why the topic of client risk tolerance should be important
to financial planners.
Be aware that most people’s intuitive understanding of risk differs from the more
mathematical definitions of risk, as in the field of finance, and that most people’s
intuitive (subjective) assessment of risk is affected by "bounded rationality."
Be able to list the first four limitations on rational thinking discussed in the text.
Definitions of each of these biases are a source of examination questions.
Be aware that the client’s risk tolerance in one of the four areas, for example,
monetary situations, can be quite different from the same client’s risk tolerance
in the other three areas (physical, social, and ethical). Can you give examples
from your own clients (for example, a pro football player who is a very
conservative investor)?
Be able to distinguish between the characteristics of monetary risk averters
and risk takers.
Study how each of the personal and occupational characteristics tend to
influence or reflect risk tolerance.
Know the following practical techniques for assessing a client’s risk tolerance:
– Focus on monetary risk taking.
– Assume the client is risk averse, unless evidence to the contrary can
be obtained.
Student Workbook for HS 300
4-3
– Remember that people are more likely to overstate than understate their
risk-taking propensity.
– Keep in mind that even risk-averse individuals may be risk-seeking in
situations where the choices are between losses. People are reluctant to
cut their losses.
– Start your assessment by looking at the client’s demographic
characteristics and personality makeup. However, just because a person
belongs to a group that is more risk tolerant or risk averse than average, it
does not mean that the client facing you today will necessarily follow
the group pattern.
– Look for quantitative-assessment devices that are accurate and allow you
to compare the client’s performance to a norm group.
– Consider the results from a questionnaire or other measurement device as
only an approximation of the client’s actual risk tolerance. No assessment
procedure is perfect; they are all susceptible to some error.
– Diversify your methods of assessment. This will allow you to draw on
the strengths of the various techniques.
– Use the information you collect to start a dialogue with the client about
risk tolerance. When an agreement is reached, ask the client to provide a
written confirmation of the results of this assessment.
– Realize that biases may be operating on the client’s risk perceptions (for
example, familiarity, availability, and so on), which may be inflating or
deflating the client’s true level of risk tolerance.
– Remember that a client’s propensity for risk taking does not necessarily
remain constant throughout his or her lifetime.
Post-Class: Test Your Comprehension
√ Complete the Quick Quiz for class 4.
√ Complete the Key Terms and Concepts matching exercise for class 4.
Student Workbook for HS 300
4-4
Quick Quiz
Circle your responses:
T F
1. One of the characteristics that distinguishes risk averters from risk
takers is that risk averters are typically more optimistic than risk
takers.
T F
2. Compared to low-risk-tolerant mutual fund investors,
high-risk-tolerant mutual fund investors are more likely to rely on
the advice of professional advisers.
T F
3. After 10 flips of a perfectly balanced coin in which the results were
all heads, most people are likely to estimate intuitively that the
chances of the eleventh coin flip being heads is less than 50–50.
T F
4. Availability bias refers to the fact that events that are easy to imagine
or recall will be judged by most people as less probable than they
actually are.
T F
5. People tend to psychologically overemphasize short-term risks in
comparison with long-term risks.
T F
6. Alcohol consumption typically causes people to become more
reckless and to take greater risks than they would ordinarily.
T F
7. Since there is very little similarity between most people’s risk
tolerance in financial matters to that in other aspects of their lives,
there is little reason for the financial planner to be concerned about
a client’s risk-taking disposition in nonfinancial matters.
T F
8. The thrill seeker is a personality type that consistently is a risk taker
in all or most dimensions of life.
T F
9. Research indicates that most people are more comfortable
describing the probability of an event in words rather than in
numbers.
Student Workbook for HS 300
4-5
T F
10. The real-life-choices approach to the assessment of risk tolerance
is based on the assumption that the best predictor of a client’s
risk tolerance is the client’s identification of his or her financial
objectives.
Answers to Quick Quiz
1-F, 2-F, 3-T, 4-F, 5-T, 6-T, 7-F, 8-T, 9-T, 10-F
Student Workbook for HS 300
4-6
Key Terms and Concepts
Match the key term or concept to the most accurate definition.
a. absolute risk tolerance
__________ 1. the tendency of some individuals to engage
in risky behaviors on a voluntary basis,
seemingly failing to appreciate the true level
of danger in the situation
b. bounded rationality
__________ 2. the manner in which human beings behave
because of the limits on their rationality
c. denial of risk
__________ 3. the characteristic of seeking to limit the size
of the potential loss rather than seeking to
minimize the variability of the potential
returns
d. familiarity bias
__________ 4. standards of measurement, such as average,
that allow comparison of an individual to a
representative group
e. illusion of control bias
__________ 5. individuals who are relatively unwilling to
incur financial risk
f. loss aversion
__________ 6. an interpretation of the riskiness of a situation,
which is not necessarily the same as the
objective riskiness
g. norms
__________ 7. an individual’s willingness to incur financial
risk in absolute (such as dollar) terms
h. perceived risk
__________ 8. an inclination or prejudice that alters people’s
perception of risk; specifically, what is
familiar is less risky than what is unfamiliar
i. risk averters (risk rejecters)
__________ 9. an inclination or prejudice that alters
people’s perception of risk; specifically,
underestimating the risk involved in events
under their control
j. uncertainty
__________ 10. a situation in which the possible outcomes
and/or their associated probabilities of
occurrence are unknown
Student Workbook for HS 300
4-7
Answers to Key Terms and Concepts Matching
1.
2.
3.
4.
5.
6.
7.
8.
9.
10.
c.
b.
f.
g.
i.
h.
a.
d.
e.
j.
Student Workbook for HS 300
_________________________G4-1_________________________
HS 300 Planning:
Process and Environment
Chapter 4:
Client Attitudes Toward
Risk
1
Learning Objectives
2
Learning Objectives
1. Distinguish between risk seekers, risk
averters, and risk indifferent; compare
subjective perceptions of risk with objective
definition of financial risk.
2. Describe characteristics of people that limit
their ability to rationally assess risk.
3. Compare the four major categories of life
situations that involve risk taking.
3
Student Workbook for HS 300
_________________________G4-2_________________________
Learning Objectives
4. Summarize the differences in the
characteristics of risk takers versus risk
averters.
5. Describe and evaluate techniques for
assessing the risk tolerance of clients.
6. Identify guidelines that a financial advisor
should follow when assessing the risk
tolerance of clients.
4
Discussion Break
Advisors often distinguish between savings
and investments. Why differentiate between
the two?
Distinguish between speculation and
investing.
5
Risk
6
Student Workbook for HS 300
_________________________G4-3_________________________
Risk Continuum
• Risk seeker (risk-tolerant)
• Risk indifferent (risk-neutral)
• Risk averters (risk rejecters)
7
Risk
• Pure risk versus speculative risk
– Pure risk has only risk of loss
• Insurable risk
– Speculative risk has potential for loss and
gain
8
Risk
• Risk in decision sciences
– Various consequences of each alternative
are known
– Exact probabilities can be specified
– Uncertainty
9
Student Workbook for HS 300
_________________________G4-4_________________________
Perceived Versus Objective Risk
• Perceived risk
– Influenced by personal experience,
circumstances, and inclination toward risktaking
• Objective risk
– Not colored by perceptions
10
Perception
11
What Influences Perception?
• Overconfidence in intuitive judgments
– Self-handicapping risks understate risks
• Nonrepresentative quality of short-run trends
– Law of large numbers depends on sample
size
12
Student Workbook for HS 300
_________________________G4-5_________________________
What Influences Perception?
• Failure to correctly evaluate exposure time
– Short- versus long-duration risk
– Low risk versus high risk
13
Perceptions
• Denial of risk
– Discounting small negative probabilities
• Familiarity bias
– Seasoned investors are more risk tolerant
14
Perceptions
• Availability bias
– What makes the news gives us the blues
– Unreported and underreported acts are off
the radar
15
Student Workbook for HS 300
_________________________G4-6_________________________
Perceptions
• Illusion of control bias
• Time horizon
– Risk level is time dependent
16
Perceptions
• Mood
– Bad mood/pessimistic
– Good mood/optimistic
• Alcohol
– Increases risk tolerance
17
Parties Bearing Consequences
• Decision makers more risk tolerant if they
alone suffer consequences of their actions
• Decision makers also more risk tolerant if
only others bear the consequences
18
Student Workbook for HS 300
_________________________G4-7_________________________
Discussion Break
In considering who bears the consequences of
a decision maker’s actions, making a mutual
fund manager invest in the fund(s) he or she
manages should reduce risk tolerance. Should
such investments be:
(A)
(B)
tolerated
allowed
(C)
(D)
encouraged
required
19
Group and Situational
Risk Taking
20
Group Dynamics of Risk Taking
• Choice shift typically toward more risky action
• Choice shift explained by
– Shared decision making
– Influential risk-tolerant group members
– Familiarity bias by-product of group
discussion
21
Student Workbook for HS 300
_________________________G4-8_________________________
Mental Accounting
• How you keep score influences the risks
you’re willing to take on and your decision
making
22
Situational Risk Taking
• Monetary situations
– Investments, gambling, job changes (loss
of capital)
• Physical situations
– Risk of bodily harm (loss of life)
23
Situational Risk Taking
• Social situations
– Respect, self-esteem (loss of reputation)
• Ethical situations
– Moral, religious, or legal standards (loss of
freedom)
24
Student Workbook for HS 300
_________________________G4-9_________________________
Risk Behaviors
• Behavior in one situation isn’t useful in
predicting behavior in another
25
Demographics
26
Demographic Characteristics
• Wealth
– Absolute risk tolerance
($ wealth allocated to risky assets)
– Relative risk tolerance
(% wealth allocated to risky assets)
27
Student Workbook for HS 300
_________________________G4-10_________________________
Demographic Characteristics
• Education
– Risk tolerance positively correlated with
education
• Age
– Risk tolerance negatively correlated with
age
28
Demographic Characteristics
• Gender
– Mixed results
• Birth order
– First born less risk tolerant
• Marital status
– Mixed results
29
Demographic Characteristics
• Occupation
– Public versus private sector
• Public sector employees more risk
averse
30
Student Workbook for HS 300
_________________________G4-11_________________________
Demographic Characteristics
• Occupation
– Professionals versus nonprofessionals
• Professionals more risk tolerant
 Differences within profession
(surgeon/g.p.)
 Mixed bag among professions
– Job tenure
31
Demographics
• Management level
– Senior level managers more risk tolerant
• Salary versus commission
– The higher the percentage of income
earned as commission the more likely it is
that individual is a monetary risk taker
32
Demographics
• Entrepreneurship
– Need for independence
– Not risk tolerant
33
Student Workbook for HS 300
_________________________G4-12_________________________
Assessment and Ranking of
Risk Tolerance
34
Assessment of the Client’s
Risk Tolerance
• Purpose
– Client education
– Advisor education
• Fiduciary standards
• Manage expectations
• Relate goals to tolerance
35
Assessment of the Client’s
Risk Tolerance
• Quantitative versus qualitative assessment
– Compliance issues favor quantitative
36
Student Workbook for HS 300
_________________________G4-13_________________________
Ranking Risk Tolerance
• Ranking investment objectives
– Liquidity
– Safety of principal
– Inflation protection
– Growth
– Current income
– Tax issues
37
Ranking Risk Tolerance
• Ranking investment choices
– CDs
– U.S. governments
– Muni’s
– Corporates
– Stocks
– Options
– Futures
38
Ranking Risk Tolerance
• Ranking by ratios
– Current portfolio composition
– Debt ratio
– Life insurance/annual salary
– Size of insurance deductibles
39
Student Workbook for HS 300
_________________________G4-14_________________________
Ranking Risk Tolerance
• Ranking by ratios
– Recreational gambling
– Job tenure
– Income volatility
– Type of mortgage
40
Ranking Risk Tolerance
• Ranking attitudes toward risk
– Anxiety meter
• Probability and payoff preferences
– Preferences for certain versus probable
gambles
– Minimum required probability of success
– Minimum required return
41
Ranking Risk Tolerance
How the questions are framed influences
results, so framing is important too
42
Student Workbook for HS 300
_________________________G4-15_________________________
Guidelines on Assessment
• Focus on monetary risk taking
• Assume client is risk averse unless proven
otherwise
• Remember that people tend to overstate risktaking propensity
43
Guidelines on Assessment
• Remember that people are reluctant to cut
losses
• Don’t rely solely on demographics—
communicate
• Find the right survey instrument/
questionnaire
44
Financial Planning in Action
45
Student Workbook for HS 300
_________________________G4-16_________________________
Chapter Four Review
• Risk tolerance is essential part of six steps in
creating financial plan
• Compliance issues make quantitative
approach to determining risk tolerance more
likely
• Communication and education are the keys
to meeting the client’s needs in this area
46
Student Workbook for HS 300
Gathering Data and Preparing Financial
Statements
5
Overview
The development of a comprehensive financial plan covering all the major planning
areas is neither a quick nor an easy task. The diversity and complexity of needed
information require a systematic method for gathering, organizing, and processing that
information. Although there are many possible ways to systematize the gathering of
information, almost all financial advisors prefer to use a structured fact-finder form.
With the financial information gathered by a fact finder, an advisor is able to determine
a client’s current financial position and cash flow status. Without this type of financial
information about the client, it would be difficult, if not impossible, for the advisor to
develop a financial plan and/or formulate strategies for achieving the client’s financial
goals.
To help us better understand the gathering and analyzing of personal financial
information, this class first focuses on fact finding and fact finders. It then shifts its focus
to a thorough discussion of how financial advisors prepare and use the two primary
data-gathering statements, that is, the financial position statement and the cash flow
statement.
Preparing for Class 5
√ Read chapter 5 in Financial Planning: Process and Environment, AT LEAST
TWICE.
√ Answer the review questions and self-test questions at the end of the chapter.
√ Study the key terms and concepts listed in the textbook.
√ Review the Study Tips for Class 5 in the student workbook, and the video
graphics starting on page G5-1.
√ Participate in the online community for this course by accessing AC Online
at <http://blackboard.theamericancollege.edu>. Explore the channels for this
course on AC Online including: Course Info, Modules, Exams, Forums, and
Tools. Stop by the Student Lounge, on the Forum channel to post a note to other
5-2
students in the course, or Office Hours to post a note for the course instructor.
Contact Student Services if you don’t have your user ID or password.
Study Tips for Class 5
•
•
•
Know the general characteristics of the financial position statement (balance
sheet) and cash flow statement including the component parts, format, basic
accounting equation, and what they reflects.
Know the key points about the three basic components of cash-flow
management—cash-flow analysis, cash-flow planning, and budgeting.
Be able to list the five steps in constructing a cash budget.
Post-Class: Test Your Comprehension
√ Complete the Quick Quiz for class 5.
√ Complete the Key Terms and Concepts matching exercise for class 5.
Student Workbook for HS 300
5-3
Quick Quiz
Circle your responses:
T F
1. One of the necessary components of a comprehensive financial plan
is a statement showing all the client’s sources and uses of cash and
the client’s net cash-flow position.
T F
2. An individual’s financial position statement shows the person’s
wealth at a specified time.
T F
3. An individual’s net worth is calculated by adding the person’s total
assets and total liabilities.
T F
4. It is now more common in personal financial planning to present
financial position statements in a single-column format rather than
as a two-column balance sheet.
T F
5. In most financial planning situations, the primary benefit of net
discretionary cash flow is to provide a source of investable funds.
T F
6. A client’s risk tolerance is easy to measure.
T F
7. The Risk/Return Investment Pyramid arranges investment media so
that the most risky and highest-yielding media are at the base of the
pyramid.
T F
8. Today, the values of many types of personal assets can be estimated
using Internet websites.
T F
9. Normally the advisor meets with the client at least once each year
to review the plan or more frequently if changing circumstances
warrant it.
T F
10. Savings, investments, household furnishings, and an education fund
are all classified as fixed expenses.
Answers to Quick Quiz
1-T, 2-T, 3-F, 4-T, 5-T, 6-F, 7-F, 8-T, 9-T, 10-F
Student Workbook for HS 300
5-4
Key Terms and Concepts
Match the key term or concept to the most accurate definition.
a. budgeting
__________ 1. organized list of the components of an individual’s
or family’s wealth at a specified time including
assets, liabilities, and net worth
b. cash flow analysis
__________ 2. assets bought primarily for the creature comforts
they provide
c. cash flow management
__________ 3. budgetary planning and control processes
d. cash flow planning
__________ 4. difference between income and expenses
e. financial position
statement
__________ 5. identifying courses of action that help optimize
net cash flow
f. income statement
__________ 6. process of creating an explicit plan for spending
and investing the resources available to the client
g. net cash flow
__________ 7. process of reviewing cash flow information
h. nonfinancial (personal)
assets
__________ 8. projected financial statement(s)
i. pro forma
__________ 9. summary of the various income and expense items
of an individual or company during an accounting
period
j. Risk/Return
Investment Pyramid
__________ 10. used to graphically show the trade-off of risk
against return
Student Workbook for HS 300
5-5
Answers to Key Terms and Concepts Matching
1.
2.
3.
4.
5.
6.
7.
8.
9.
10.
e.
h.
c.
g.
d.
a.
b.
i.
f.
j.
Student Workbook for HS 300
_________________________G5-1_________________________
HS 300 Financial Planning:
Process and Environment
Chapter 5: Gathering Data
and Preparing
Financial Statements
1
Learning Objectives
2
Learning Objectives
1. Describe initial financial planning session, and
explain importance of fact finders.
2. Describe types of information in comprehensive fact
finders, and explain the importance of periodic
review and update.
3. Explain importance of personal financial statements
in financial planning.
3
Student Workbook for HS 300
_________________________G5-2_________________________
Learning Objectives
4–6. Explain financial position statements:
– what they mean to financial advisors
– key components
– steps in preparing
– how used in the process
4
Learning Objectives
7–10.
11.
12.
Explain cash flow statements:
– what they mean to financial advisors
– key components
– purpose of cash flow management
– steps in preparing
– how used in the process
Define, calculate, and explain the
importance of consumer financial ratios in
financial planning.
Discuss importance of monitoring the client’s
financial position and the use of pro forma
forecasts as evaluation tools in the
5
monitoring process.
Discussion Break
How do you feel about the client collecting
and documenting the “fact-finder” data for a
second meeting?
If your firm has a fact-finder form, what are
the form’s strengths and weaknesses?
6
Student Workbook for HS 300
_________________________G5-3_________________________
Initial Session
7
Initial Session
• Single prospect versus seminar
• Need for complete financial information
• Nature and scope of services
– What’s in a plan
– Your background/qualifications/ strengths
– Compensation
8
Fact Finder
9
Student Workbook for HS 300
_________________________G5-4_________________________
Fact Finder
• Chronicles client’s current:
– Financial position
• Income
• Out-go
• Holdings
– Domestic situation
• Comprehensive vs. single-/multi-purpose
planning
10
Fact Finder
• Personal data
– Includes other advisory relationships
– Domestic
 Spouse, children
 Trusts, gifts, inheritances
 Benefits
11
Fact Finder
• Financial objectives
– Client listing
Budgeting
Savings
Investments
– Planning process may redefine
• Gifting (yea/nay/in play)
• Satisfaction in current strategies
12
Student Workbook for HS 300
_________________________G5-5_________________________
Fact Finder
• What’s on the client’s mind
– Desired age of retirement
– Distribution of estate
– Attitudes and moods
• Expectations of financial planning process
13
Financial Condition
14
Financial Condition
•
•
•
•
Sources of income
Inventory of assets and liabilities
Individually-owned insurance
Employment-related benefits
15
Student Workbook for HS 300
_________________________G5-6_________________________
Financial Condition
• Income and lump-sum needs for disability,
retirement, and death
• Tax-planning checklist
• Personal assessment of the client
16
Risk/Return
17
Risk/Return Profile
• Effect of client’s attitude toward risk on
building a financial plan
• Fact finder provides a quick assessment
18
Student Workbook for HS 300
_________________________G5-7_________________________
Risk/Return Investment Pyramid
rea
os
so
fC
rd
ap
thr
ita
ou
l
gh
Ap
pr e
cia
tio
n
Inc
fL
isk
o
Re
wa
se
dR
nti
al
ote
In c
rea
In c
rea
er
se
dP
w
Po
ing
as
rch
l
Pu
ipa
of
inc
ss
Pr
Lo
of
of
ty
afe
dS
se
rea
Bonds
isk
Inc
Stocks
dR
se
Speculative
Cash and Near-Cash Equivalents
19
Base of the Pyramid
• Strongest portion, which supports everything
above it
• Comprised of investments that are low in risk
and have foreseeable returns
• May be largest area and comprise bulk of
your assets
• Emergency fund and transaction balances
part of base
20
Middle Portion
• Medium-risk investments
• Offer a stable return while still allowing for
capital appreciation
• Relatively safe investments
21
Student Workbook for HS 300
_________________________G5-8_________________________
Summit
• High-risk investments
• Smallest area of the pyramid (portfolio)
• Money you can lose without serious
repercussions
• Not needed for liquidity so you won't have to
sell prematurely at a loss
22
Discussion Break
Prospective clients often aren’t particularly
organized about their finances but an
incomplete financial picture makes it next
to impossible to prepare a comprehensive
plan.
How do you get the client to provide a
complete picture and/or how do you
protect yourself from incomplete pictures?
23
Financial Position Statement
24
Student Workbook for HS 300
_________________________G5-9_________________________
Preparing a Financial Position
Statement
• Assets
– Cash
– Other financial
assets
– Personal assets
• Total assets
• Liabilities
– Unsecured
– Secured
• Total liabilities
• Net worth
25
Preparing a Financial Position
Statement
Net Worth = Total Assets – Total Liabilities
Ours = Own – Owe
26
Financial Position Statement
Assets
Cash
Other financial assets
$
$
80,000
440,000
Personal assets
$
410,000
Total assets
$
930,000
Liabilities
Unsecured
Secured
Total Liabilities
$
6,000
$ 232,000
$ 238,000
Net Worth
TL + NW
$ 692,000
$ 930,000
27
Student Workbook for HS 300
_________________________G5-10_________________________
Cash Flow Planning
28
Cash Flow Planning
• Examines sources of cash
• Examines uses of cash
• Affects all of the major planning areas
29
Preparing a Cash Flow Statement
• Income
• Expenditures
– Fixed
• Contractual
• Mandatory
• Needs
– Discretionary
• Lifestyle
• Investments/
savings
• Wants
30
Student Workbook for HS 300
_________________________G5-11_________________________
Cash Flow Statement
31
Cash Flow Statement
Income
Salaries
Bonus
Dividend & Interest (taxable)
Interest (tax exempt)
Total Annual Income
$
$
$
$
$
140,000
3,000
7,000
3,000
153,000
92%
2%
5%
2%
100%
Fixed Expenses
Mortgage
Utilities
Taxes
Transportation
Insurance
Other
Total Fixed Exp.
$
$
$
$
$
$
$
15,500
7,000
36,000
8,000
13,000
41,000
120,500
10%
5%
24%
5%
8%
27%
79%
Discretionary
Vacation
Recreation
Gifts
Investments
Other
Total Discretionary Exp.
$
$
$
$
$
$
4,000
5,000
7,500
10,500
10,000
37,000
3%
3%
5%
7%
7%
24%
Total Expenses
$
157,500
103%
Net Cash Flow
$
(4,500)
Income
Salaries
Bonus
Dividend & Interest (taxable)
Interest (tax exempt)
Total Annual Income
$ 140,000
$
3,000
$
7,000
$
3,000
$ 153,000
92%
2%
5%
2%
100%
-3%
32
Cash Flow Statement
Income
Salaries
Bonus
Dividend & Interest (taxable)
Interest (tax exempt)
Total Annual Income
$
$
$
$
$
140,000
3,000
7,000
3,000
153,000
92%
2%
5%
2%
100%
Fixed Expenses
Mortgage
Utilities
Taxes
Transportation
Insurance
Other
Total Fixed Exp.
$
$
$
$
$
$
$
15,500
7,000
36,000
8,000
13,000
41,000
120,500
10%
5%
24%
5%
8%
27%
79%
Discretionary
Vacation
Recreation
Gifts
Investments
Other
Total Discretionary Exp.
$
$
$
$
$
$
4,000
5,000
7,500
10,500
10,000
37,000
3%
3%
5%
7%
7%
24%
Total Expenses
$
157,500
103%
Net Cash Flow
$
(4,500)
-3%
Fixed Expenses
Mortgage
Utilities
Taxes
Transportation
Insurance
Other
Total Fixed Exp.
$
$
$
$
$
$
$
15,500
7,000
36,000
8,000
13,000
41,000
120,500
10%
5%
24%
5%
8%
27%
79%
33
Student Workbook for HS 300
_________________________G5-12_________________________
Cash Flow Statement
Income
Salaries
Bonus
Dividend & Interest (taxable)
Interest (tax exempt)
Total Annual Income
$
$
$
$
$
140,000
3,000
7,000
3,000
153,000
92%
2%
5%
2%
100%
Fixed Expenses
Mortgage
Utilities
Taxes
Transportation
Insurance
Other
Total Fixed Exp.
$
$
$
$
$
$
$
15,500
7,000
36,000
8,000
13,000
41,000
120,500
10%
5%
24%
5%
8%
27%
79%
Discretionary
Vacation
Recreation
Gifts
Investments
Other
Total Discretionary Exp.
$
$
$
$
$
$
4,000
5,000
7,500
10,500
10,000
37,000
3%
3%
5%
7%
7%
24%
Total Expenses
$
157,500
103%
Net Cash Flow
$
(4,500)
Discretionary
Vacation
Recreation
Gifts
Investments
Other
Total Discretionary Exp.
$
$
$
$
$
$
4,000
5,000
7,500
10,500
10,000
37,000
3%
3%
5%
7%
7%
24%
-3%
34
Cash Flow Statement
Income
Salaries
Bonus
Dividend & Interest (taxable)
Interest (tax exempt)
Total Annual Income
$
$
$
$
$
140,000
3,000
7,000
3,000
153,000
92%
2%
5%
2%
100%
Fixed Expenses
Mortgage
Utilities
Taxes
Transportation
Insurance
Other
Total Fixed Exp.
$
$
$
$
$
$
$
15,500
7,000
36,000
8,000
13,000
41,000
120,500
10%
5%
24%
5%
8%
27%
79%
Discretionary
Vacation
Recreation
Gifts
Investments
Other
Total Discretionary Exp.
$
$
$
$
$
$
4,000
5,000
7,500
10,500
10,000
37,000
3%
3%
5%
7%
7%
24%
Total Expenses
$
157,500
103%
Net Cash Flow
$
(4,500)
Total Annual Income
.
.
.
Total Expenses
$
153,000
100%
$
157,500
103%
Net Cash Flow
$
(4,500)
–3%
-3%
35
Pro Forma Financial Statements
• Projections for the future
• Useful for
– Benchmarking
– Variance analysis
36
Student Workbook for HS 300
_________________________G5-13_________________________
Ratio Analysis
37
Ratio Analysis
• Corporate finance focuses on
– Profitability
– Liquidity
– Solvency
– Debt management
– Asset management
• Personal ratio analysis has different
perspective
38
Ratio Analysis
• Liquidity ratio = liquid assets / total current
debts
– Total current debts = current liabilities +
annual loan payments
– Liquidity and emergency fund needs are
related
• Solvency ratio = net worth / total assets
39
Student Workbook for HS 300
_________________________G5-14_________________________
Ratio Analysis
• Debt service = mortgage payment + debt
payments / net income
• Savings ratio = NCF + savings / annual aftertax income
40
Setting Goals
41
Goals
• Use those listening skills
– Don’t presume
– Don’t assume
• Get client talking about life goals and
translate to financial goals
42
Student Workbook for HS 300
_________________________G5-15_________________________
Goals
• Financial statements provide initial point of
reference
• Client’s life goals are points out in the future
• Financial plan lays out the strategy to fund
life goals by achieving financial goals
• Financial calculators and software help put a
number on the financial goals
43
Monitoring the Plan
• Pro formas
• Variance analysis
• Benchmarking investments
44
Financial Planning in Action
45
Student Workbook for HS 300
_________________________G5-16_________________________
Chapter Five Review
46
Chapter Five Review
•
•
•
•
•
Gather facts
Build financial statements
Determine life goals
Translate life goals to financial goals
Build a plan that has a high probability of
achieving the goals
• Monitor the financials as part of monitoring
the plan
47
Student Workbook for HS 300
Time Value of Money: Basic Concepts
and Applications
6
Overview
This class covers several basic concepts that are essential to understanding the
operation of the time value of money and its application to financial planning. These
concepts can be divided into those involving either present values or future values. The
present value concepts are the present value of a single sum (PVSS), the present value
of an annuity (PVA), and the present value of an annuity due (PVAD). Future value
concepts include the future value of a single sum (FVSS), the future value of an annuity
(FVA), and the future value of an annuity due (FVAD). Emphasis is placed on showing
how these concepts are used to solve time value of money problems in the broader
context of financial planning.
To help you solve time-value-of-money problems, this class introduces two types of
tools. At the low end of the tech ladder, explanations are provided for solving many
problems by using mathematical formulas and factor tables. Higher up the technological
ladder is the use of financial calculators. While most financial calculators with time-value
capabilities could be used to solve time-value-of-money problems, this assignment only
explains how to solve these problems using an HP-10BII. If you already own another
financial calculator and do not want to acquire an HP-10BII, then make sure you know
how to solve the various types of time-value-of-money problems with your calculator.
Although this assignment and the next one present many time-value-of-money
concepts mathematically with the aid of factor tables, you should avoid becoming
bogged down in the math. Instead, concentrate on recognizing the different types of
problems and learning how to use an HP-10BII to solve them.
Preparing for Class 6
√ Read chapter 6 in Financial Planning: Process and Environment, AT LEAST
TWICE.
√ Answer the review questions and self-test questions at the end of the chapter.
√ Study the key terms and concepts listed in the textbook.
6-2
√ Review the Study Tips for Class 6 in the student workbook, and the video
graphics starting on page G6-1.
√ Participate in the online community for this course by accessing AC Online
at <http://blackboard.theamericancollege.edu>. Explore the channels for this
course on AC Online including: Course Info, Modules, Exams, Forums, and
Tools. Stop by the Student Lounge, on the Forum channel to post a note to other
students in the course, or Office Hours to post a note for the course instructor.
Study Tips for Class 6
•
•
Assignments 6 and 7 may require considerably more study time than the others.
The following points are important to study of this material:
– TVM analysis is important to the work of a financial services professional
– You don’t need high-powered mathematical skills to solve TVM
problems. (You need only to understand TVM concepts, recognize their
application, and push buttons on a financial calculator.)
– You should not worry about small rounding differences in the answers
you calculate.
•
Memorize the formulas for treatment of single sum problems:
– the future value formula: FVSS = (1+i)n
– the present value formula:
PVSS =FVSS ×1(1+i)n
Recognize that the future value factor [(1/(1+i)n] is the reciprocal of the
present value factor [(1/(1+i)n]. You should also recognize the impact of a
change in the interest rate or the number of years. Specifically,
– as i rises, FVSS rises or PVSS declines
– as n rises, FVSS rises or PVSS declines
•
•
Note that any battery operated, non-printing calculator that cannot store words
is permissible at the HS 300 exam. However, the only calculator supported
by the HS 300 materials is the HP-10BII by Hewlett Packard. It is the better
choice for both price and functionality.
Know the difference between an annuity and an annuity due, the difference
being one more year of interest for beginning-of-period problems. That is,
Student Workbook for HS 300
6-3
FVAD = FVA (1 – i)
PVAD = PVA (1 + i)
Post-Class: Test Your Comprehension
√ Complete the Quick Quiz for class 6.
√ Complete the Key Terms and Concepts matching exercise for class 6.
Student Workbook for HS 300
6-4
Quick Quiz
Circle your responses:
T F
1. The rate of interest in a financial transaction is the way in which the
opportunity cost in the transaction is quantified.
T F
2. If the PVSS factor for 5 years and 8 percent is .6806, the FVSS
factor for 5 years and 8 percent can be calculated by dividing 1 by
.6806.
T F
3. If $1,000 is deposited in a 10 percent savings instrument and left
to accumulate for 8 years, at the end of the 8th year there will be
$1,800.00 in the account.
T F
4. If you think a piece of raw land you are considering purchasing will
be worth $100,000 in 8 years, you should be willing to pay $53,349
for it today if you want to earn a 10 percent compound annual rate
of return on your investment.
T F
5. If $1,000 is placed in a savings instrument at the end of each of
the next 8 years, no withdrawals are made, and all deposits earn
10 percent interest per year, at the end of the 8th year there will be
$11,435.89 in the account.
T F
6. If you wish to accumulate $100,000 by making eight annual
deposits beginning today, and if all deposits will earn 10 percent
compound annual interest, the size of each deposit should be
$8,744.40.
T F
7. The present value of a series of eight annual payments of $100
beginning today discounted at 10 percent is $586.84.
T F
8. If a $10,000 loan with a 10 percent annual interest rate is to be repaid
in eight equal annual installments beginning one year from now, the
annual payment will be $1,874.44.
T F
9. According to the Rule of 72, a single sum accumulating at 10
percent compound annual interest will double in value in about 3.6
years.
Student Workbook for HS 300
6-5
T F
10. For any given interest rate and number of years, the PVAD factor is
always larger than the PVA factor.
Answers to Quick Quiz
1-T, 2-T, 3-F, 4-F, 5-T, 6-F, 7-T, 8-T, 9-F, 10-T
Note
Correct answers are 3—$2,143.59; 4—$46,650.74; 6—$7,949.46; (payments
are at beginning of year) 9—7.2 years
Student Workbook for HS 300
6-6
Key Terms and Concepts
Match the key term or concept to the most accurate definition.
a. annuity
__________ 1. process by which money today (present value)
grows over time to a larger amount (future
value)
b. compounding
__________ 2. process by which money due in the future
(future value) is reduced over time to a smaller
amount today (present value)
c. discounting
__________ 3. present value of a series of equal periodic
payments made at the end of each period and
discounted at an appropriate rate
d. future value of an annuity
due (FVAD)
__________ 4. increment of return required above the risk-free
rate that an investor demands to reward him or
her for accepting risk
e. future value of a single
sum (FVSS)
__________ 5. fund where a target amount is achieved by
contributing a series of equal payments
f. present value of an
annuity (PVA)
__________ 6. concept that a specific amount of money
received (paid) in a specific time period has a
different value than the same amount received
(paid) in a different time period
g. present value of a single
sum (PVSS)
__________ 7. amount to which an annuity due would
accumulate by the end of its term if the
payments earned a specific rate of return during
the entire time period
h. risk premium
__________ 8. amount determined by discounting a future
value at a particular interest rate for a particular
length of time
i. sinking fund
__________ 9. amount determined by compounding a present
value at a particular interest rate for a particular
length of time
j. time value of money
__________ 10. a finite stream of equal payments made at the
end of each of a number of consecutive periods
Student Workbook for HS 300
6-7
Answers to Key Terms and Concepts Matching
1.
2.
3.
4.
5.
6.
7.
8.
9.
10.
b.
c.
f.
h.
i.
j.
d.
g.
e.
a.
Student Workbook for HS 300
_________________________G6-1_________________________
HS 300 Financial Planning:
Process and Environment
Chapter 6:
Time Value of Money—
Basic Concepts and Applications
1
Learning Objectives
2
Learning Objectives
1. Describe several basic concepts underlying
the time value of money.
2. Calculate future value of single sum, and
calculate the number of periods or the
interest rate in future-value-of-a-single-sum
problems.
3
Student Workbook for HS 300
_________________________G6-2_________________________
Learning Objectives
3. Calculate the present value of single sum,
and calculate the number of periods or
interest rate in present-value-of-a-single-sum
problems.
4. Calculate future value of an annuity or an
annuity due, and solve sinking fund
problems.
4
Learning Objectives
5. Calculate present value of annuity or annuity
due, and solve debt service/ capital-sumliquidation problems.
6. Create an amortization schedule for a level
payment loan, and delineate the level
payment into principal and interest
components.
5
Learning Objectives
7. Solve single sum and annuity problems with
five values/variables.
6
Student Workbook for HS 300
_________________________G6-3_________________________
Discussion Break
Risk-based pricing has become an important
part of determining the interest rate on a loan.
For the following loans, at what interest rate
would you advise a client to avoid the debt
versus take out the loan?
(A) First mortgage
(C) Auto loan
(B) Home equity line of credit (D) Credit card
7
Present Value of a Single Sum
(PVSS)
8
Present Value of a Single Sum
(PVSS)
Problem
FV =$1,000
I/YR = 4%
N=5
PV = ?
Keystrokes
1000,FV
4, I/YR
5, N
PV
– 821.93
9
Student Workbook for HS 300
_________________________G6-4_________________________
PVSS Solving for N
Problem
FV = $1,000
I/YR = 6%
PV = -$800
N=?
Keystrokes
1000, FV
6, I/YR
800, +/-, PV
N
3.83
10
PVSS Solving for I/YR
Problem
FV = $1,000
PV = -$800
N=5
I/YR = ?
Keystrokes
1000, FV
800, +/-, PV
5, N
I/YR
4.56
11
Future Value of a Single Sum
(FVSS)
12
Student Workbook for HS 300
_________________________G6-5_________________________
Future Value of a Single Sum (FVSS)
Problem
PV = $1,000
I/YR = 7%
N = 20
FV = ?
Keystrokes
1000,+/–,PV
7, I/YR
20, N
FV
3,869.68
13
FVSS Solving for N
Problem
PV = $10,000
I/YR = 5%
FV = $20,000
N=?
Keystrokes
10000, +/–, PV
5, I/YR
20000, FV
N
14.21
14
FVSS Solving for I/YR
Problem
PV = –$10,000
FV = $20,000
N = 10
I/YR = ?
Keystrokes
10000, +/–, PV
20000, FV
10, N
I/YR
7.18
15
Student Workbook for HS 300
_________________________G6-6_________________________
Present Value of an Annuity
(PVA)
16
Present Value of an Annuity (PVA)
• Annuity is periodic cash flow stream
– Dollar amount constant over time
– Set number of periods (not in perpetuity)
• Ordinary annuity has cash flows at end of
each period
• Annuity due has cash flows at beginning of
each period
• 4 or 5 variable problem with 1 unknown
17
PVA
Problem
PMT = $1,000
N = 20
I/YR = 5%
PV = (?)
Keystrokes
1000, PMT
20, N
5, I/YR
PV
–12,462.21
18
Student Workbook for HS 300
_________________________G6-7_________________________
Discussion Break
Give a real-world example of an annuity due
and an ordinary annuity.
19
Present Value of an
Annuity Due (PVAD)
20
Present Value of an Annuity Due
(PVAD)
Problem
PMT = $1,000
N = 20
I/YR = 5%
PV = (?)
Keystrokes
Set calculator to BEGIN by
(orange), (BEG/END)
1000, PMT
20, N
5, I/YR
PV
– 13,085.32
21
Student Workbook for HS 300
_________________________G6-8_________________________
Future Values of Annuity and
Annuity Due
22
Future Value of an Annuity (FVA)
Problem
PMT = – 3,000
I/YR = 6%
N = 30
FV = ?
Keystrokes
3,000,+/ –,PMT
6, I/YR
30,N
FV
237,174.56
23
FV Annuity Due (FVAD)
Problem
PMT = – 3,000
I/YR = 6%
N = 30
FVAD = ?
Keystrokes
[orange],[BEG/END] sets the
calculator to BEGIN
3,000,+/ –,PMT
6, I/YR
30,N
FV
251,405.03
24
Student Workbook for HS 300
_________________________G6-9_________________________
Amortization
25
Amortization Schedule
• Loan payments on self-amortizing loans can
be broken down into principal and interest
components
• In early years of loan, more of payment goes
towards interest than principal; in later years,
opposite is true
• Use financial calculator to determine
breakdown for any loan payment
26
Amortization Breakdown (AMORT)
Problem
12,000, PV
5, N
6, I/YR
PMT
– 2,848.76
Find the interest/principal
breakdown of the third loan
payment and the resulting loan
balance
Keystrokes (after calc.
pmt)
3,[INPUT],[orange],
[AMORT]
= – 2,391.87 principal
= – 456.89 interest
= 5,222.88 balance
27
Student Workbook for HS 300
_________________________G6-10_________________________
Chapter Six Review
28
Chapter Six Review
• Move money through time either by discounting
back to today or compounding out to a future date
• Problems will specify all but one of variables, and
financial calculator will solve for unknown variable—
regardless of what it is
29
Course Overview
30
Student Workbook for HS 300
_________________________G6-11_________________________
Horizons Instructor
• Craig Lemoine, CFP®
– Assistant Professor of Financial Planning
at The American College
– Other professional endeavors
• Academic Journals
• 8 Years of Life Insurance Industry
Experience
• Financial Planner
31
Course Overview
HS 300 Financial Planning:
Process and Environment
• First course in
– CFP certification curriculum
– ChFC® designation curriculum
• Elective in the CLU® designation curriculum
32
CFP Topics List
General principles of financial planning
Insurance planning and risk management
Employee benefits planning
Investment planning
Income tax planning
Retirement planning
Estate planning
33
Student Workbook for HS 300
_________________________G6-12_________________________
Horizons HS 300 Overview
• Facilitator guided
• Ten classes covering ten chapters
• Review class
• Improved success rate
34
Course Resources
•
•
•
•
•
•
•
Textbook
DVDs
Facilitator’s guide
Supplement
Audio review
Flash cards (optional)
Review book (optional)
35
Other Course Resources
• AC Online
http://blackboard.theamericancollege.edu
– Announcements
– Forum
– Interactive exercises
– Assessment
• Quizzes
• Practice exams
36
Student Workbook for HS 300
_________________________G6-13_________________________
Pretest Question
Is financial planning a
product or a process?
37
Learning Objectives
38
Learning Objectives
1. Explain the six steps in the financial planning
process.
2. Describe three different approaches to
financial planning, and identify several areas
of specialization in which
advisors concentrate their activities.
39
Student Workbook for HS 300
_________________________G6-14_________________________
Learning Objectives
3. Identify the subjects that should be included
in a comprehensive financial plan.
4. Describe what is meant by a person’s
financial life cycle, and explain how it relates
to life-cycle financial planning.
40
Learning Objectives
5. Explain how a financial plan can be
developed around the steps in the financial
planning process.
6. Explain how a financial plan can be
developed using the financial planning
pyramid.
41
Learning Objectives
7. Explain the trends that are creating
opportunities in the financial planning
marketplace.
42
Student Workbook for HS 300
_________________________G6-15_________________________
Learning Objectives
8. Identify the principal financial goals/
concerns of most consumers, and
describe three major obstacles that
prevent them from achieving these goals.
43
Financial Planning Defined
CFP Board definition:
“Financial planning is the process of meeting
your life goals through the
proper management of your finances.”
44
Financial Planning Defined
Text definition:
“Financial planning is a process that
ascertains the client’s financial goals
and develops a plan for achieving the client’s
goals.”
45
Student Workbook for HS 300
_________________________G6-16_________________________
Six Steps in the
Financial Planning Process
46
Six Steps in the
Financial Planning Process
1. Establishing and defining the clientplanner relationship
2. Gathering client data, including goals
47
Six Steps in the
Financial Planning Process
3. Analyzing and evaluating your financial
status
4. Developing and presenting financial
planning recommendations and/or
alternatives
48
Student Workbook for HS 300
_________________________G6-17_________________________
Six Steps in the
Financial Planning Process
5. Implementing the financial planning
recommendations
6. Monitoring the financial planning
recommendations
49
The General Model
• Step 1: Establish and define advisorclient relationship
– Build relationship
– Explain process
– Describe products and/or services
– Clarify responsibilities
– Disclose advisor’s background,
philosophy, method of compensation
50
The General Model
• Step 2: Determine goals and gather data
– Encourage client to determine and
prioritize goals
– Gather client information
• Fact-finding forms
• Questionnaires
• Counseling
• Examination of documents
51
Student Workbook for HS 300
_________________________G6-18_________________________
The General Model
• Step 3: Analyze and Evaluate the Data
– Analyze and evaluate client’s financial
condition relative to achieving goals
– Revise goals if necessary
52
The General Model
• Step 4: Develop and present a plan
– Design recommended strategies to
achieve goals
• Include alternatives
– Use outside experts as needed
– Present plan
– Obtain client approval
53
The General Model
• Step 5: Implement the plan
– Help client acquire products and services
– Establish accounts
– Use outside experts as needed
54
Student Workbook for HS 300
_________________________G6-19_________________________
The General Model
• Step 6: Monitor the plan
– Evaluate performance of implementation
vehicles
– Review changes in client’s circumstances
and the financial environment
– Revisit other steps as necessary
55
Financial Planning in Action
56
Discussion Break
The six steps of financial planning convey
a fairly linear approach to planning.
Discuss how you could adapt this process
to be more free wheeling while still
covering the six steps.
57
Student Workbook for HS 300
_________________________G6-20_________________________
Different Approaches to
Planning
58
Different Approaches to Planning
• Single-purpose
– How do I invest my 401(k)?
• Multi-purpose
– How can I meet my goals to retire at 60
and fund my children’s education?
59
Different Approaches to Planning
• Comprehensive
– How can we manage our wealth to meet
our needs for current and future income
including the income needs of our planned
estate?
60
Student Workbook for HS 300
_________________________G6-21_________________________
Tim
e
an
dW
ea
lth
Financial Planning Pyramid
Stage 3
Stage 2
Managing Retirement
and the Estate
Growing Investments
Guarding Against
Uncertainty
Stage 1
61
Life-Cycle Financial Planning
62
Life-Cycle Financial Planning
•
•
•
•
Early career (ages 25–35)
Career development (ages 35–50)
Peak accumulation (ages 50–62)
Preretirement (3–6 years before planned
retirement)
• Retirement (ages 62–66+)
63
Student Workbook for HS 300
_________________________G6-22_________________________
Financial Planning Needs
• Up and coming (ages 20–39)
– 28 percent have a written financial plan
– Most tolerant of risk
• Mid-life (ages 40–54)
– 39 percent have written financial plan
– Low to moderate risk tolerance
64
Financial Planning Needs
• Retirement cusp (ages 55–69)
– 47 percent have written financial plan
– Most likely to have financial professional as
advisor
65
Top 10 Reasons People Begin
Financial Planning
66
Student Workbook for HS 300
_________________________G6-23_________________________
Top 10 Reasons People Begin
Financial Planning
10. Sheltering income from taxes
(25 percent)
9. Generating current income
(26 percent)
67
Top 10 Reasons People Begin
Financial Planning
8. Providing insurance protection
(29 percent)
7. Accumulating capital (31 percent)
6. Building a college fund (32 percent)
68
Top 10 Reasons People Begin
Financial Planning
5. Planning vacation/travel (34 percent)
4. Managing/reducing current debt
(34 percent)
3. Building an emergency fund
(40 percent)
69
Student Workbook for HS 300
_________________________G6-24_________________________
Top 10 Reasons People Begin
Financial Planning
2. Purchasing/renovating home (41 percent)
1. Building a retirement fund (82 percent)
Source: CFP Board of Standards, Inc.
2004 National Consumer Survey
70
Planning’s Partners
Client
Attorney
Accountant
Financial
Advisor
Insurance
Agent
Registered
Representative
71
Financial Planning in Action
72
Student Workbook for HS 300
_________________________G6-25_________________________
Planning Trends
•
•
•
•
•
•
•
Aging population
Increase in dual income households
Market volatility
Technological change
Increasing sophistication of consumers
Financial planning profession still growing
International economy
73
Building the Plan
74
Building a Plan
•
•
•
•
•
Goals
Budget
Net worth
Risk tolerance
Investments
75
Student Workbook for HS 300
_________________________G6-26_________________________
Using the Pyramid Model
• Liquidity and security
• Capital accumulation
• Income and purchasing power
76
What’s in a Comprehensive Plan?
• Goals, financial statements, risk tolerance,
advisor and client responsibilities
• Insurance and risk management
• Employee benefits
77
What’s in a Comprehensive Plan?
• Investment planning
• Income tax planning
• Retirement planning
• Estate planning
78
Student Workbook for HS 300
_________________________G6-27_________________________
Monitoring the Plan
79
Monitoring the Plan
• Determine level of service
• Perform periodic reviews
– Update financial situation
– Analyze variances
– Review goals and revise as needed
– Track performance of investments
80
Monitoring the Plan
• Establish need for rebalancing
– Time diversification
– Risk modeling
81
Student Workbook for HS 300
_________________________G6-28_________________________
Obstacles to Success
82
Obstacles to Success
• Procrastination
– Works against the miracle of compound
interest
• Deficit spending
– There’s nothing to save for tomorrow
83
Obstacles to Success
• Gaps in understanding
– Keeps client from visualizing planning
solutions
• Unmanaged expectations
– Unrealistic goals meet unrealized returns
84
Student Workbook for HS 300
_________________________G6-29_________________________
Goal Tending
• Consumption/investment decisions
– Budgeting allows client to manage cash
flow
– Net worth keeps score (own-owe)
85
Goal Tending
• Asset allocation balances risk/return tradeoffs
when investing for the future
– Liquidity
– Return
– Risk
86
Where the Industry Is Going
87
Student Workbook for HS 300
_________________________G6-30_________________________
Where the Industry Is Going
• Aging baby boomers
• Sandwich generation
• Kids’ college fund / our retirement / aging
parents
• Investing inheritances
88
Where the Industry Is Going
• Social Security concerns
• Growth in planners
– BLS Occupational Survey expects 21–35
percent growth in field over next 10 years
89
Chapter One Review
• Planning is a six step process
• Single-purpose, multi-purpose, and
comprehensive approaches to plans
– Only comprehensive incorporates all
seven topic areas
90
Student Workbook for HS 300
_________________________G6-31_________________________
Review
• Looking back (monitoring) can help you move
forward
• Overcoming obstacles and maintaining focus
on client goals improves odds of success
91
Review (Continued)
• Population trends
– Demographics: statistical market
characteristics
• age
• sex
• income
• educational level
92
Review (Continued)
• Population trends
– Psychographics: classification of
population groups according to
psychological variables
• attitudes
• values
• fears
Source: Merriam-Webster’s Unabridged Dictionary
93
Student Workbook for HS 300
_________________________G6-32_________________________
Review (Continued)
• Projected growth in profession
94
Student Workbook for HS 300
Time Value of Money: Advanced
Concepts and Applications
7
Overview
After a thorough grounding in the basics of time value of money in class 6, you should
be ready for more advanced concepts and applications. These include understanding
uneven cash flows; evaluating investments using discounted cash flow analysis; and
increasing the compounding, discounting, or payment frequency. While the mechanics
of using your HP-10BII to solve these advanced problems are more complex than those
described in class 6, the general strategy remains the same. You enter the known values
of a problem and solve for the unknown value.
Preparing for Class 7
√ Read chapter 7 in Financial Planning: Process and Environment, AT LEAST
TWICE.
√ Answer the review questions and self-test questions at the end of the chapter.
√ Study the key terms and concepts listed in the textbook.
√ Review the Study Tips for Class 7 in the student workbook, and the video
graphics starting on page G7-1.
√ Participate in the online community for this course by accessing AC Online
at <http://blackboard.theamericancollege.edu>. Explore the channels for this
course on AC Online including: Course Info, Modules, Exams, Forums, and
Tools. Stop by the Student Lounge, on the Forum channel to post a note to other
students in the course, or Office Hours to post a note for the course instructor.
Study Tips for Class 7
•
•
Be sure you recognize the definition/characteristics of grouped versus
ungrouped data in these problems.
You do not need to memorize the constant growth formula. However, you
should see how the formula works in the review questions.
7-2
•
Make sure you are able to define net present value and internal rate of return.
– NPV is the difference between the PV of all the inflows and the PV of all
the outflows, both discounted at the same, investor-specified interest rate.
– IRR is the interest rate that equates the PV of the inflows with the PV of
the outflows so that the NPV is zero.
•
•
Know the key points regarding the use of net present value and internal rate of
return in evaluating proposed investments.
Practice, practice, practice using your financial calculator.
Post-Class: Test Your Comprehension
√ Complete the Quick Quiz for class 7.
√ Complete the Key Terms and Concepts matching exercise for class 7.
Student Workbook for HS 300
7-3
Quick Quiz
Circle your responses:
T F
1. There are no grouped cash flows in the following sequence:
+$1,000; –$1,000; +$500; $0; +$500.
T F
2. In calculating the future value of the following set of uneven
cash flows as of the end of 4 years, the final cash flow should be
compounded for one year.
End
of
Year
Amount
1
$100
2
$200
3
$300
4
$400
T F
3. In the calculation of the net present value of a proposed investment
project, the riskiness associated with the cash inflows and outflows
is ignored.
T F
4. Generally, when used to decide whether a particular proposed
investment project is acceptable, the NPV and IRR techniques will
lead the investor to the same conclusion.
T F
5. In the calculation of an investment’s net present value, years of
negative net cash flows should be ignored.
T F
6. The PVSS is smaller when the discount rate is applied to a given
amount quarterly rather than annually.
T F
7. To change the following formula to reflect semiannual rather than
annual compounding, change i to .06 and n to 10.
FVSS = $100(1.12)5
Student Workbook for HS 300
7-4
T F
8. A simple annuity is one in which the frequency of the payments and
the frequency of compounding or discounting are the same.
Questions 9 and 10 refer to the following fact pattern:
Sparacio Bleach Company is considering an investment project that
requires an initial outflow of $100,000 with cash flows at the end of
years one–5 of $40,000, $40,000, –$20,000, $40,000, and $40,000,
respectively.
T F
9. The required rate of return for a project of this type is 14 percent.
T F
10. The net present value of the investment project is –$3,175.05.
T F
11. The internal rate of return of this project is 12.62 percent.
Answers to Quick Quiz
1-T, 2-F, 3-F, 4-T, 5-F, 6-T, 7-T, 8-T, 9-T, 10-T
Student Workbook for HS 300
7-5
Key Terms and Concepts
Match the key term or concept to the most accurate definition.
a. annual percentage rate
(APR)
__________ 1. process of compounding interest an infinite
number of times per year rather than at discrete
time intervals
b. complex annuity due
__________ 2. present value of a stream of cash inflows minus
the present value of a stream of cash outflows,
with both present values calculated on the basis
of an appropriate rate of interest
c. continuous compounding
__________ 3. periodic interest rate multiplied by the number
of periods in a year
d. conversion of interest
earnings into principal
__________ 4. occurs when interest is credited to a sum and
begins to earn interest on itself
e. effective annual interest
rate (EAR)
__________ 5. interest rate that equates the present value of
the stream of cash inflows to the present value
of the stream of cash outflows
f. Fisher effect
__________ 6. describes the dilutive effect of inflation on
nominal interest rates to arrive at a real rate of
return
g. internal rate of return
(IRR)
__________ 7. cash flow sequences that include no consecutive
payments of the same amount and arithmetic
sign (positive or negative)
h. net present value (NPV)
__________ 8. annuity in which the frequency of payments and
the frequency of compounding or discounting
are identical
i. simple annuity
__________ 9. annuity due in which the frequency of
payments and the frequency of compounding
or discounting are different
j. ungrouped cash flows
__________ 10. annual interest rate that produces in one
compounding the same amount of interest
as does the nominal annual rate with its
compounding frequency
Student Workbook for HS 300
7-6
Answers to Key Terms and Concepts Matching
1.
2.
3.
4.
5.
6.
7.
8.
9.
10.
c.
h.
a.
d.
g.
f.
j.
i.
b.
e.
Student Workbook for HS 300
_________________________G7-1_________________________
HS 300 Financial Planning:
Process and Environment
Chapter 7:
Time Value of Money—
Advanced Concepts
and Applications
1
Learning Objectives
2
Learning Objectives
1. Calculate present value of series of uneven
cash flows.
2. Calculate present value of series of cash
flows that grow by a constant percentage.
3. Calculate future value of series of uneven
cash flows.
3
Student Workbook for HS 300
_________________________G7-2_________________________
Learning Objectives
4. Calculate future value of series of cash
flows that grow by a constant percentage.
5. Compute net present value (NPV) of
investment project.
4
Learning Objectives
6. Compute internal rate of return (IRR) on
investment project.
7. Solve several types of time-value problems
when compounding or discounting occurs
more frequently than annually.
5
Discussion Break
When you are in a client meeting will you
have confidence in running time-value-ofmoney calculations in front of the client? Will
you do it using:
(A) the back of an envelope
(B) your trusty financial calculator
(C) your trusty laptop
(D) nothing; your assistant handles anything
that requires mental acuity
6
Student Workbook for HS 300
_________________________G7-3_________________________
Class Overview
7
Taking It Up a Notch
• Unequal cash flows
• More frequent compounding/discounting
– Effective interest rates
– APRs and APYs
• Complex annuities
8
Unequal Cash Flows
9
Student Workbook for HS 300
_________________________G7-4_________________________
Unequal Cash Flows
• Equal annual payments isn’t always a
realistic situation
– Example:
 IRA contributions
 future contribution limits indexed to
inflation
• Calculate present value of cash flow stream
using financial calculator
10
Present Value of
Unequal Cash Flows
11
PV Unequal CF Stream #1
Problem
CF0 = –$2,000
CF1 = $500
CF2 = $750
CF3 = $1,000
I/YR = 5%
Keystrokes
2000,+/–, CFj
500, CFj
750, CFj
1000, CFj
5, I/YR
[orange] NPV
20.30
12
Student Workbook for HS 300
_________________________G7-5_________________________
PV Unequal CF Stream #2
Problem
CF0 = –$2,000
CF1 = $500
CF2 = $500
CF3 = $750
CF4 = $750
I/YR = 5%
Keystrokes
2000,+/–, CFj
500, CFj , 2, [orange], Nj
750, CFj, 2, [orange], Nj
5, I/YR
[orange] NPV
194.61
13
PV Unequal CF Stream #3
Problem
CF0 = –$3,000
CF1-10 = $500
CF11 = $800
CF12 = –$1000
I/YR = 6%
Keystrokes
3000,+/–, CFj
500, CFj , 10,[orange], Nj
800, CFj
1000, +/–, CFj
6, I/YR
[orange] NPV
604.50
14
Internal Rate of Return of
Unequal Cash Flows
15
Student Workbook for HS 300
_________________________G7-6_________________________
Solving for the Internal Rate of
Return (IRR%)
• Internal rate of return is interest rate that
makes net present value equal to zero
• When a cash flow stream has a positive NPV
the IRR>i%
• When a cash flow stream has a negative
NPV the IRR<i%
• When a cash flow stream has an NPV of zero
the IRR=i%
16
IRR Cash Flow Stream # 3
Problem
CF0 = –$3,000
CF1-10 = $500
CF11 = $800
CF12 = –$1000
IRR = ?
Keystrokes
3000,+/–, CFj
500, CFj , 10,[orange], Nj
800, CFj
1000, +/–, CFj
[orange] IRR/YR
10.27
17
Discussion Break
Finance professors prefer NPV. Most
business professionals prefer IRR. NPV is
the better measure, so why is IRR so
popular?
18
Student Workbook for HS 300
_________________________G7-7_________________________
Future Values of
Unequal Cash Flows
19
FV of Unequal CF Stream
• Two-step process
– Compute NPV
– Then find FVSS of that NPV
20
FV Unequal CF Stream #2
Problem
Keystrokes
CF0 = –$2,000
CF1 = $500
CF2 = $500
CF3 = $750
CF4 = $750
I/YR = 5%
NPV = 194.61
FV = ?
2000,+/–, CFj
500, CFj , 2, [orange], Nj
750, CFj, 2, [orange], Nj
5, I/YR
[orange] NPV
194.61, +/–, PV
4, N
FV
236.55
21
Student Workbook for HS 300
_________________________G7-8_________________________
More Frequent Cash Flows
22
More Frequent Cash Flows
• Cash flows typically more than annual
• Examples
– Monthly mortgage payments
– Semiannual coupon interest payments
– Automatic investment programs
23
Bond Valuation
Problem
A bond with a 6 percent
coupon pays interest
semi-annually. If
investors require an 8
percent return, what is
the price of a $1,000
face value bond with 4
years to maturity?
Keystrokes
2, [orange], P/YR
30, PMT,
1000, FV
8, I/YR
4, [orange], x P/YR
PV
–932.67
24
Student Workbook for HS 300
_________________________G7-9_________________________
Finding YTM on a Bond
Problem
A bond with a face value
of $1,000 is priced at
$1,100. It pays semiannual interest at a
coupon rate of 5 percent
and has 7 ½ years until it
matures. What is its
yield to maturity?
Keystrokes
2, [orange], P/YR
1000, FV
1100, +/–, PV
25, PMT
7.5, [orange], x P/YR
I/YR
3.47
25
Effective Rates
26
Effective Rates
• Effective rates adjust for more frequent
compounding
– Nominal rates do not
27
Student Workbook for HS 300
_________________________G7-10_________________________
Effective Rates
• Formula:
EAR = (1 + nominal rate/# periods)(#per)–1
• Problem:
Calculate EAR for a 5 percent nominal
interest paid semiannually
EAR = (1+.05/2)2 – 1=.0506 = 5.06%
28
Effective Rates Problem #1
Problem
Six percent interest is
paid quarterly. What is
the effective rate?
Keystrokes
4, [orange], P/YR
6, I/YR
[orange] EFF%
6.1364
by formula it is:
(1+.06/4)4–1=.061364
29
Effective Rates Problem #2
Problem
What is the EAR of a 5
percent loan if interest
compounds:
– weekly?
– daily ?
Keystrokes
52, [orange], P/YR
5, I/YR
[orange], EFF%
5.1246
365, [orange], P/YR
5, I/YR
[orange], EFF%
5.1267
30
Student Workbook for HS 300
_________________________G7-11_________________________
APRs and APYs
31
APRs and APYs
• APRs are Truth-in-Lending Act based
calculations that allow borrowers to compare
rates across lenders.
• APYs are Truth-in-Savings Act based
calculations that allow investors to compare
rates across financial institutions.
• You borrow at a rate (APR) but earn a yield
(APY).
32
APRs
• Periodic rate multiplied by number of periods
in a year equals APR
• Example:
If a credit card charges a monthly rate of 1.25
percent, then APR for credit card is:
APR = 1.25% x 12= 15%
EAR = (1.0125)12-1= .1608 = 16.08%
33
Student Workbook for HS 300
_________________________G7-12_________________________
APRs
EAR not equal to APR
but APY equivalent to the EAR
34
More Frequent Compounding
35
More Frequent Compounding
• When interest is compounded more
frequently than paid, you have to adjust
interest rate to reflect disparity
– Example: CD compounded daily/paid
monthly
– Use effective rate as interest rate rather
than nominal rate
36
Student Workbook for HS 300
_________________________G7-13_________________________
More Frequent Compounding
Problem
Keystrokes
You invest $500 in a 2-year
CD that compounds
interest monthly but credits
it to your account annually.
12, [orange], P/YR
5, I/YR
[orange] EFF%
5.1162
I/YR
1,[orange], P/YR
500, +/–, PV
2, N
FV
552.47
The CD pays a 5 percent
nominal rate. What is the
maturity value?
37
Payments Growing by
Constant Percentage
38
Payments Growing by
Constant Percentage
• Annuity payment growing at constant rate
– Example: Inflation-indexed IRA contribution
limits
• On financial calculator, adjust expected
investment return by expected growth rate,
and then calculate time value of money
39
Student Workbook for HS 300
_________________________G7-14_________________________
Payments Growing by
Constant Percentage
Problem:
Cathy Client plans on
maximizing her contributions
to an IRA account until she
retires 30 years from now.
Assume a contribution today
of $3,000, growth in
contribution limits of 3
percent/year, and average
annual returns of 6 percent.
Keystrokes:
[orange] BEG/END {set to Begin}
1.06,÷,1.03,-,1, x,100,=
I/YR {sets I/YR to 2.9126}
3000,+/-, PMT
30, N
PV
61,203.24
+/-,PV, 0, PMT
6, I/YR, 30, N
FV
351,520.24
40
Chapter Seven Review
41
Chapter Seven Review
• Practice your calculator skills and enhance
your understanding of time-value-of-money
concepts.
• Know how to figure out when the answer on
the calculator is incorrect.
42
Student Workbook for HS 300
_________________________G7-15_________________________
Chapter Seven Review
• Do the problem twice
– Discounting to a present value will give a
result smaller than the sum of the cash
flows
– Future value will have a result greater than
the sum of cash flow for any positive, nonzero, interest rate
43
Student Workbook for HS 300
Financial Planning Applications
8
Overview
Class 8 covers three basic financial planning topics—funding a college education,
emergency fund planning, and credit and debt management. In addition to their
importance to financial advisors in providing professional financial planning services,
these topics also illustrate various applications of the concepts covered in earlier
chapters. For example, helping parents plan the funding of a college education for their
children requires estimating future costs that increase with time due to inflation as well
as periodic amounts of savings that increase with time due to investment return. Given
the influence of time, inflation, and investment return on the college-funding problem,
the financial advisor must apply several time-value-of-money concepts presented in
chapters 6 and 7—the future value of a single sum, the present value of a single sum,
and the present value of an annuity or an annuity due-in determining the amount that
must be saved periodically (for example, monthly) to accumulate the funds required to
pay for the college education.
The steps in the financial planning process presented in class one provide a format
for explaining the key activities involved in credit and debt management and how credit
and debt management is integrated with other planning techniques in developing a
client’s financial plan. Also, the client’s financial statements—the financial position
statement and the cash flow statement—discussed in class 5 are indispensable for
gathering client data, analyzing the client’s financial status, developing financial
planning recommendations and/or alternatives, and monitoring financial planning
recommendations in providing emergency fund planning and credit or debt management
services to clients.
Preparing for Class 8
√ Read chapter 8 in Financial Planning: Process and Environment, AT LEAST
TWICE.
√ Answer the review questions and self-test questions at the end of the chapter.
√ Study the key terms and concepts listed in the textbook.
8-2
√ Review the Study Tips for Class 8 in the student workbook, and the video
graphics starting on page G8–1.
√ Participate in the online community for this course by accessing AC Online
at <http://blackboard.theamericancollege.edu>. Explore the channels for this
course on AC Online including: Course Info, Modules, Exams, Forums, and
Tools. Stop by the Student Lounge, on the Forum channel to post a note to other
students in the course, or Office Hours to post a note for the course instructor.
Study Tips for Class 8
•
•
•
•
•
Focus on the importance of inflation in calculating education funding needs.
Know education benefits from the Taxpayer Relief Act of 1997. Note that
phaseouts eliminate these benefits for many clients.
Review the appropriateness of various education investment alternatives. If
your state offers a tuition account program, you may want to review its features.
Remember that, for most clients, education funding is not done in isolation
from other goals. Examine how retirement planning and estate planning are
integrated with education planning.
Review obtaining financial aid, especially the FAFSA formula and federal
government sources of funds.
Post-Class: Test Your Comprehension
√ Complete the Quick Quiz for class 8.
√ Complete the Key Terms and Concepts matching exercise for class 8.
Student Workbook for HS 300
8-3
Quick Quiz
Circle your responses:
T F
1. Four inputs are required to calculate education funding needs.
T F
2. Lifetime learning credits are available to all taxpayers, regardless
of income level.
T F
3. Stafford loans are education loans for which interest may be
subsidized for students with financial need.
T F
4. Grandparents can use a Section 2503(e) exclusion from taxable gifts
to make college tuition payments for grandchildren.
T F
5. The Free Application for Federal Student Aid determines the
expected parent contribution and expected student contribution and
combines them to arrive at the expected family contribution.
T F
6. Section 529 College Savings Plan contributions generate a federal
income tax deduction.
T F
7. The size of your overall portfolio will influence the size of your
emergency fund.
T F
8. The debt service ratio includes estimated federal income taxes.
T F
9. The lower your credit score the lower the interest rate on your loan.
T F
10. A bankruptcy filing remains on the petitioner’s credit report for 6
years.
Answers to Quick Quiz
1-F, 2-F, 3-T, 4-T, 5-T, 6-F, 7-T, 8-F, 9-F, 10-F
Student Workbook for HS 300
8-4
Key Terms and Concepts
Match the key term or concept to the most accurate definition.
a. asset allocation
__________ 1. ability to quickly convert an asset to cash with
little or no uncertainty as to value
b. debt burden ratio
__________ 2. compares a client’s total monthly consumer
credit payments (excluding mortgage
payments) with the amount of his or her
monthly take-home pay
c. expected parent
contribution (EPC)
__________ 3. how the portfolio is divided between different
types of investments
d. Federal Perkins loans
__________ 4. loan that has no collateral or assets backing the
loan agreement
e. liquidity
__________ 5. loans of up to $4,000 per year ($20,000
cumulative) for students who show exceptional
need
f. loan-to-value (LTV)
__________ 6. ratio of debt to the lower of appraised value or
market price
g. prepaid tuition plan
__________ 7. report that is sent out by the Department of
Education after it processes a free application
for student aid (FAFSA)
h. Roth IRA
__________ 8. total amount that a student’s parents are
expected to contribute annually for college
education before becoming eligible for financial
aid
i. student aid report (SAR)
__________ 9. type of individual retirement account in
which after-tax contributions are required, but
earnings and distributions after age 591/2 are
not taxed
j. unsecured debt
__________ 10. type of Section 529 plan that allows tuition
costs to be locked in years before enrollment
Student Workbook for HS 300
8-5
Answers to Key Terms and Concepts Matching
1.
2.
3.
4.
5.
6.
7.
8.
9.
10.
e.
b.
a.
j.
d.
f.
i.
c.
h.
g.
Student Workbook for HS 300
_________________________G8-1_________________________
HS 300 Financial Planning:
Process and Environment
Chapter 8:
Financial Planning
Applications
1
Learning Objectives
2
Learning Objectives
1. Identify the two funding requirements associated
with financing a college education.
2. Describe the process for calculating the resources
required to fund a college education including the
types of investments typically considered for this
purpose.
3. Describe the various tax deductions, tax credits, and
penalty waivers that are available to encourage
saving for a college education.
3
Student Workbook for HS 300
_________________________G8-2_________________________
Learning Objectives (Continued)
4. Explain how a college student’s financial need is
determined for purposes of obtaining federal student
aid, and describe the various types of federal
student aid available.
5. Explain why emergency fund planning is an
indispensable part of financial planning.
6. Explain how an adequate amount of funding for
emergencies is estimated and what types of
investments are most appropriate for this purpose.
4
Learning Objectives (Continued)
7.Identify the key types of household debt.
8.Explain the difference between secured and
unsecured debt.
9.Describe the activities involved in credit and
debt management and explain how they
relate to the steps in the financial planning
process.
5
Learning Objectives (Continued)
10. Explain the uses of the financial position
statement and the cash flow statement in
carrying out credit and debt management
activities.
11. Explain the ways in which leasing an
automobile differs from purchasing it, and
identify the key characteristics of people who
are particularly suited for leasing rather than
buying an automobile.
12. Explain how a lease’s monthly payment is
calculated.
6
Student Workbook for HS 300
_________________________G8-3_________________________
Learning Objectives (Continued)
13. Explain the differences between a home equity
loan and a home equity line of credit, and
calculate the maximum amount a consumer can
borrow under each.
14. Calculate the amount of home equity debt on
which interest is deductible for federal income
tax purposes.
15. Describe the key features and principal uses of
the three types of bankruptcy available to
consumers in the United States.
7
Discussion Break
Should Sec. 529 plan assets be included as
student assets, parental assets, or excluded
from the financial aid calculation?
8
Discussion Follow-up:
Sec. 529 Plan Assets
9
Student Workbook for HS 300
_________________________G8-4_________________________
Sec. 529 Plans:
Treatment for Financial Aid
• College savings account in parent’s name
with student as beneficiary
– Considered parent’s assets for Federal
Financial Aid (FFA)
10
Sec. 529 Plans:
Treatment for Financial Aid
• College savings account in student’s name,
or a trust or custodian for the student with
student as beneficiary
– Not considered student’s assets for FFA
11
Sec. 529 Plans:
Treatment for Financial Aid
• College savings account in third party’s name
(grandparents) with student as beneficiary
– Has no effect on FFA
12
Student Workbook for HS 300
_________________________G8-5_________________________
Sec. 529 Plans:
Treatment for Financial Aid
• College savings account in parent’s name,
with student’s siblings as beneficiaries
– Has no effect on FFA
13
Sec. 529 Plans:
Treatment for Financial Aid
• Prepaid tuition plan
– Legislation enacted in 2006 makes the tax
treatment consistent with the Section 529
Savings Plans
14
College Savings
15
Student Workbook for HS 300
_________________________G8-6_________________________
College Savings
• Education inflation
• Calculating the funding requirement
– Lump sum
16
College Savings
• Calculating the funding requirement
– Monthly savings
Current cost
Education inflation rate
Number of years until matriculation
Investment returns (tax status)
17
Selecting a Portfolio
• Taxable account
• Tax advantaged
– Coverdell Education Savings Account
– Sec. 529 Qualified Tuition Plans
– Savings bonds for education
– Roth and traditional IRAs
18
Student Workbook for HS 300
_________________________G8-7_________________________
Selecting a Portfolio
•
•
•
•
•
•
CDs
Savings bonds
Baccalaureate bonds
Stocks/bonds/cash
Mutual funds investing in stocks/bonds/cash
CollegeSure CDs
19
Whose Money?
20
Titling Accounts in Child’s Name
• Uniform Gift to Minors (UGMA)
• Uniform Transfers to Minors (UTMA)
• Tax code changes in 2007 to kiddie tax make
these approaches unattractive as a repository
for college savings
21
Student Workbook for HS 300
_________________________G8-8_________________________
Trusts
• Sec. 2503(b) trust
– Income paid annually to beneficiaries
– Trust can retain funds past age of majority
– Annual gift tax exclusion for income rights
– Delayed payment of principal until funds
are needed for college
22
Trusts
• Section 2503(c) trust
– Income can accumulate
– Delay payment of principal
– Trust payable at age of majority
23
Trusts
• Irrevocable trust with Crummey provision
– Allows income and principal
distribution/accumulation as needed
– Trust can last as long as the income
beneficiaries live
24
Student Workbook for HS 300
_________________________G8-9_________________________
Trusts
• Disadvantages
– FAFSA counts trusts as asset of child (35
percent expected contribution)
– Trust expenses
Set-up
Taxation
Administration
25
Sec. 2503(e)
• Exclusion from taxable gifts if grandparents
pay college costs directly to the college
26
Ownership Issues
• Kiddie tax impact
• Age of majority issues
• FAFSA implications
27
Student Workbook for HS 300
_________________________G8-10_________________________
Tax Incentives
28
Tax Credits
• Hope Scholarship Tax Credit
– $1,800/student/year first 2 years of
postsecondary education
Phaseout based on modified adjusted
gross income (MAGI)
Must be at least half-time student for at
least one academic period
29
Tax Credits
• Lifetime Learning Tax Credit
– $2,000 calculated as 20% first $10,000
qualified expenses
– Available for an unlimited number of years
– Student does not have to be pursuing
degree
– Phaseout based on MAGI
30
Student Workbook for HS 300
_________________________G8-11_________________________
Tax Deduction
• Deductibility of higher education expenses
– $4,000 maximum deduction
– Started in 2004
– Phaseout based on MAGI
– Use of tax credits and other taxadvantaged programs limits deduction
31
Interest Exclusion
• Savings bonds for education
– Phaseout subject to MAGI
– Bonds cannot be held in child’s name
32
Alternative Strategies
33
Student Workbook for HS 300
_________________________G8-12_________________________
Alternative Strategies
•
•
•
•
•
Investing in college housing
Family partnerships
401(k) plan loans
Cash value life insurance policy loans
Roth & traditional IRA distributions
34
Applying for Financial Aid
35
Free Application for
Federal Student Aid (FAFSA)
• Expected student contribution 35 percent
– Spend kid’s money first
• Expected parent contribution 5.6 percent
– Exemptions and exclusions drive down
asset base and expected contributions
36
Student Workbook for HS 300
_________________________G8-13_________________________
Free Application for
Federal Student Aid (FAFSA)
• Terminology
– Expected family contribution (EFC)
– Cost of attendance (COA)
– Student aid report (SAR)
37
Student Aid
• Federal Pell Grant
– Need-based
– Up to $4,050/year
38
Student Aid
• Federal Supplemental Education
Opportunity Grant (FSEOG)
– Need-based
– Up to $4,700/year
39
Student Workbook for HS 300
_________________________G8-14_________________________
Student Aid
• Federal Perkins loans
– Need-based
– $4,700/year
– $20,000 cumulative
40
Student Aid
• Stafford loans
– Available from Direct Loans or FFEL
– Subsidized (need based): interest deferred
– Unsubsidized: interest accrues
– Variable interest rate set annually
41
Student Aid
• PLUS loans
– Available from Direct Loans or FFEL
– Interest accrues from disbursement
• College resources
– Scholarships
– Grants
42
Student Workbook for HS 300
_________________________G8-15_________________________
Emergency Fund Planning
43
Emergency Fund
• Keeps the financial ship afloat in fiscal storms
– 3 to 6 months’ worth of living expenses
– Cash or liquid investments
44
Emergency Fund
• Keeps the financial ship afloat in fiscal storms
– Amount influenced by
Job security
Wages/salary/commission/bonus
Characteristics of investment portfolio
Other available sources of cash
45
Student Workbook for HS 300
_________________________G8-16_________________________
Credit and Debt Management
46
Credit and Debt Management
• Mortgage debt (secured)
– First mortgage
– Second mortgage
Home equity loan
Home equity line of credit (HELOC)
47
Credit and Debt Management
• Consumer debt
– Personal loans
– Auto loans (secured)
– Credit card debt
48
Student Workbook for HS 300
_________________________G8-17_________________________
Credit and Debt Management
• Client financial statements
• Credit report
• Credit score
49
Credit and Debt Management
• Five C(s) of credit
– Character
– Capacity
– Collateral
– Capital
– Conditions
50
Ratio Analysis
• Debt service ratio
Total loan payments/net income
• Front ratio (28%)
Housing costs (PITI)/gross income
• Back ratio (36%)
Total debt/gross income
51
Student Workbook for HS 300
_________________________G8-18_________________________
Financing Alternatives
52
Leasing Versus Buying a Car
• Open versus closed-end lease
• Capitalized cost
• Money factor = interest rate/24
53
Leasing Versus Buying a Car
• Residual value
– Monthly depreciation
– Monthly lease rate
– Monthly sales tax
54
Student Workbook for HS 300
_________________________G8-19_________________________
First Mortgages
• First mortgage
– 80 percent LTV is the cutoff for PMI
– FHA and VA have government guarantees
– Fixed 30/15
55
First Mortgages
• First mortgage
– Adjustable (ARM)
– Hybrid
– Interest only
56
Discussion Break
When should a client prepay his or her
mortgage— either from the perspective of
paying it off or making additional principal
payments each month?
Would your answer change if you had an
assets-under-management agreement with the
client?
57
Student Workbook for HS 300
_________________________G8-20_________________________
Financing Alternatives
(Continued)
58
Second Mortgages
• Status as second means LTV is critical to
lender
• Home equity loan
– Fixed rate
– Fixed term
– Fixed payment
59
Second Mortgages
• Home equity line of credit (HELOC)
– Variable rate
Pricing index plus spread
Caps/floors/collars
Interest only payments in early years
– Hybrids available
60
Student Workbook for HS 300
_________________________G8-21_________________________
Bankruptcy After the 2005
Bankruptcy Reform Act
There is now a means-based test to determine who
can qualify for a Chapter 7 filing.
Mandatory credit counseling and financial education
became part of the bankruptcy process.
You cannot file a Chapter 7 bankruptcy if you
obtained a discharge of your debts in a Chapter 7
case within the last 8 years, or a Chapter 13 case
within the last 6 years.
61
Chapter 7 Bankruptcy
•
Chapter 7 liquidation
– Formation of bankruptcy estate from
petitioner’s eligible assets used to pay
creditors
– Court discharges all eligible debts
– Stays on credit report for 10 years
62
Chapter 13 Bankruptcy
• Petitioner must have income that exceeds
reasonable living expenses
• Three-year repayment plan
• Eligible debts remaining after completion of
repayment plan are discharged by the bankruptcy
court
• Debtor retains property whether exempt or nonexempt
• Filing remains on credit report for 7 years
63
Student Workbook for HS 300
_________________________G8-22_________________________
Chapter 11 Reorganization
• Less common for individuals
• Plan must be approved by a majority of
creditors in each class and by enough
creditors in each class to equal 2/3 of the
indebtedness in that class
• Debtor retains possession of assets and
continues to operate business
64
Chapter Eight Review
65
Chapter Eight Review
• Three main applications
– College funding
– Emergency fund planning
– Debt management
• Know your client and make appropriate
recommendations
66
Student Workbook for HS 300
The Regulation of Financial Advisors
9
Overview
Financial planning as a profession continues to grow and evolve. The growth of the
financial services industry that started in the late 1970s and continues through today
makes the regulation of financial advisors an increasingly important topic to investors,
consumer advocates, regulatory authorities, elected officials, and even to financial
advisors themselves. This class reviews federal legislation and regulatory actions that
govern the activities of financial planners.
A good example of the rapidly changing regulatory environment is an important piece
of legislation that Congress passed and President Clinton signed in October 1996. As
explained later in the chapter, the Investment Advisers Supervision Coordination Act of
1996 changed the regulatory environment significantly. As of July 1997, the Securities
and Exchange Commission (SEC) no longer requires financial planners with less than
$30 million of assets under management to register with the SEC as investment advisers.
Exceptions to this exemption include advisors to investment companies and financial
planners in states that have no investment adviser laws. In effect, regulation of "small"
investment advisers becomes the exclusive purview of the states.
In the current decade, SEC rules have increasingly focused on compliance and ethical
standards and established the regulatory requirements of investment advisers practicing
predominately on the Internet. This class looks at the monitoring of these issues as well.
Preparing for Class 9
√ Read chapter 9 in Financial Planning: Process and Environment, AT LEAST
TWICE.
√ Answer the review questions and self-test questions at the end of the chapter.
√ Study the key terms and concepts listed in the textbook.
√ Review the Study Tips for Class 9 in the student workbook, and the video
graphics starting on page G9-1.
9-2
√ Participate in the online community for this course by accessing AC Online
at <http://blackboard.theamericancollege.edu>. Explore the channels for this
course on AC Online including: Course Info, Modules, Exams, Forums, and
Tools. Stop by the Student Lounge, on the Forum channel to post a note to other
students in the course, or Office Hours to post a note for the course instructor.
Study Tips for Class 9
•
•
Review the three tests provided in SEC Release No. IA-770 for determining
whether (in the absence of specific exemptions) a financial services
professional’s activities are subject to regulation under the Investment Advisers
Act of 1940. Remember that if any of the tests are not met, the Investment
Advisers Act of 1940 would not apply to the practitioner’s activities and the
planner would not have to register.
Know the six groups who are not subject to the Investment Advisers Act of
1940. The act does not apply to any of their activities. Keep in mind that
lawyers and accountants are not automatically excluded.
– Know the investment advisers who are exempted from registration
with the SEC under the Investment Advisers Act of 1940. Be sure you
understand the issue of state versus SEC registration.
– Also, make sure you understand how the Investment Advisers
Supervision Coordination Act of 1996 changed the rules for state and
federal registration. The vast majority of investment advisers must
register at the state level and do not need to register at the federal level.
•
•
•
Know the seven categories of responsibilities of investment advisers who are
subject to SEC regulation and be able to expand upon several of them.
Review the list of 17 record-keeping requirements in the text.
When reviewing the brochure rule, note that most RIAs use part II of Form
ADV, rather than creating a separate brochure. Form ADV is less expensive
and less likely to be challenged by the SEC, though obviously Form ADV lacks
marketing "sizzle."
Post-Class: Test Your Comprehension
√ Complete the Quick Quiz for class 9.
√ Complete the Key Terms and Concepts matching exercise for class 9.
Student Workbook for HS 300
9-3
Quick Quiz
Circle your responses:
T F
1. The term security is defined in the Investment Advisers Act of 1940
to mean stocks and bonds.
T F
2. An individual who receives only a commission on the sale of a
securities product (but is paid no fee for giving investment advice)
does not meet the compensation test in SEC Release No. IA-770.
T F
3. All financial services professionals who meet all three of the tests
contained in SEC Release No. IA-770 are subject to regulation
under the Investment Advisers Act of 1940.
T F
4. Under certain circumstances, a lawyer, an accountant, and even a
securities broker may be exempt from the Investment Advisers Act
of 1940.
T F
5. The SEC has issued a ruling that an investment adviser must
disclose to clients all material facts regarding potential conflicts of
interest.
T F
6. The Investment Advisers Supervision Coordination Act specifies
that investment advisers must register with the SEC unless they
have more than $25 million in assets under management.
T F
7. Form ADV requires disclosure of information concerning the types
of clients for which the registrant will provide discretionary or other
account management services.
T F
8. Registered investment advisers are exempt from federal rules and
regulations prohibiting insider trading.
T F
9. The SEC prohibits advisers from using the initials RIA after their
names on business cards or letterheads.
T F
10. The brochure rule can be satisfied by providing the client with a
copy of part II of the Form ADV.
Answers to Quick Quiz
1-F, 2-F, 3-F, 4-T, 5-T, 6-F, 7-T, 8-F, 9-T, 10-T
Student Workbook for HS 300
9-4
Key Terms and Concepts
Match the act or release with the provision of that act or release.
a. Insider Trading Sanction
Act of 1984
__________ 1. specifies how to determine an investment
adviser’s assets under management, which is
the determining factor in whether the adviser
must register with the SEC or the state
b. Insider Trading and
Securities Fraud
Enforcement Act of 1988
__________ 2. sets forth three separate tests to determine
whether the Investment Advisers Act of 1940
applies to a financial advisor’s activities
c. Investment Advisers Act of
1940
__________ 3. says investment advisers must have written
policies and procedures that reduce the
likelihood of insider trading and securities
fraud
d. Investment Advisers
Supervision Coordination
Act of 1996
__________ 4. requires registered investment advisers
to name a compliance officer, write and
implement compliance procedures, and
review compliance annually
e. SEC Release No. IA-770
__________ 5. requires registered investment advisers to
adopt and follow a code of ethics
f. SEC Release No. IA-1092
__________ 6. requires investment advisers to register with
either the SEC or state authorities, but not
both, based on level of assets under advisor’s
management
g. SEC Release No. IA-1633
__________ 7. reaffirmed the three-prong approach set forth
in IA-770 and clarified certain antifraud and
full disclosure provisions of the Investment
Advisers Act of 1940
h. SEC Release No. IA-2091
__________ 8. gives the SEC the authority to seek the
imposition of civil penalties against
insider-trading violations for as much as
three times the profit gained (or loss avoided)
as a result of the unlawful purchase or sale
of securities
i. SEC Release No. IA-2204
__________ 9. controls the regulation of investment advisers
j. SEC Release No. IA-2256
__________ 10. allows investment advisers operating
principally on the Internet, with no
connection to a particular state for operations,
to be SEC registered rather than state
registered
Student Workbook for HS 300
9-5
Answers to Key Terms and Concepts Matching
1.
2.
3.
4.
5.
6.
7.
8.
9.
10.
g.
e.
b.
i.
j.
d.
f.
a.
c.
h.
Student Workbook for HS 300
_________________________G9-1_________________________
HS 300 Financial Planning:
Process and Environment
Chapter 9:
The Regulation of Financial
Advisers
1
Learning Objectives
2
Learning Objectives
1. Describe tests in Investment Advisers Act of
1940 that determine if financial adviser
must register with SEC as an investment
adviser.
2. Identify financial advisers who are not
required to become registered investment
advisers with the SEC.
3
Student Workbook for HS 300
_________________________G9-2_________________________
Learning Objectives
3. Summarize how Investment Advisers
Supervision Coordination Act of 1996
determines whether investment advisers
must register at federal or state level.
4. Describe the procedure for becoming a
registered investment adviser and the
requirements imposed on one who does
register.
4
Learning Objectives
5. Discuss how self-regulatory organizations
relate to SEC’s oversight of registered
representatives and investment advisers.
6. Discuss responsibility of fiduciary to his or
her client regarding financial planning.
7. Differentiate between registered
representative’s suitability standards and
registered investment adviser’s fiduciary
standards.
5
Discussion Break
There are a host of financial planning
designations, but a recent Morningstar article
suggested that for broad financial planning
assistance an individual choose between a:
– CFP
– ChFC®
– CPA-PFS
(continued)
6
Student Workbook for HS 300
_________________________G9-3_________________________
Discussion Break
Would it be better for the industry to move to
one designation or is the current mix more
appropriate...
in terms of client service?
in terms of marketing your practice?
in terms of regulating the industry?
7
How Planners Became
Investment Advisers
8
How Planners Became
Investment Advisers
• Federal or state governments regulate the
activities of planners
– selling
– recommending investments and insurance
• SEC wanted to control security sales
9
Student Workbook for HS 300
_________________________G9-4_________________________
Laws That Govern
Securities Industry
•
•
•
•
Securities Act of 1933
Securities Exchange Act of 1934
Public Utility Holding Company Act of 1935
Trust Indenture Act of 1939
10
Laws That Govern
Securities Industry
• Investment Company Act of 1940
• Investment Advisers Act of 1940
• Sarbanes-Oxley Act of 2002
11
SEC Release No. IA-770
• Three-pronged test to determine if person
needs to register as adviser (1981)
– Advice or analysis about a security
– Business standard
– Compensation test
12
Student Workbook for HS 300
_________________________G9-5_________________________
SEC Release IA-1092
• Confirms three-pronged standard for
registration under Investment Adviser Act of
1940 (1987)
• Antifraud provisions implemented
13
Exceptions and Exemptions
From Investment Advisers Act
of 1940
14
Exclusions From
SEC Registration
• Banks or bank holding companies defined in
the Bank Holding Company Act of 1956 and
not an investment company
• Lawyers, accountants, engineers, or teachers
whose advisory services are incidental to
profession
15
Student Workbook for HS 300
_________________________G9-6_________________________
Exclusions From
SEC Registration
• Brokers, dealers, or registered
representatives whose performance of
advisory services is solely incidental to
business as a broker or dealer and receives
no “special compensation”
• Publisher of any newspaper, news magazine,
or business or financial publication of general
or regular circulation
16
Exclusions From
SEC Registration
• Person whose advice, analyses, or reports
relate only to securities that are direct
obligations of or obligations guaranteed by
U.S. government
• Other persons not within intent of law as SEC
may designate
17
Exemptions From
SEC Registration
• Investment advisers
– Clients are all residents of state where
principal office and place of business are
maintained
– Does not furnish advice, analyze, or report
on listed securities or securities admitted to
unlisted trading privileges on national
securities exchange
18
Student Workbook for HS 300
_________________________G9-7_________________________
Exemptions From
SEC Registration
• Investment advisers
– Insurance companies are only clients
– Fewer than 15 clients during previous 12
months
– Neither presents themselves to the general
public as investment adviser nor acts as
investment adviser to any registered
investment company
19
Exemptions From
SEC Registration
• Investment advisers
– Is employed by a charity
– Advises, analyzes, or reports only to
charity or charitable funds
– Provides investment advice exclusively to
church employee pension plans
20
Coordination Act of 1996 and
SEC Release IA-1633
21
Student Workbook for HS 300
_________________________G9-8_________________________
Coordination Act and
National Securities Market
Improvement Act of 1996
• Allocates regulation of investment advisers
between the SEC and state authorities
– Based on the level of assets under an
adviser’s management
• Register either with SEC or state
authorities, but not both
22
“Not Prohibited” From
SEC Registration
• Advisers with “assets under management” of
$25 million or more
• Advisers to registered investment companies
23
“Not Prohibited” From
SEC Registration
• Advisers not “regulated or required to be
regulated” in state of “principal office and
place of business”
• Advisers exempted from prohibition by SEC
rule or order
24
Student Workbook for HS 300
_________________________G9-9_________________________
Defining Assets Under Management
(AUM)
• Sec. 203A(a)(2) definition
– Securities portfolio
– Continuous and regular supervisory or
management services
– Accounts with 50 percent securities
including cash or cash equivalents
25
Advisers to Registered
Investment Companies
• Must register with SEC regardless of asset
amount if
– Contracted to provide advisory services
– Includes sub-advisers
• Investment company must be
– Registered under Investment Company Act
of 1940
– Operational
26
Advisers Not Regulated or Required
to Be Regulated by State
• Advisers with principal office in Wyoming
• Advisers with principal office in a foreign
country
27
Student Workbook for HS 300
_________________________G9-10_________________________
Advisers Exempt From Prohibition
on SEC Registration
• Must register with SEC if
– Nationally recognized statistical rating
organizations
– Pension consultants, including advisers of
government plans and church plans, if
value of assets of plans is $50 million or
more
28
Advisers Exempt From Prohibition
on SEC Registration
• Must register with SEC if
– Certain affiliates under control of
investment advisers are eligible for or are
registered with the SEC
– Certain start-up investment advisers with
expectation of eligibility to register with the
SEC within 120 days
29
Other Aspects of
Coordination Act
30
Student Workbook for HS 300
_________________________G9-11_________________________
De Minimis Standard
• Relieves advisers from regulatory burden
– Not required to register in state unless
place of business is there
– Not required to register in state unless
more than five clients were residing in state
during preceding 19 months
31
De Minimis Standard
• No de minimis rule for federally registered
investment advisers in Texas, New
Hampshire, and Nebraska
– Texas does not recognize the de minimis
standard for state-registered investment
advisers
32
De Minimis Standard
• No de minimis rule for federally registered
investment advisers in Texas, New
Hampshire, and Nebraska
– State-registered investment advisers
permitted five clients in New Hampshire
and Nebraska, unlike federal RIAs
– Whole firm is only allowed five clients or
less under de minimis rules
33
Student Workbook for HS 300
_________________________G9-12_________________________
Supervised Persons
• Sec. 203A(b) preempts state law re:
supervised persons of SEC-registered
advisers
– State may continue to license, register, or
otherwise qualify supervised person who is
investment adviser representative and has
business located in state
34
Supervised Persons
• Sec. 202(a)(25) defines as “partner, officer,
director ... or employee of an investment
adviser, or person who provides investment
advice on behalf of investment adviser and is
subject to supervision and control of
investment adviser.”
35
Investment Adviser Representative
• Rule 203A-3(a) defines as
– Supervised person of an investment
adviser if clients represent more than
10 percent of clients of supervised person
36
Student Workbook for HS 300
_________________________G9-13_________________________
Federal Laws and Regulations
Applying to State-Registered
Advisers
• Sec. 206 makes unlawful fraudulent,
deceptive, or manipulative practices
• Sec. 204A requires establishment,
maintenance, and enforcement of written
procedures to prevent the misuse of
nonpublic information (insider trading)
37
Federal Laws and Regulations
Applying to State-Registered
Advisers
• Sec. 205
– Prohibits advisory contracts that
(1) contain certain performance fees
(2) permit assignment of contract without
the consent of the client, and
(3) fail to notify clients of a change in
membership of partnership
38
Federal Laws and Regulations Apply
to State-Registered Advisers
• Sec. 206(3)
– Unlawful for any investment adviser acting
as principal for own account to knowingly
sell security to or purchase security from, a
client, without written disclosure to client of
capacity in which adviser is acting and
obtaining the client’s consent
39
Student Workbook for HS 300
_________________________G9-14_________________________
Federal Laws and Regulations Apply
to State-Registered Advisers
• Sec. 206(3)
– Limitation also applies if acting as broker
for person other than client in effecting
such a transaction
40
SEC Release No. IA-2091
• Exemption for certain investment advisers
operating through the Internet
– Became effective in 2003
– Permits advisers, whose businesses are
not connected to particular state, to
register with SEC instead of with state
41
Discussion Break
IA-2091 Exemption for Certain Investment
Advisers Operating Through the Internet
permits SEC registration for web-based
practices.
I’d like you to to speculate how an Internetbased practice would differ from a bricks-andmortar, meet-and-greet operation.
42
Student Workbook for HS 300
_________________________G9-15_________________________
Responsibilities of
Investment Advisers
Who Must Register With SEC
43
Responsibility Requirement
Categories
1.
2.
3.
4.
5.
6.
7.
Registering
Record keeping
Charging fees
Assigning contracts
Using labels
Delivering brochures
Avoiding fraudulent practice
44
Record-Keeping Requirements
• 17 steps in process
• SEC Release No. IA-2256
– RIAs must adopt code of ethics addressing
compliance issues including record
keeping
45
Student Workbook for HS 300
_________________________G9-16_________________________
SEC Release No. IA-2204
Compliance Programs
• Investment advisers and investment
companies registered with the SEC
– Must adopt and implement written policies
and procedures to prevent violation of the
federal securities laws
– Must annually review policies and
procedures
46
SEC Release No. IA-2204
Compliance Programs
• Designate a chief compliance officer to
administer the policies and procedures
– Reports directly to the fund’s board if an
investment company
47
Brochure Rule
• Mandates delivery of disclosure statement to
existing and prospective clients
(1) delivered within 48 hours of entering into
investment advisory agreement or
(2) delivered when contract is entered into if
client can terminate such contract within 5
days
48
Student Workbook for HS 300
_________________________G9-17_________________________
Brochure Rule
• Advisers comply with rule by providing client
or prospect with copy of Part II of Form ADV
• 14 basic categories of data that brochure
must communicate to client
49
State Regulations
50
State Regulations
• Exam requirements
– Series 63 and 65, 66 and 7
• Capital requirements
• Bond
51
Student Workbook for HS 300
_________________________G9-18_________________________
State Regulations
• Review
– Audited financial statements
– Contracts
– Disclosure statements
– Marketing literature
52
Self-Regulatory Organizations
(SROs)
53
Self-Regulatory Organizations
(SROs)
• National securities exchange, registered
securities association, or registered clearing
agency authorized by Securities Exchange
Act of 1934 to regulate conduct and activities
of its members
• Subject to oversight by specific government
regulatory agency
54
Student Workbook for HS 300
_________________________G9-19_________________________
Self-Regulatory Organizations
(SROs)
• SROs with SEC oversight
– National Association of Securities Dealers
(NASD) and New York Stock Exchange
(NYSE) recently merged as the Financial
Industry Regulatory Authority (FINRA)
– American Stock Exchange (AMEX)
55
Professional Regulatory
Organization (PRO)
• CFP Board
– Acts as professional regulatory
organization for its members
– Provides code of ethics and practice
standards
– Issues advisory opinions
56
Fiduciary Duty
57
Student Workbook for HS 300
_________________________G9-20_________________________
• A recent appellate court victory in Financial
Planning Association (FPA) v. the SEC
determined that the SEC’s controversial “Merrill
Lynch Rule” was in conflict with the Investment
Advisers Act of 1940.
• Registered representatives offering fee-based
brokerage accounts will need to change the
account relationship or potentially be required to
register as investment advisers under the 1940
act. This is a major change in the regulation of
financial advisers since registered investment
advisers are held to a fiduciary standard in their
dealings with clients versus the suitability
standard required of registered representatives.
58
Fiduciary Duty
• Higher standard than product suitability
• Follows prudent-man standard
– Would a prudent person take this course of
action?
59
Chapter Nine Review
60
Student Workbook for HS 300
_________________________G9-21_________________________
Chapter Nine Review
• Hard to avoid registering as an adviser if you
are doing comprehensive planning
– State or SEC
• Registration and compliance issues are
dynamic
– Practice has to adapt to changing
requirements
61
Chapter Nine Review
• Compliance is at the forefront with recent
scandals in
– mutual fund trading
– investment analysis
– other financial services company business
practices
• Fiduciary standards are important to planners
held to that standard in advising clients
62
Student Workbook for HS 300
The Legal and Economic Environment
of Financial Institutions
10
Overview
While class 9 focused on the advisor’s regulatory responsibility to the client, this
class looks at the function, purpose, and regulation of the financial services industry and
the corresponding consumer protection laws that relate to financial services. It is better
to take the broader perspective in learning about the regulatory environment rather than
getting caught up in the minutia of an individual act. Get a sense of what the regulations
are trying to achieve, how regulations are a product of their times, and how the passage
of time that shifts the focus of regulation can help you differentiate between what is
required as an advisor and how those requirements are changing. This assignment also
looks at the body of consumer protection acts, the regulators who uphold these acts, and
the trends in consumer protection legislation.
Preparing for Class 10
√ Read chapter 10 in Financial Planning: Process and Environment, AT LEAST
TWICE.
√ Answer the review questions and self-test questions at the end of the chapter.
√ Study the key terms and concepts listed in the textbook.
√ Review the Study Tips for Class 10 in the student workbook, and the video
graphics starting on page G10-1.
√ Participate in the online community for this course by accessing AC Online
at <http://blackboard.theamericancollege.edu>. Explore the channels for this
course on AC Online including: Course Info, Modules, Exams, Forums, and
Tools. Stop by the Student Lounge, on the Forum channel to post a note to other
students in the course, or Office Hours to post a note for the course instructor.
Study Tips for Class 10
•
Know the reasons behind government regulation
10-2
•
•
•
•
•
•
•
•
Name who benefits from government regulation
Be able to discuss how regulation can foster innovation
Be able to discuss current trends in financial services regulation
Be able to identify the principal acts and laws in financial services regulation
Name the regulators in consumer protection legislation
Discuss the role of consumer protection legislation
Be able to identify the principal acts and laws in consumer protection legislation
Be able to name current trends in consumer protection legislation.
Post-Class: Test Your Comprehension
√ Complete the Quick Quiz for class 10.
√ Complete the Key Terms and Concepts matching exercise for class 10.
Student Workbook for HS 300
10-3
Quick Quiz
Circle your responses:
T F
1. Financial services regulation can influence monetary policy.
T F
2. The Great Depression created the need for greater government
regulation.
T F
3. McCarran-Ferguson made insurance a federally regulated industry.
T F
4. The SEC was established by the Investment Company Act of 1940.
T F
5. Gramm-Leach-Bliley is important for repealing the Glass-Steagall
Act.
T F
6. The OCC regulates national banks.
T F
7. The Public Company Accounting Oversight Board was created by
the Sarbanes-Oxley Act.
T F
8. The F.T.C. regulates most consumer protection acts.
T F
9. Most regulators provide synopses of their acts as well as links to the
acts in U.S. Code on their websites.
T F
10. State attorney generals can influence federal regulation of financial
services firms.
Answers to Quick Quiz
1-T, 2-F, 3-F, 4-F, 5-T, 6-T, 7-T, 8-T, 9-T, 10-T
Student Workbook for HS 300
10-4
Key Terms and Concepts
Match the act with the provision of that act.
a. Competitive Equality Banking
Act of 1987
__________ 1. allows consumers to get one free copy
of their credit report annually from each
consumer-reporting agency
b. Consumer Credit Reporting
Reform Act
__________ 2. authorized the SEC to award bounty to
informants with the bounty coming from
money paid as penalties under the act
c. Depository Institution Act of
1982 (Garn-St. Germain)
__________ 3. designed to prevent terrorists and others
from anonymously using the U.S. financial
system to move funds obtained from or
destined for illegal activity
d. Fair and Accurate Credit
Transaction Act of 2003
__________ 4. established new standards for expedited
funds availability and recapitalized
the Federal Savings & Loan Insurance
Company (FSLIC)
e. Federal Deposit Insurance
Corporation Improvement Act
of 1991
__________ 5. established standards for trust indentures
in sale of debt securities such as bonds,
debentures, and notes offered for public
sale
f. Identity Theft and Assumption
Deterrence Act
__________ 6. expanded FDIC powers to assist troubled
banks
g. Insider Trading and Securities
Fraud Enforcement Act of 1988
__________ 7. made the theft of identity information
a crime and established restitution
provisions for individual victims
h. International Money Laundering
Abatement and Financial
Anti-Terrorism Act of 2001
__________ 8. permits adequately capitalized and
managed bank holding companies to
acquire banks in any state one year after
enactment
i. Riegle-Neal Interstate Banking
and Branching Efficiency Act of
1994
__________ 9. recapitalized the Bank Insurance Fund
and allowed the FDIC to strengthen the
fund by borrowing from the Treasury
j. Trust Indenture Act of 1939
__________ 10. strengthened the provisions of the Fair
Credit Reporting Act
Student Workbook for HS 300
10-5
Answers to Key Terms and Concepts Matching
1.
2.
3.
4.
5.
6.
7.
8.
9.
10.
d.
g.
h.
a.
j.
c.
f.
i.
e.
b.
Student Workbook for HS 300
_________________________G10-1_________________________
HS 300 Financial Planning:
Process and Environment
Chapter 10:
The Legal and Economic
Environment of Financial
Institutions
1
Learning Objectives
2
Learning Objectives
1. Learn why financial institutions are so
heavily regulated.
2. Understand the body of laws governing
financial institutions, how to research laws,
and find guidance in interpreting them.
3. Evaluate how deregulation is reshaping the
financial services sector.
3
Student Workbook for HS 300
_________________________G10-2_________________________
Learning Objectives
4. Evaluate the trends in regulating the
financial services industry.
5. Understand body of consumer protection
laws and how to research those laws and
find assistance in interpreting them.
6. Evaluate trends in regulating consumer
protection laws.
7. Describe the elements of supply and
demand.
4
Discussion Break
Why study financial services and consumer
protection regulation?
– How does it impact your business?
– How does it influence your clients?
5
Who and Why of Regulation
6
Student Workbook for HS 300
_________________________G10-3_________________________
Financial Services Industry
•
•
•
•
•
Banking
Securities
Insurance
Mutual funds
Pensions
7
What Regulation Accomplishes
• Public confidence
– Safety of principal
• Depression era issues
• S&L crisis issues from deregulation
8
What Regulation Accomplishes
• Open access to markets
• Monetary policy
– Open market operations
– Reserve requirements
– Margin requirements
9
Student Workbook for HS 300
_________________________G10-4_________________________
Safety of Principal
10
Safety of Principal
• FDIC
– $100,000 per depositor
• Full faith and credit
• FSLIC subsumed by FDIC
– Higher limits ($250,000 per depositor) available
for certain retirement accounts as separate
insurance amount in addition to $100,000 per
depositor.
– Trusts and payable on death (POD) accounts can
increase insurance coverage
11
Safety of Principal
• NCUSIF
– $100,000 per depositor
• Full faith and credit
– Higher limits ($250,000 per depositor)
available for certain retirement accounts as
separate insurance amount in addition to
$100,000 per depositor.
– Trusts and payable on death (POD)
accounts can increase insurance coverage
12
Student Workbook for HS 300
_________________________G10-5_________________________
Safety of Principal
• SIPC $500,000/$100,000 cash
– Protects against brokerage firm failing
• Does not protect investor from trading
losses
• Does not protect against being sold
fraudulent securities
13
Safety of Principal
• Pension Benefit Guaranty Corporation
(PBGC)
– Quasi-federal institution not backed by full
faith and credit of the government
14
Safety of Principal
• Insurance premiums in banking
– Risk-based modeling
15
Student Workbook for HS 300
_________________________G10-6_________________________
Safety of Principal
• Bank for International Settlements
– Basle Accords of 1993 & 2007
• Addressed capital adequacy
• Went beyond simple equity/total assets
approach to two risk-based capital
measures
• For member countries of Bank for
International Settlements
16
Who Benefits From FSI Regulation?
• Government
– Public confidence
– Monetary policy
• Consumers
– Public confidence
– Open access
17
Who Benefits From FSI Regulation?
• FSI firms
– Public confidence
– Margins (Stigler)
– Barriers to entry (Stigler)
– Regulation fosters innovation (Kane)
• CDARs example
18
Student Workbook for HS 300
_________________________G10-7_________________________
FDIC Limits Adjusted for Inflation
$250,000
Lim it A djus ted for CP I
s inc e Las t Change in
Lim it
$200,000
$150,000
FDIC Lim it
$100,000
Lim it A djus ted for CP I
s inc e $5,000 Lim it P ut
in E ffec t
$50,000
2004
1997
1990
1983
1976
1969
1962
1955
1948
1941
1934
$0
19
Deregulation
20
Deregulation in Financial Services
• May Day 1975
– Negotiated commission structure for
brokerage industry
21
Student Workbook for HS 300
_________________________G10-8_________________________
Deregulation in Financial Services
• Major deregulation in 1980
– Req Q abolished
– Interest on checking was allowed
– S&Ls could offer checking and commercial
loans
– Commercial banks could offer savings and
mortgages
22
Deregulation in Financial Services
• Gramm-Leach-Bliley Act 1999
– Repealed last vestiges of Glass-Steagall
Banks as
• Securities underwriters
• Insurance underwriters
• Insurance agents
23
Deregulation in Financial Services
• Gramm-Leach-Bliley Act 1999
– Important financial privacy legislation
provisions
 Requires companies to give consumers
privacy notices
 Consumers have the right to limit
sharing of their information
24
Student Workbook for HS 300
_________________________G10-9_________________________
Current Trends in Regulation
25
Current Trends in FSI Regulation
• FPA v. SEC
– The ruling from the appeals court,
combined with the SEC’s decision not to
appeal that ruling, means that brokerage
firms can no longer use the “Merrill Rule”
exemption to avoid complying with the
provisions of the Investment Advisers Act
of 1940.
26
Current Trends in FSI Regulation
• Sarbanes-Oxley Act
– Debate over the cost-benefit ratio in
compliance with the Sarbanes-Oxley Act,
especially for small firms, may point to a
modification of that act to provide
regulatory relief from full compliance.
27
Student Workbook for HS 300
_________________________G10-10_________________________
Current Trends in FSI Regulation
• Sarbanes-Oxley Act
– The compliance discussions also consider
whether the act has had a positive or
negative effect on the flow of capital to
U.S. capital markets, as corporations
consider listing their company’s stock on
foreign exchanges to avoid “Sarbox”
requirements.
28
Discussion Break
If it’s true that regulation fosters innovation,
regulators have to choose between being
reactive or proactive when changing rules and
acts.
When is it better to be proactive, and when is it
better to be reactive?
29
Select Regulatory Acts
30
Student Workbook for HS 300
_________________________G10-11_________________________
Securities Act of 1933
• “Truth-in-securities” law
• Two basic objectives:
– Requires that investors receive significant
information concerning securities being
offered for public sale
– Prohibits deceit, misrepresentations, and
other fraud in the sale of securities
31
Securities Exchange Act of 1934
• Created the SEC
– Empowered to register, regulate, and
oversee securities firms, transfer agents,
clearing agencies and SROs
• Identifies and prohibits certain conduct in
markets
32
Securities Exchange Act of 1934
• Granted SEC disciplinary powers over
regulated entities and associates
• Requires periodic reporting of information by
companies with publicly traded securities
33
Student Workbook for HS 300
_________________________G10-12_________________________
Banking Act of 1933
(Glass-Steagall Act)
• Created FDIC
– Established temporary agency to insure
deposits
– Limited banks to underwrite securities to
issuances for governmental bodies
• Permitted statewide branching for national
banks if allowed by governing state law
• Made permanent by the Banking Act of 1935
34
Investment Company
Act of 1940
• Regulates organization of companies that
engage in investing, reinvesting, and trading
in securities, and whose own securities
offered to public
– Includes mutual funds
35
McCarran-Ferguson Act of 1945
• Regulation of insurance remains with states
• NAIC helped establish regulatory bodies in
states
• Federal government influences state laws
through prospect of federal regulation in the
absence of state controls
36
Student Workbook for HS 300
_________________________G10-13_________________________
Employee Retirement Income
Security Act of 1974 (ERISA)
• Set minimum standards for pension plans in
private industry
• Created the Pension Benefit Guaranty
Corporation (PBGC)
• Requires that employers who establish plans
must meet certain minimum standards
• Has broadened in scope since initial passage
37
Gramm-Leach-Bliley
Act of 1999
• Repealed Glass-Steagall
• Instituted privacy requirements for financial
information
38
Sarbanes-Oxley Act of 2002
• Mandated reforms to enhance corporate
responsibility and financial disclosures
• Combats corporate and accounting fraud
• Created the Public Company Accounting
Oversight Board (PCAOB)
– Oversees auditing profession
39
Student Workbook for HS 300
_________________________G10-14_________________________
Charting, Regulation, and
Oversight
40
Chartering, Regulation, and
Oversight
• Depository institutions
– OCC
• Oversight of national banks
• State banks chartered by states
41
Chartering, Regulation, and
Oversight
• Depository Institutions
– FED
• Safe and sound banking
• Lender of last resort
• Protection of consumers in financial
transactions
• Oversight Reg A to Reg EE
42
Student Workbook for HS 300
_________________________G10-15_________________________
Chartering, Regulation, and
Oversight
• Depository institutions
– OTS
• Replaced Federal Home Loan Bank
Board
43
Chartering, Regulation, and
Oversight
• Credit unions
– Nonprofit depository institutions
– Member depositors own them
– National, corporate, and local tiers
– Federal or state charters
– National Credit Union Administration
regulates federally chartered credit unions
44
Chartering, Regulation, and
Oversight
• Insurance
– McCarran-Ferguson Act of 1945
 Allows states to regulate insurance
companies operating in their state
 Indirect influence on state laws
45
Student Workbook for HS 300
_________________________G10-16_________________________
State Regulation of Insurance
• Insurance companies choose where they are
incorporated based on how friendly the
state’s laws are for insurance companies
– Delaware and Arizona popular
46
State Regulation of Insurance
• Regulations focus on
– insurance premiums
– sales practices
– commissions
– profitability
– solvency regulations
– investment of reserves
47
Mutual Funds
• SEC primary regulator
• Security Act of 1933 establishes
– SEC registration parameters
– Prospectus requirements
48
Student Workbook for HS 300
_________________________G10-17_________________________
Securities Firms
• National Securities Market Improvement Act
of 1996
– Federal regulation of securities firms
– SEC head regulator in securities industry
• SROs regulate securities firms in conjunction
with SEC
– Established in the Securities Exchange Act
of 1934
– Stock exchanges are SROs
49
Pension Companies
• ERISA requires
– Pension plans to establish minimum
vesting requirement with maximum vesting
period of 10 years
– Plan transferability between employers
when changing jobs
50
Pension Companies
• ERISA requires
– Fiduciary responsibilities for plan sponsor,
trustee(s) and investment managers(s)
– Plan must meet reporting requirements
• Pension Benefit Guaranty Corporation
established
51
Student Workbook for HS 300
_________________________G10-18_________________________
Consumer Protection
52
Consumer Protection Legislation
• Emphasis on business practices, disclosure,
open access, and recourse
• Primary regulators
– FTC
– FED
– OCC
53
Consumer Protection Legislation
and Credit
• Fair Credit Reporting Act (Reg. V)
– Consumer Credit Reporting Reform Act of
1996
– Fair and Accurate Credit Transactions Act
of 2003
 Free credit reports
• Fair Credit Billing Act
54
Student Workbook for HS 300
_________________________G10-19_________________________
Consumer Protection Legislation
and Credit
• Fair Credit and Charge Card Disclosure Act
• Fair Debt Collection Practices Act
• Identity Theft and Assumption Deterrence Act
of 1998
• Credit Repair Organizations Act
55
Consumer Protection Legislation
• Truth-in-Lending (Reg. Z)
• Truth-in-Savings (Reg. DD)
• Home Ownership and Equity Protection Act
(HOEPA)
56
Consumer Protection Legislation
• Electronic Fund Transfers (Reg. E)
• Community Reinvestment Act (Reg. BB)
• Equal Credit Opportunity (Reg. B)
• Consumer Leasing Act (Reg. M)
57
Student Workbook for HS 300
_________________________G10-20_________________________
Trends in
Consumer Protection Legislation
• Privacy and protection
– Goes beyond the financial services
industry
• Challenges of Internet commerce
– Identity theft: phishing
58
Finding the Laws
59
Finding the Laws
• U.S. Code
http://www.law.cornell.edu/uscode/
• Securities Lawyer's Deskbook
– University of Cincinnati College of Law
http://www.law.uc.edu/CCL/
60
Student Workbook for HS 300
_________________________G10-21_________________________
Finding the Laws
• Regulators’ web sites
– Using the Fed as an example
 http://www.federalreserve.gov/
regulations/default.htm
• FTC site (Consumer guides are fact friendly.)
– www.ftc.org
61
The Elements of Supply and
Demand
62
Supply and Demand
• Economics is the study of unlimited wants
and scarce resources.
• The pricing of resources, at least in a
capitalistic economy, is the primary method of
allocating these scarce resources among
competing uses.
63
Student Workbook for HS 300
_________________________G10-22_________________________
Supply and Demand
• The pricing of resources, at least in a
capitalistic economy, is the primary method of
allocating these scarce resources among
competing uses.
• Price, then, is the driving force in determining
both the supply and demand for a good.
64
Supply and Demand
• Demand connotes both the willingness and
ability to buy a good at its market price.
• Supply connotes both the willingness and
ability to sell a good at its market price.
65
Laws of Supply and Demand
• The higher the price for a good, the lower the
demand for that good.
• At higher prices, more goods become
available.
• The quantity of a good will rise with a price
increase and fall with a price decrease.
• Together these are known as the laws of
supply and demand.
66
Student Workbook for HS 300
_________________________G10-23_________________________
Laws of Supply and Demand
• If the quantity demanded, QD, equals the
quantity supplied, QS, the market for the
good is in equilibrium. If supply outstrips
demand or demand outstrips supply, the
market is in disequilibrium.
67
Supply/Demand Relationship
68
Shift in the Demand Curve
69
Student Workbook for HS 300
_________________________G10-24_________________________
Two Demand Functions
70
Two Supply Functions
71
Chapter 10 Review
72
Student Workbook for HS 300
_________________________G10-25_________________________
Chapter 10 Review
• Regulations born of legislation and legislation
born of politics
• Financial services regulation can
– instill public confidence
– promote open access
– define business practices
– facilitate monetary policy
– foster innovation
73
Chapter 10 Review
• Over- or under-regulating possible
• Consumer protection legislation
– Promotes public confidence
– Establishes business practices
– Provides open access to markets
• Technology and privacy are forefront in consumer protection
legislation
• Supply and demand curves
– Movement along the curve, disequilibrium
– Shifting curves
– Elasticity demand curve
– Inelastic demand curve
74
Student Workbook for HS 300
Financial Planning for Special
Circumstances
11
Overview
Chapter 11 focuses on unique planning circumstances. This chapter is a compilation
of concepts written by different authors at The American College. The material in
Chapter 11 consists of concepts that are on the current CFP® exam, but were not on
the exam before 2004. These concepts include divorce planning, same sex marriage,
nontraditional living arrangements, unique estate planning situations and special needs
trusts.
This chapter stresses the importance of developing a wide body of knowledge and
networking with a capable and competent financial planning team. Financial planning
solutions are rarely yes or no answers. Complex situations are found in any family
situation, and as clients or parents of clients age they may find themselves in unique
living situations. This chapter address estate planning for elderly clients. With an aging
population, developing skill sets to work with elderly clients is just as important today
as ever before. This class covers a wide range of topics, and students are encouraged to
supplement the concepts covered in Chapter 11 with additional course work and readings.
Preparing for Class 11
√ Read chapter 11 in Financial Planning: Process and Environment AT LEAST
TWICE.
√ Answer the review questions and self-test questions at the end of the chapter.
√ Study the key terms and concepts listed in the textbook.
√ Review the Study Tips for Class 11 in the student workbook, and the video
graphics.
√ Participate in the online community for this course by accessing AC Online
at <http://blackboard.theamericancollege.edu>. Explore the channels for this
course on AC Online including: Course Info, Modules, Exams, Forums, and
Tools. Stop by the Student Lounge, on the Forum channel to post a note to other
students in the course, or Office Hours to post a note for the course instructor.
11-2
Study Tips for Class 11
•
•
•
•
•
•
•
Understand the tax implications of divorce
Learn the tax status of alimony and child support payments
Be able to discuss planning for nontraditional couples and families
Understand income tax benefits for parents of children with special needs
Learn the unique needs of estate planning for the elderly and incapacitated
Discuss techniques used in estate planning for young married couples and
singles
Be able to discuss estate planning strategies for nontraditional couples
Post-Class: Test Your Comprehension
√ Complete the Quick Quiz for class 11.
√ Complete the Key Terms and Concepts matching exercise for class 11.
Student Workbook for HS 300
11-3
Quick Quiz
Please indicate which of the following statements are true.
T F
1. Younger couples are responsible for more divorce than their older
counterparts.
T F
2. Same sex couples can file joint federal tax returns.
T F
3. Child support payments are income tax-free to the recipient.
T F
4. Parents can deduct special school expenses for children who do not
live at home as an itemized deduction.
T F
5. Parents may only take the tax credit for adoption expenses once
every three calendar years.
T F
6. A taxpayer whose spouse who is physically or mentally incapable
of caring for himself or herself can take the child and dependent
care credit.
T F
7. Settlement Special Needs Trusts are established by nonfamily or
interested parties on behalf of an incapacitated person.
T F
8. Estate planning for married couples requires listening to the goals
of both spouses.
T F
9. Titling assets as Joint Tenants with Right of Survivorship is an
effective method of transferring property at death for same sex
couples.
T F
10. Titling assets as Sole and Separate property is an effective method
of transferring property at death for same sex couples.
Student Workbook for HS 300
11-4
Key Terms and Concepts
Match the term below with the correct definition.
a. alimony
__________ 1. property management devices used to protect the
assets of individuals who anticipate long-term
disability situations in the future
b. child tax credit
__________ 2. payments made between ex-spouses that are
taxable to the recipient and deductible to the
payor
c. conservator
__________ 3. a tax credit used to encourage economically
disadvantaged individuals to work by partially
offsetting Social Security taxes
d. dependent tax care credit
__________ 4. a tax credit used to help care for a qualifying
relative or member of a taxpayer’s household
e. earned income tax credit
__________ 5. a tax credit available for children of special needs
children
f. Grantor Retained Annuity
Trust
__________ 6. a court appointed guardian or conservator to act
on behalf of a disabled individual
g. guardian
__________ 7. a term referring specifically to care for a person
h. special-needs trust
__________ 8. a term referring specially for the management of
property
i. standby trust
__________ 9. a type of trust used in planning for incompetency
j. ward
__________ 10. a type of trust used to transfer property for
individuals in same sex relationships
Student Workbook for HS 300
11-5
Answers to Key Terms and Concepts Matching
1.
2.
3.
4.
5.
6.
7.
8.
9.
10.
h.
a.
e.
d.
b.
j.
g.
c.
i.
f.
Student Workbook for HS 300
_________________________G11-1_________________________
HS 300 Financial Planning:
Process and Environment
Chapter 11:
Financial Planning for
Special Situations
1
Learning Objectives
2
Learning Objectives
1. Explain the tax implications for divorcing couples.
2. Describe the opportunities, challenges, and
strategies for nontraditional families and couples.
3. Understand and capitalize on income tax benefits
for parents of children and children with special
needs.
4. Explain how the dependent-care and earned income
tax credits impact economically disadvantaged
individuals.
3
Student Workbook for HS 300
_________________________G11-2_________________________
Learning Objectives
5. Explain how estate planning can be tailored
to meet the special needs of elderly and
incapacitated individuals.
6. Describe some of the basic issues that
pertain to estate planning for married couples
and single individuals.
7. Explain various estate planning
arrangements for individuals in nontraditional
living arrangements.
4
Discussion Break
What are the financial considerations of
Divorce?
– Immediate?
– 6 Months?
– Long Term?
5
Discussion Break
• Immediate Needs:
– Housing
– Security
– Liquidity
• 6 Months:
– Asset Distribution
– Long Term Housing
– Children?
• Long Term
– Social Security
– Medicare
– Retirement
6
Student Workbook for HS 300
_________________________G11-3_________________________
Financial Considerations of Divorce
7
Divorce Considerations
• Divorce and annulments have steadily
declined over the past seven years from
4.0% of the population in 2000 to 3.6% in
2007
• Young couples divorce more frequently than
older couples
• Over 50% of marriages will end in divorce or
annulment
8
Divorce Considerations
• Social Security
– If the marriage lasts 10 years or longer,
divorced spouses are entitled to their ex
spouses benefit record
– If spouse re-marries they may loose
access to benefits
9
Student Workbook for HS 300
_________________________G11-4_________________________
Payments in connection with Annulment
or Divorce
• Alimony
– Tax Deductible (to payer)
– Taxable Income (to recipient)
– Set by Court Order
– Beware Front Loading
10
Front Loading of Alimony
In the payer’s first postseparation tax year, the
"excess" payment is computed by taking the
average of the alimony payments in the
second and third postseparation years and
adding $15,000 to that figure.
• Any alimony payment for the first
postseparation year that is in excess of that
amount is the "excess
• alimony payment."
11
Front Loading Example
• Bill and Melinda divorce in 2005. Bill pays
Melinda
– $100,000 in 2005
– $40,000 in 2006
– $20,000 in 2007
• Bill will recapture excess alimony of $55,000
($100,000 - $30,000 - $15,000)
12
Student Workbook for HS 300
_________________________G11-5_________________________
Child Support Payments
• Court Ordered
• Non Deductible
• Legal Obligation with Severe Penalties
13
Financial Planning for Unmarried
Couples
• No Federal Tax Benefits
– Filing Status
– Gift and Estate Exemptions
– Intestate Considerations
• Same Sex Couples
– 5 States Recognize Same Sex Marriage
– Others may have Civil Union Benefits
– Federal Govt. Does not
14
Know the Rules
• State Specific
– Recognize other State’s Marriages
– Some may have civil unions
– Team up with local attorneys for estate
planning
15
Student Workbook for HS 300
_________________________G11-6_________________________
Tax Planning for Children with Special
Needs
• Tax Deductions
• Tax Credits
16
Medical Expenses
• Medical expenses for children of divorced
parents are deductible by the parent who
pays for them, regardless of who has the
dependency exemption
• Eligible expenses deductible only to the
extent they exceed 7 ½% of AGI (10% of
AGI for the AMT).
17
Eligible Medical Expenses
Deductible medical expenses include any
amount paid for
– Diagnosis, cure, mitigation, treatment or
prevention of disease, or for the purpose of
affecting any structure or function of the
body
– Transportation for and essential to the
above
– Insurance which covers the above
18
Student Workbook for HS 300
_________________________G11-7_________________________
Eligible Medical Expenses
No deductions for
– Illegal operations or treatment
– Cosmetic surgery except reconstructive
surgery for deformities, and disfigurements
arising from trauma or disease
– Trips to improve general health
– Nonprescribed OTC medicines/drugs
19
Nonrefundable Credits
Credit for Adoption Expenses
• Qualified expenses include: legal fees, court
costs, and other related costs with regard to
legal adoption
20
Nonrefundable Credits
Credit for Adoption Expenses
• The limit is per child, not per year
(c/o of 5 years)
• Credit phaseout begins for TPs with AGIs
exceeding approximately $170,000
21
Student Workbook for HS 300
_________________________G11-8_________________________
Nonrefundable Credits
• Child and Dependent Care Credit
– Purpose: the law expands the credit to
relieve burden of two-earner families who
must pay for dependent care
22
Nonrefundable Credits
• Child and Dependent Care Credit
– AGI reduction point is $15,000.
– When AGI exceeds $43,000, a 20%
maximum credit is available.
23
Nonrefundable Credits
• Child and Dependent Care Credit
– Amount of the Credit: 35% (reduced to
20% as AGI increases) of employmentrelated expenses
• Credit limitations = Lesser of
–Earned income
–$3,000 for one individual, $6,000 for
two or more individuals
24
Student Workbook for HS 300
_________________________G11-9_________________________
Estate Planning for Unique Situations
• Elderly and Incapacitated
– Special Needs Trusts
– Elder Law and Durable Power of Attorney
– Guardianship and Conservatorship
– Considerations of Medical Assistance
Programs
25
Special Needs Trusts
• Family SNT—the most widely used SNT;
frequently used as an estate planning tool for
incapacitated persons
• Third-party SNT—established by nonfamily or
interested parties on behalf of an
incapacitated person
26
Special Needs Trusts
• Settlement SNT—funded with personal injury
lawsuit damages received by the
incapacitated individual from a court decision
• Pooled SNT—funded with assets (including
the disabled individual’s personal assets) that
are pooled with numerous other sources to
increase investment results. States have
unique agencies to help establish funds.
27
Student Workbook for HS 300
_________________________G11-10_________________________
Durable Power of Attorney
• Appoint an attorney in fact
• “Spring” into action at incapacity or disability
• Health care and Financial POA
– Manage Assets
– Accelerate Medicaid
– Representation
28
Guardianship & Conservatorship
• In the absence of a durable power of attorney, a
court proceeding may be held to establish
incompetency to manage one’s affairs.
• After hearing evidence of incompetency, a court
may appoint a guardian or conservator to act on
behalf of the disabled party, called a ward.
• The process is generally an unpleasant one that, in
the end, results in substantial loss of a ward’s
liberties.
29
Guardianship & Conservatorship
• In some cases, the term guardian pertains
specifically to care of the person
• and the term conservator to management of
the property.
30
Student Workbook for HS 300
_________________________G11-11_________________________
Estate Planning in Unique Situations
• Nontraditional Living Arrangements
– Beware of Intestate
– Revocable Trust Planning
– Testamentary Instruments
• Wills
• Health Care Powers
• Coordination of Beneficiaries
31
Estate Planning in Unique Situations
• Divorcing Couples
– Update Testamentary Documents
– Consider the Needs of Children and
Dependents
– Independent Representation
– Beneficiary Designations
32
Student Workbook for HS 300
Review
12
Review of material in:
Financial Planning: Process and Environment, 3d Edition
Craig Lemoine and Don A. Taylor Editors
Chapter 1—The Financial Planning Process
Chapter 2—Communicating Effectively with Clients
Chapter 3—Ethics, Professionalism, and Practice Standards
Chapter 4—Client Attitudes Toward Risk
Chapter 5—Gathering Data and Preparing Financial Statements
Chapter 6—Time Value of Money: Basic Concepts and Applications
Chapter 7—Time Value of Money: Advanced Concepts and Applications
Chapter 8—Financial Planning Applications
Chapter 9—The Regulation of Financial Advisors
Chapter 10—The Legal and Economic Environment of Financial Institutions
Chapter 11—Financial Planning for Special Circumstances
Preparing for Review Class
√ Review all materials.
√ Take Practice Exam and the Sample Exam.
√ Use the results of the exams to point out troublesome areas and restudy those
areas.
12-2
√ Participate in the online community for this course by accessing AC Online
at <http://blackboard.theamericancollege.edu>. Explore the channels for this
course on AC Online including: Course Info, Modules, Exams, Forums, and
Tools. Stop by the Student Lounge, on the Forum channel to post a note to other
students in the course, or Office Hours to post a note for the course instructor.
Student Workbook for HS 300
HS 300 Practice Examination
PE
Note
Following are 25 objective questions to be used as an additional review and to
familiarize you with the format of The American College objective examinations.
These questions are followed by an Answer Key and an explanation of the
answers.
DIRECTIONS
Each of the questions or incomplete statements below on the sample examination is
followed by four suggested answers or completions. Select the one that is best in each
case and circle the letter next to it.
1.
2.
3.
Which of the following statements concerning the relationship between
law and ethics is correct?
(A) All law is included within ethics.
(B) All ethics are included within law.
(C) Ethics and law are equivalent.
(D) Ethics and law overlap.
Which of the following limitations on rational thinking is influencing a
person who overestimates the risks of bungee jumping because of the high
degree of publicity given to a recent bungee jumping accident?
(A) denial of risk bias
(B) availability bias
(C) hindsight bias
(D) confirmation bias
Asking a question without waiting for the answer to a previous question is
an example of
(A)
(B)
(C)
(D)
question bombardment
an open-ended question
a "why" question
a leading question
PE-2
4.
5.
Assume that $1,000 will be deposited today in an account that will earn 8
percent compound interest per year. How much will be in the account at
the end of 5 years, rounded to the nearest dollar?
(A) $1,217
(B) $1,218
(C) $1,469
(D) $1,477
Assume that $1000 will be received today and at the end of each of the
next 4 years. On the basis of a 7 percent interest assumption, what is this
series of payments worth today, rounded to the nearest dollar?
6.
(A) $4,100
(B) $4,387
(C) $4,515
(D) $4,748
Assume that $1,000 is deposited into a savings account today and that
$500 is deposited into the same account at the end of each of the next 3
years. On the basis of a 7 percent interest assumption, how much will be
in the account at the end of the 3 years, rounded to the nearest dollar, if no
withdrawals are made?
7.
(A) $2,832
(B) $2,662
(C) $2,580
(D) $4,771
Bob is evaluating an investment project that requires an outlay of $6,000
today and $6,000 in one year. The project will generate inflows of $8,000
at the end of years 2, 3, and 4, and outflow of $6,000 at the end of year 5,
and inflows of $4,000 per year at the end of years 6, 7, and 8. Which of
the following is the net present value, rounded to the nearest dollar, of the
project if the required rate of return on this type of project is 10 percent?
8.
(A) $27,443
(B) $16,534
(C) $10,000
(D) $ 9,083
Mary is evaluating a capital budgeting project. The project requires an
immediate investment of $20,000. No other cash flows will be generated
until the end of the third year. At that time, a cash inflow of $4,000 will
occur to be followed by $4,000 per year for the next 7 years. What is the
internal rate of return on the project?
(A)
(B)
(C)
(D)
7.74 %
9.33 %
11.81 %
cannot be determined from the information provided.
Student Workbook for HS 300
PE-3
9.
Which of the following statements concerning communication with clients
is (are) correct?
I. Paraphrasing the client’s statements impedes communication because
it implies that the client is incapable of expressing himself or herself
clearly.
II. Responding to clients with brief responses, such as "uh-huh," tends
to cause the client to think the adviser is not really paying attention.
10.
(A) I only
(B) II only
(C) Both I and II
(D) Neither I nor II
Federal securities laws and/or regulations prohibit a registered investment
adviser from doing which of the following?
I. using part I of Form ADV to satisfy the brochure rule
II. using part II of Form ADV to satisfy the brochure rule
11.
(A) I only
(B) II only
(C) Both I and II
(D) Neither I nor II
Forms of nonverbal communication include which of the following?
I. voice tone
II. word selection
12.
(A) I only
(B) II only
(C) Both I and II
(D) Neither I nor II
Which of the following is (are) among the areas that should be addressed
in a comprehensive financial plan?
I. federal income tax analysis
II. cash-flow management
(A)
(B)
(C)
(D)
I only
II only
Both I and II
Neither I nor II
Student Workbook for HS 300
PE-4
13.
Which of the following statements concerning the compounding of interest
rates is (are) correct?
I. Increasing the frequency of compounding will increase the future
value of a single sum.
II. Decreasing the frequency of compounding will decrease the present
value of a single sum.
14.
(A) I only
(B) II only
(C) Both I and II
(D) Neither I nor II
Which of the following statements concerning limitations in rational
thinking and behavior is (are) correct?
I. People often settle for a course of action that is less than optimal.
II. Most people fully understand how they make risky decisions.
15.
(A) I only
(B) II only
(C) Both I and II
(D) Neither I nor II
Which of the following questions is (are) open-ended?
I. How many years will you work before you retire?
II. Do you want to start a retirement plan?
16.
(A) I only
(B) II only
(C) Both I and II
(D) Neither I nor II
Which of the following factors typically contribute(s) to an increase in
anticipatory regret associated with a decision?
I. The decision is viewed as an important one.
II. The differences among the choices are readily apparent.
17.
(A) I only
(B) II only
(C) Both I and II
(D) Neither I nor II
For the same interest rate and number of payments, which of the following
time-value-of-money relationships is (are) correct?
I. The FVA factor is larger than the FVAD factor.
II. The FVAD factor is larger than the PVAD factor.
(A)
(B)
(C)
(D)
I only
II only
Both I and II
Neither I nor II
Student Workbook for HS 300
PE-5
READ THE
CONTINUING
FOLLOWING
DIRECTIONS
BEFORE
The questions below differ from the preceding questions in that they all
contain the word EXCEPT. So you understand fully the basis used in
selecting each answer, be sure to read each question carefully.
18.
19.
20.
21.
22.
Tests used by the Securities and Exchange Commission to determine
whether a financial planner is subject to the terms and provisions of the
Investment Advisers Act of 1940 include all the following EXCEPT
(A) Does the individual provide a brochure to prospective clients?
(B) Does the individual give advice or analyses concerning securities?
(C) Does the individual present himself or herself to the public as being
in the business of providing advice about securities?
(D) Does the individual receive compensation for services?
All the following are among the 6 steps included in the general model of
the financial planning process EXCEPT
(A) implementing the plan
(B) analyzing the client’s future position
(C) obtaining the client’s approval
(D) reviewing and revising the plan
All the following are considered discretionary expenses in budgeting
EXCEPT
(A) household supplies and maintenance
(B) savings
(C) education fund
(D) gifts and contributions
Calculating the internal rate of return requires all the following information
EXCEPT
(A) cash outflows
(B) cash inflows
(C) the number of time periods
(D) the relevant interest rate
The CFP® Board has passed and made effective at least one practice
standard for each of the following aspects of the financial planning process
EXCEPT
(A)
(B)
(C)
(D)
establishing and defining the relationship with the client
filing tax and other documents with authorities
implementing the plan
monitoring the plan
Student Workbook for HS 300
PE-6
23.
All the following steps to improve ethical decision making in organizations
are correct EXCEPT
(A)
(B)
(C)
(D)
24.
25.
communicate clear ethical goals and standards
create positive reinforcement and rewards for ethical behavior
recognize and remove barriers to ethical decisions
specify punishment that employees will receive for certain unethical
behaviors.
All the following describe the major types of life situations involving risk
EXCEPT
(A) capital
(B) pessimism
(C) freedom
(D) life
All the following are characteristics of typical risk averters EXCEPT
(A)
(B)
(C)
(D)
They have success as a goal.
They like structure.
They prefer low variability.
They overestimate risk.
Student Workbook for HS 300
PE-7
Answer Key for Practice Examination
Question Number
Correct Answer
Chapter Number
1
D
3
2
B
4
3
A
2
4
C
6
5
B
6
6
A
6
7
D
7
8
A
7
9
D
2
10
A
9
11
A
2
12
C
1
13
A
6
14
A
4
15
D
2
16
A
4
17
B
6
18
A
9
19
B
1
20
A
5
21
D
7
22
B
3
23
D
3
24
B
4
25
A
4
Student Workbook for HS 300
PE-8
Explanation of Answers to Practice Exam Questions
1. The answer is (D).
There is considerable overlap between law and ethics, but there are ethical
standards that are not codified into law and laws that may be considered beyond
the realm of ethics.
2. The answer is (B).
Availability bias refers to the fact that dramatic and vivid events are easily
available to one’s mind and, therefore, tend to be overestimated.
3. The answer is (A).
(B) is incorrect because it refers to a type of question that allows the respondent
to elaborate.
(C) is incorrect because it refers to questions that tend to imply the questioner’s
disapproval.
(D) is incorrect because it refers to a type of question that directs the client to
a conclusion that may be more reflective of the questioner’s beliefs than the
respondent’s.
4. The answer is (C).
The problem is structured as a future value of a single sum. Keystrokes are as
follows: HP-10BII:
, C ALL, 1000, +/-, PV, 8, I/YR, 5, N, FV
5.
The answer is (B).
The problem is structured as a present value of a 5-period annuity due. The cash
flow is structured as follows:
Keystrokes are as follows: HP-10BII:
, C ALL, (if BEGIN is not displayed),
, BEG/END, 1000, PMT, 5, N, 7, I/YR, PV
6. The answer is (A).
The cash flow is structured as follows:
Keystrokes are as follows:
HP-10BII:
, C ALL, (if END is not displayed), , BEG/END, 1000, +/-, PV,
500, +/-, PMT, 3, N, 7, I/YR, FV
7. The answer is (D).
The cash flow structure is as follows, in thousands of dollars:
Student Workbook for HS 300
PE-9
Keystrokes are as follows:
HP-10BII: , C ALL, 6000, +/-, CFj, 6000, +/-, CFj, 8000, CFj, 3,
+/-, CFj, 4000, CFj, 3, , Nj, 10, I/YR,
, NPV
8. The answer is (A).
The cash flow structure is as follows, in thousands of dollars
9.
10.
11.
12.
13.
14.
15.
16.
17.
, Nj, 6000,
Keystrokes are as follows:
HP-10BII:
, C ALL, 20000, +/-, CFj, 0, CFj, 0, CFj, 4000, CFj, 8,
, Nj,
, IRR/YR
The answer is (D).
I is incorrect because paraphrasing communicates the desire to understand and
helps clarify the client’s thoughts.
II is incorrect because so-called "minimal encouragers" communicate to the
speaker, "Go on, I’m with you."
The answer is (A).
II is incorrect because the SEC specifically allows use of part II of Form ADV to
satisfy the brochure rule.
The answer is (A).
II is incorrect because it relates to verbal communication.
The answer is (C).
Both I and II are correct. Federal income tax analysis and a cash-flow
management statement are both areas that should be addressed in a
comprehensive financial plan.
The answer is (A).
II is incorrect because decreasing the frequency of compounding will increase
the present value of a single sum.
The answer is (A).
II is incorrect because few people actually have an insight into how they make
risky decisions.
The answer is (D).
I and II are incorrect because both are close-ended questions that give the
respondent little opportunity to elaborate on his or her opinions, thoughts, ideas,
values, or feelings.
The answer is (A).
I is correct because anticipatory regret is likely to increase when the decision is
viewed as important.
II is incorrect because anticipatory regret increases when the differences among
the choices are not very apparent.
The answer is (B).
Student Workbook for HS 300
PE-10
18.
19.
20.
21.
22.
23.
24.
25.
I is incorrect because FVAD factors are larger than FVA factors with the same rate
and number of payments. The FVAD payments are received one period sooner
than FVA payments, allowing for one more period of compounding and therefore
a larger future value.
The answer is (A).
The requirement to register as an investment adviser is independent of the
individual’s decision or licensing to sell financial products. If the questions in
(B), (C), and (D) are answered affirmatively, the financial planner must register
regardless of whether he or she sells or has the legal credentials to sell financial
products.
The answer is (B).
Analyzing the client’s future financial position may be a part of a financial plan,
but it is not specified as one of the 6 steps in the financial planning process.
The answer is (A).
Household supplies and maintenance is considered a fixed expense.
The answer is (D).
It is not necessary to know any interest rate to calculate the internal rate of return.
The internal rate of return can be considered the discount rate, which causes the
present value of the inflows to equal the present value of the outflows.
The answer is (B).
None of the 10 practice standards specifically addresses filing tax and other
documents with authorities.
The answer is (D).
Specifying punishment that employees will receive for certain unethical
behaviors is not a step to improve ethical decision making in organizations. It is
a negative approach that represents a "stick" rather than a "carrot."
The answer is (B).
Pessimism is not one of the major life situations involving risk taking. The major
types of life situations are monetary (loss of capital), physical (loss of life), ethical
(loss of freedom) and the fourth is social (loss of face).
The answer is (A).
Risk averters are more likely to have happiness as a goal, rather than success. In
contrast to the monetary risk averters, who stated that their long-term goal was
happiness, the monetary risk takers listed success as their primary goal.
Student Workbook for HS 300
HS 300 SAMPLE EXAMINATION
NOTE
Below are 100 objective questions to be used as an additional review and to familiarize
you with the format of American College objective examinations. These questions are
followed by an Answer Key and an explanation of the answers.
DIRECTIONS
Each of the questions or incomplete statements below is followed by four suggested
answers or completions. Select the one that is best in each case and circle the letter that
corresponding to that answer.
1.
2.
3.
4.
Which of the following steps in the financial planning process is where
the advisor identifies the strengths and weaknesses in the client’s present
financial condition?
(A) Establish and define the advisor-client relationship.
(B) Determine goals and gather data.
(C) Develop and present a plan.
(D) Analyze and evaluate the data.
Which of the following types of questions is likely to be the most effective
in client counseling?
(A) open-ended
(B) close-ended
(C) leading
(D) either/or
Which of the following statements concerning ethics in business is correct?
(A) It is good public relations for a business.
(B) It is about creating a good image for a business.
(C) It is a luxury to indulge in after meeting the bottom line.
(D) It is about how people conduct business every day.
If $100 is deposited today in an account paying 9 percent compound annual
interest, how much will be in the account at the end of 4 years?
(A)
(B)
(C)
(D)
$70.84
$141.16
$323.97
$457.31
2
5.
6.
7.
8.
9.
10.
If Bob deposits $8,000 today in an account that pays 3 percent compound
annual interest, how long will it take for the account to reach $10,000?
(A) 3.77 years
(B) 4.58 years
(C) 6.12 years
(D) 7.55 years
For SEC registration purposes, which of the following acts sets the limit
for assets under management?
(A) Sarbanes-Oxley Act
(B) Investment Advisers Supervision Coordination Act
(C) Investment Advisers Act
(D) Securities Exchange Act
Which of the following is an appropriate investment for an emergency
fund?
(A) a money market mutual fund
(B) bonds
(C) real estate
(D) stocks
Assume that payments of $1,000 are to be received at the end of each of
the next 5 years. On the basis of a 9 percent compound annual interest
assumption, what is the series of payments worth today?
(A) $649.93
(B) $1,538.58
(C) $3,889.65
(D) $5,984.71
How much money do you need to deposit today in a savings account
earning 9 percent compound annual interest if your goal is to accumulate
$10,000 at the end of 4 years?
(A) $3,086.71
(B) $7,084.25
(C) $7,721.83
(D) $14,115.82
Jim needs to accumulate $25,000 in a savings account over the next 5
years. He can make five annual deposits of $4,000 starting today. What
compound annual rate of return must the account earn in order for Jim to
meet his goal?
(A)
(B)
(C)
(D)
5.00%
6.12%
6.67%
7.53%
Student Workbook for HS 300
3
11.
12.
13.
14.
15.
16.
Bill expects to receive $10,000 each year for 4 years beginning one year
from today. If Bill can earn 6 percent compound annual interest on these
funds, what is the present value of this series of payments?
(A) $34,651.06
(B) $36,298.95
(C) $49,173.24
(D) $52,421.37
If $1,000 is deposited into a savings account at the beginning of each of
the next 4 years starting today, how much money will be in the account 4
years from now if no withdrawals are made and the account earns 9 percent
compound annual interest?
(A) $3,239.72
(B) $3,531.29
(C) $4,573.13
(D) $4,984.71
What is the yield to maturity of a $1,000 face amount bond that currently
sells for $875, pays $60 of interest at the end of each year, and matures in
10 years?
(A) 7.85%
(B) 8.75%
(C) 9.35%
(D) 10.00%
According to a 2004 consumer survey conducted by the CFPŠ Board of
Standards, the top reason why people begin financial planning is to
(A) accumulate capital
(B) purchase or renovate a home
(C) build a retirement fund
(D) generate current income
Which of the following steps in the financial planning process is where
the advisor reviews changes in the client’s circumstances and the financial
environment?
(A) Implement the plan.
(B) Monitor the plan.
(C) Establish and define the advisor-client relationship.
(D) Determine goals and gather data.
Which of the following is considered the most direct approach to
measuring a client’s financial risk tolerance?
(A)
(B)
(C)
(D)
Determine the client’s attitudes toward risk.
Determine the client’s preferences for various investment products.
Determine the client’s real-life choices involving risk.
Determine the client’s investment objectives.
Student Workbook for HS 300
4
17.
The liquidity ratio is
(A)
(B)
(C)
(D)
18.
19.
20.
21.
total debt payments divided by gross income
net worth divided by total assets
liquid assets divided by total current debts
net cash flow plus savings and investments divided by annual after-tax
income
Pam and Kevin were divorced in 2005. Pam paid Kevin $100,000 of
alimony in 2006, $50,000 in 2007, and $10,000 in 2008. Pam took an
income tax deduction for the alimony paid every year. How much income
must Pam include on her tax return for front-loading alimony payments in
2009?
(A) $0
(B) $45,000
(C) $55,000
(D) $90,000
Bob wants to accumulate $100,000 in his retirement fund over the next 5
years. He plans to make five equal annual payments into the fund starting
today. If the fund can earn 8 percent compound annual interest, how large
must each annual payment be?
(A) $10,472.18
(B) $15,783.01
(C) $17,045.65
(D) $68,058.32
Jill plans to make 7 annual payments of $1,000 each to a savings account.
Her plan calls for the first payment to be made one year from today. How
much money will be in Jill’s account at the end of 7 years if the account
earns 4 percent compound annual interest?
(A) $4,246.46
(B) $4,439.94
(C) $7,898.29
(D) $8,654.02
Which of the following is the effective annual interest rate for a 5 percent
nominal rate that is compounded daily?
(A)
(B)
(C)
(D)
5.0945%
5.1162%
5.1246%
5.1267%
Student Workbook for HS 300
5
22.
Using a discount rate of 6 percent, what is the net present value of an
investment with the following cash flow structure?
Timing of Cash Flow
Amount of Cash Flow
Beginning of year 1
$2,000 cash outflow
End of year 1
3,000 cash inflow
End of year 2
1,000 cash outflow
End of years 3 and 4
23.
24.
0 cash flow
End of year 5
3,000 cash inflow
End of year 6
3,000 cash inflow
(A) $4,296.85
(B) $6,076.84
(C) $8,296.85
(D) $10,076.84
What is the present value (rounded to the nearest dollar) of $12,000 due
in 6 years discounted semiannually at a 5 percent nominal annual rate of
interest?
(A) $8,891
(B) $8,895
(C) $8,906
(D) $8,923
What is the present value of the following series of payments based on a 5
percent compound annual interest rate?
(A)
(B)
(C)
(D)
Timing of Payment
Amount of Payment
End of year 1
$1,000
End of year 2
3,000
End of year 3
2,000
End of year 4
0
End of year 5
2,000
End of year 6
2,000
$6,968.20
$8,460.63
$9,044.28
$10,000.00
Student Workbook for HS 300
6
25.
At the end of year 5, what is the future value of the following series
of deposits to a savings account that earns 6 percent compound annual
interest?
Timing of Deposit
End of year 1
End of year 2
End of year 3
End of year 4
End of year 5
26.
27.
28.
Amount of Deposit
$1,000
1,000
0
2,000
2,000
(A) $4,912.10
(B) $5,096.82
(C) $5,206.82
(D) $6,573.49
A comprehensive financial plan organized to follow the financial planning
process should start by
(A) describing the client’s present situation based on both the personal
and financial data gathered from the client
(B) specifying the client’s stated goals indicating the priority of each one
and the time frame for achieving it
(C) specifying the responsibilities of each party for implementing the plan
and carrying it through to completion
(D) identifying problems that the client would encounter in attempting to
accomplish stated goals
Which of the following statements concerning the relationship between
education and financial risk tolerance is correct?
(A) Risk tolerance increases with the degree of formal education.
(B) The relationship between the degree of formal education and risk
tolerance increases through high school and then decreases.
(C) Risk tolerance decreases with the degree of formal education.
(D) There is no recognized relationship between the degree of formal
education and risk tolerance.
According to the 2001 Federal Reserve Board’s Survey of Consumer
Finances, borrowing for which of the following purposes accounts for the
largest percent of total family debt?
(A)
(B)
(C)
(D)
education
home purchase
investments
vehicle purchase
Student Workbook for HS 300
7
29.
30.
31.
32.
33.
Which of the following statements concerning types of interviews is
correct?
(A) A directive interview usually takes more time than a nondirective
interview.
(B) The advisor usually controls the pace and content of a nondirective
interview.
(C) A nondirective interview tends to be more flexible than a directive
interview.
(D) The client usually controls the pace and content of a directive
interview.
Step 2 (determining goals and gathering data) of the financial planning
process is related to which of the following Practice Standards?
(A) defining the scope of the engagement
(B) defining the monitoring responsibilities
(C) presenting the financial planning recommendations
(D) obtaining quantitative information and documents
Nancy is being offered a series of 6 annual payments in exchange for her
ownership in a widget factory. The first payment of $200,000 will be
payable immediately. Future payments will grow at a rate of 20 percent
per year. If Nancy can earn 9 percent on her investments, what would the
lump-sum equivalent of the offer be worth today?
(A) $868,715.27
(B) $956,383.78
(C) $1,546,699.49
(D) $1,702,788.43
What is the internal rate of return of a project with the following cash
flows?
Timing of Cash Flow
Amount of Cash Flow
Immediate
$10,000
cash outflow
End of year 1
5,000
cash outflow
End of year 2
0
cash flow
End of year 3
10,000
cash inflow
End of year 4
20,000
cash inflow
(A) 17.61%
(B) 19.32%
(C) 21.54%
(D) 23.12%
To what amount (rounded to the nearest dollar) will $8,000 grow in 3 years
in a savings account that earns 8 percent interest compounded weekly?
(A)
(B)
(C)
(D)
$10,078
$10,123
$10,146
$10,168
Student Workbook for HS 300
8
34.
35.
36.
37.
38.
How much should an investor be willing to pay for a bond that pays $25 of
interest every 6 months for the next 3 years and the $1,000 face amount at
the end of 3 years if the investor requires a yield to maturity of 8 percent?
(A) $858.23
(B) $921.37
(C) $1,000.00
(D) $1,055.90
What is the correct definition of a pooled special needs trust?
(A) the most widely used SNT; frequently used as an estate planning tool
for incapacitated persons
(B) established by nonfamily or interested parties on behalf of an
incapacitated person
(C) funded with personal injury lawsuit damages received by the
incapacitated individual from a court decision
(D) funded with assets that are commingled with numerous other sources
to increase investment results. Each state has a nonprofit organization
established to achieve investment benefits.
Which of the following financial advisors using the financial planning
process is practicing multiple-purpose financial planning?
(A) a multiline insurance agent who sells life, health, and property and
liability insurance to a client
(B) a personal finance counselor who shows a client how to set up and
live within a budget
(C) a stockbroker who advises a customer to buy shares of common stock
in the "XYZ" company
(D) a banker who opens a trust account for the benefit of a customer’s
handicapped child
Which of the following statements concerning the financial planning
pyramid is correct?
(A) As the client moves up the pyramid, the focus of the plan shifts from
wealth accumulation goals to income protection needs.
(B) The middle part of the pyramid is built with emergency savings,
insurance coverages, and a properly drawn will.
(C) The pyramid is a widely accepted approach for developing a
comprehensive financial plan over a period of time.
(D) The foundation of the pyramid addresses both the management of
retirement assets and the conservation/distribution of the estate.
A person who overestimates the risk of flying because of the high degree
of publicity given to a recent airplane crash is being influenced by which
of the following limitations on rational thinking?
(A)
(B)
(C)
(D)
denial of risk bias
availability bias
hindsight bias
confirmation bias
Student Workbook for HS 300
9
39.
40.
41.
Which of the following is the appropriate standard of conduct for a
fiduciary?
(A) acting in the client’s best interest
(B) acting as a prudent manZRPDQ
(C) acting on a best efforts basis
(D) acting on self-interest
Which of the following statements best describes cash flow planning?
(A) It involves identifying courses of action that will help optimize net
cash flow.
(B) It is the process of gathering data about an individual’s cash flow
situation.
(C) It is the difference between an individual’s income and expenses.
(D) It is the difference between an individual’s total assets and total
liabilities.
Which of the following statements concerning leading responses by the
advisor is (are) correct?
I. The explanatory response is designed to make the client feel better in
a time of need.
II. The interpretive response is designed to discourage self-interpretation
by the client.
42.
(A) I only
(B) II only
(C) Both I and II
(D) Neither I nor II
Which of the following activities can the attorney in fact of a durable power
of attorney engage in?
I. expedite filing for Medicare on behalf of the disabled individual
II. borrow against the cash value of a life insurance policy owned by the
disabled individual
43.
(A) I only
(B) II only
(C) Both I and II
(D) Neither I nor II
Most ethics codes applicable to financial advisors share which of the
following common themes and sentiments?
I. They require advisors to continue the learning process throughout
their careers.
II. They call on advisors to look out for their own best interests.
(A)
(B)
(C)
(D)
I only
II only
Both I and II
Neither I nor II
Student Workbook for HS
10
44.
Which of the following lifestyle characteristics would seem to indicate that
a client is disposed toward financial risk tolerance?
I. The client has been in the same job with the same company for 25
years.
II. The client’s annual income has varied substantially from year to year.
45.
(A) I only
(B) II only
(C) Both I and II
(D) Neither I nor II
Other things being equal, a client’s net worth will increase as a result of
which of the following actions?
I. the addition to assets through retaining income
II. a decrease in liabilities through forgiveness
46.
(A) I only
(B) II only
(C) Both I and II
(D) Neither I nor II
Which of the following federal student aid programs is (are) need based?
I. Parent Loans for Undergraduate Students
II. Federal Perkins loan
47.
(A) I only
(B) II only
(C) Both I and II
(D) Neither I nor II
Which of the following questions must be answered in the affirmative
before the value of an account can be counted toward meeting the $25
million threshold for assets under management?
I. Is the account a securities portfolio?
II. Does the account receive continuous and regular supervisory or
management services?
48.
(A) I only
(B) II only
(C) Both I and II
(D) Neither I nor II
Which of the following statements concerning the Pension Benefit
Guaranty Corporation is (are) correct?
I. It was created by ERISA in 1974.
II. It protects vested benefits in defined benefit plans.
(A)
(B)
(C)
(D)
I only
II only
Both I and II
Neither I nor II
Student Workbook for HS 300
11
49.
Which of the following statements concerning interviews is (are) correct?
I. They typically take place in a formal and structured setting.
II. Digressions from the subject are encouraged.
50.
(A) I only
(B) II only
(C) Both I and II
(D) Neither I nor II
Which of the following statements concerning the ability of people to
rationally assess risk is (are) correct?
I. People tend to perceive more risk in things that are familiar to them.
II. People tend to overestimate the risk involved in activities under their
control.
51.
(A) I only
(B) II only
(C) Both I and II
(D) Neither I nor II
Which of the following statements concerning disintermediation is (are)
correct?
I. It reduces reserves in the banking system.
II. It reduces deposits held in banks.
52.
(A) I only
(B) II only
(C) Both I and II
(D) Neither I nor II
Which of the following statements concerning client resistance in the
financial planning environment is (are) correct?
I. The financial advisor should avoid discussing the client’s marital
problems.
II. The financial advisor should deal with covert hostility directly by
focusing on the client’s behavior.
(A)
(B)
(C)
(D)
I only
II only
Both I and II
Neither I nor II
Student Workbook for HS 300
12
53.
Frank and Jim (both male) are married and live in Massachusetts. Under
which status can they file their federal income taxes?
I. married filing jointly
II. married filing single
54.
(A) I only
(B) II only
(C) Both I and II
(D) Neither I nor II
Which of the following statements concerning a client’s net cash flow is
(are) correct?
I. A client should have a positive net cash flow in the long run if he or
she wants the flexibility to fund additional financial objectives.
II. A client who has a negative net cash flow will have to invest (spend)
additional resources in order to balance income and expenses.
55.
(A) I only
(B) II only
(C) Both I and II
(D) Neither I nor II
Federal income tax deductions are available to all taxpayers for which of
the following types of education funding costs?
I. interest on education loans up to an annual maximum deductible
amount
II. qualified tuition and related expenses up to an annual maximum
deductible amount
56.
(A) I only
(B) II only
(C) Both I and II
(D) Neither I nor II
Which of the following statements concerning monetary policy is (are)
correct?
I. It is used to manage the deficit.
II. It is used to manage inflation.
(A)
(B)
(C)
(D)
I only
II only
Both I and II
Neither I nor II
Student Workbook for HS 300
13
57.
Which of the following statements concerning The American College’s
Code of Ethics is (are) correct?
I. Enforcement of the code is through specified procedures.
II. Violations of the code are subject to disciplinary sanctions.
58.
(A) I only
(B) II only
(C) Both I and II
(D) Neither I nor II
Thrill seekers are characterized in which of the following ways?
I. They tend to be outgoing and spontaneous.
II. They tend to take their time when making decisions.
59.
(A) I only
(B) II only
(C) Both I and II
(D) Neither I nor II
Which of the following statements concerning the Sarbanes-Oxley Act of
2002 is (are) correct?
I. It created an oversight board to monitor the auditing profession.
II. It increased corporate reporting responsibilities.
60.
(A) I only
(B) II only
(C) Both I and II
(D) Neither I nor II
Which of the following statements concerning nonverbal behaviors is (are)
correct?
I. They are linguistic signs used in communication.
II. Active listening is ineffective in interpreting these behaviors.
61.
(A) I only
(B) II only
(C) Both I and II
(D) Neither I nor II
Which of the following statements concerning the CFP® Board’s Practice
Standards is (are) correct?
I. They are enforceable by the CFP® Board against CFP® designees.
II. They are designed to be a basis for legal liability when they are
breached.
(A)
(B)
(C)
(D)
I only
II only
Both I and II
Neither I nor II
Student Workbook for HS
14
62.
Monetary risk takers can be characterized in which of the following ways?
I. They list their primary long-term goal as happiness.
II. They tend to possess leadership and sales skills.
63.
(A) I only
(B) II only
(C) Both I and II
(D) Neither I nor II
Kevin and Christine were divorced last year. According to the terms of the
divorce, Kevin paid Christine $50,000 of alimony payments and $75,000
of child support payments. Which of the following statements is correct?
I. Christine must add $125,000 of income on her tax return.
II. Christine can deduct $75,000 from her tax return.
64.
(A) I only
(B) II only
(C) Both I and II
(D) Neither I nor II
Which of the following statements concerning financial advisors as
professionals is (are) correct?
I. They are held to higher standards in the execution of their business
duties.
II. They are expected to place their own business interests above those
of clients.
65.
(A) I only
(B) II only
(C) Both I and II
(D) Neither I nor II
Which of the following statements concerning net worth on the financial
position statement is (are) correct?
I. Net worth is the key to understanding the nature of the client’s assets
and liabilities.
II. Net worth is what remains if all the client’s assets are sold at their fair
market values and all debts are paid.
(A)
(B)
(C)
(D)
I only
II only
Both I and II
Neither I nor II
Student Workbook for HS 300
15
66.
Which of the following statements concerning the public benefit from
regulating the financial services industry is (are) correct?
I. It fosters public confidence in institutions.
II. It reduces the profit margins of firms.
67.
(A) I only
(B) II only
(C) Both I and II
(D) Neither I nor II
Monetary policy tools include which of the following?
I. setting margin requirements on brokerage accounts
II. changing federal income tax rates
68.
(A) I only
(B) II only
(C) Both I and II
(D) Neither I nor II
Which of the following statements concerning the Pension Benefit
Guaranty Corporation is (are) correct?
I. Single-employer pension plans pay a basic flat-rate premium of $19
a participant per year.
II. Underfunded pension plans pay an additional charge of $9 per $1,000
of unfunded vested benefits.
69.
(A) I only
(B) II only
(C) Both I and II
(D) Neither I nor II
Which of the following is (are) guaranteed to keep pace with increases in
college tuition costs for a state school?
I. CollegeSure CDs
II. zero coupon bonds
(A)
(B)
(C)
(D)
I only
II only
Both I and II
Neither I nor II
Student Workbook for HS 300
16
70.
The Investment Advisers Act of 1940 generally applies to the activities of
a financial services professional if which of the following questions is (are)
answered in the affirmative?
I. Does the financial services professional provide advice or analysis
about securities?
II. Is the financial services professional registered with the NASD to sell
securities?
(A)
(B)
(C)
(D)
I only
II only
Both I and II
Neither I nor II
READ THE
CONTINUING
FOLLOWING
DIRECTIONS
BEFORE
The questions below differ from the preceding questions in that they all
contain the word EXCEPT. So you understand fully the basis used in
selecting each answer, be sure to read each question carefully.
71.
72.
Financial advisor activities considered to be part of step 4 (develop and
present a plan) in the financial planning process include all the following
EXCEPT
(A) obtaining the client’s approval of the plan
(B) presenting alternative plan strategies to the client
(C) writing a report for the client that describes the plan
(D) reviewing the plan to see that it is performing satisfactorily
All the following statements concerning counseling by an advisor are
correct EXCEPT
(A)
(B)
(C)
(D)
73.
The advisor’s primary method of communication is asking questions.
Counseling is less formal and less structured than interviewing.
The advisor’s role as a counselor takes place over a period of time.
Through the counseling process, the advisor and client develop an
interpersonal relationship.
All the following statements concerning the relationship between wealth
and financial risk tolerance are correct EXCEPT
(A) Absolute risk tolerance increases with wealth since the wealthy have
more money to spend on everything.
(B) People who earned their own wealth tend to be more risk tolerant than
those who inherited their wealth from beloved relatives.
(C) Relative risk tolerance increases with wealth if housing is included in
the definition of wealth or classified as a risky asset.
(D) People who acquired their wealth through a windfall are more apt to
expose it to risk than those who worked hard for it.
Student Workbook for HS 300
17
74.
75.
76.
77.
78.
79.
Assets classified as nonfinancial (personal) assets include all the following
EXCEPT
(A) residential real estate
(B) trusts with an equity interest
(C) equity in privately held businesses
(D) tangible personal assets such as collectibles
The Free Application for Federal Student Aid form (FAFSA) is used as
the basis for estimating the expected contribution from all the following
EXCEPT
(A) parents
(B) student
(C) family
(D) grandparents
All the following statements concerning auto leases are correct EXCEPT
(A) A closed-end lease permits the lessee to buy the car at the end of the
lease for the car’s residual value.
(B) An open-end lease permits the lessee to buy the car at the end of the
lease for the car’s residual value.
(C) A closed-end lease is riskier for the consumer than an open-end lease.
(D) Most auto leases are closed-end leases.
All the following individuals always have a fiduciary duty to the client
EXCEPT
(A) CFA charterholder
(B) registered investment adviser
(C) CFP® Certificant
(D) insurance agent
According to a survey conducted by the CFP® Board in 1999, the top three
financial planning areas of specialization that advisors engage in include
all the following EXCEPT
(A) closely held business planning
(B) pension/retirement planning
(C) comprehensive planning
(D) investment planning/advice
All the following statements concerning basic communication principles
are correct EXCEPT
(A)
(B)
(C)
(D)
Communication is learned through experience.
The same word may have different meanings to different people.
Listening is a form of communication.
Communication occurs mainly through verbal discourse.
Student Workbook for HS
18
80.
81.
82.
83.
84.
Unmarried partners face all of the following unique challenges in estate
planning EXCEPT
(A) Unmarried couples may owe federal gift taxes when titling property
into joint accounts.
(B) They may owe state inheritance taxes when transferring assets to one
another at death.
(C) They are required to pay federal estate tax when transferring assets to
one another at death.
(D) Unmarried couples may owe federal gift taxes when putting property
into an irrevocable trust.
All the following typically are considered discretionary expenses for cash
flow statement purposes EXCEPT
(A) savings and investments
(B) household furnishings
(C) vacations and travel
(D) household maintenance costs
Estimating the cost of funding a college education requires information
about all the following EXCEPT
(A) rate of increase in the consumer price index (CPI)
(B) number of years until matriculation
(C) number of years of college attendance
(D) current cost of a year of college
All the following statements concerning state regulation of investment
advisers are correct EXCEPT
(A) Many states require a minimum capitalization amount as a
prerequisite to granting investment adviser registration approval.
(B) Almost every state requires that the participants in the registered
investment advisory operation have certain NASD registrations.
(C) The application of state law has been eliminated with respect to
investment adviser representatives of SEC-registered advisers.
(D) State securities commissions may want to see all contracts, disclosure
statements, documents, and literature the registered investment
adviser used with the client.
To develop rapport with clients an advisor must create an environment that
promotes openness by doing all the following EXCEPT
(A)
(B)
(C)
(D)
alleviating the concerns of clients
having informal and unstructured meetings with clients
responding to the social styles of clients
communicating effectively with clients
Student Workbook for HS 300
19
85.
86.
87.
88.
89.
As relates to the Principle of Professionalism, a CFP® Board designee shall
do all of the following EXCEPT
(A) offer advice only in those areas in which the CFP® Board designee
has competence
(B) use the marks in compliance with the rules and regulations of the
CFP® Board
(C) return the client’s original records in a timely manner after their return
has been requested by the client
(D) comply with all applicable renewal requirements established by the
CFP® Board
Cash flow management consists of all the following components EXCEPT
(A) budgeting
(B) cash flow analysis
(C) cash flow implementation
(D) cash flow planning
All the following statements concerning a client’s individually owned
insurance policies are correct EXCEPT
(A) Life insurance coverage amounts are a factor in the value of the
client’s estate.
(B) Property insurance coverage amounts are a factor in the client’s
liquidity position.
(C) Disability insurance coverages constitute one element of the client’s
resources available for disability needs.
(D) Liability insurance coverages should be analyzed to see if they
provide protection against potential liability losses.
All the following methods of funding a college education are subject to
income restrictions or demonstrated financial need a EXCEPT
(A) section 529 prepaid tuition plan
(B) Coverdell education savings account
(C) Hope scholarship tax credit
(D) Federal Pell Grant
All the following are exempt from regulation under the Investment
Advisers Act of 1940 EXCEPT
(A) the publisher of a financial magazine with general or regular
circulation
(B) an accountant who gives investment advice in an incidental manner
while practicing the accounting profession
(C) a broker or dealer who receives special compensation for advisory
services
(D) a financial services professional whose only clients are insurance
companies
Student Workbook for HS
20
90.
91.
92.
93.
94.
95.
Starting at a relatively young age, a career-minded person typically will
pass through all the following phases of his or her financial life cycle en
route to retirement EXCEPT
(A) early career
(B) career development
(C) peak accumulation
(D) postretirement
All the following Practice Standards are related to step 4 (developing and
presenting a plan) of the financial planning process EXCEPT
(A) identifying and evaluating financial planning alternatives
(B) determining a client’s personal and financial goals, needs, and
priorities
(C) presenting the financial planning recommendations
(D) developing the financial planning recommendations
All the following statements concerning guidelines on risk assessment are
correct EXCEPT
(A) Results from a questionnaire or other measurement device are only
an approximation.
(B) The assessment should begin by looking at the client’s demographic
characteristics and personality makeup.
(C) The client’s level of risk tolerance should be ascertained without
involving him or her in the assessment.
(D) Biases that distort the client’s true level of risk tolerance may be
operating on his or her risk perceptions.
All the following types of loans are secured EXCEPT
(A) home equity line of credit
(B) personal loan
(C) auto loan
(D) home equity loan
Richard inherited $200,000 from his grandmother. Richard’s inheritance
will be protected in case of divorce when the money is invested in any of
the following account titles EXCEPT
(A) irrevocable trust with Richard’s wife as trustee and children as
beneficiaries
(B) revocable trust with Richard as trustee and his wife as beneficiary
(C) individual
(D) revocable trust with Richard as trustee and his children as the
beneficaries
Reasons why an individual may want to reduce the size of an emergency
fund include all the following EXCEPT
(A)
(B)
(C)
(D)
The individual has worked for the same employer for many years.
The individual has his or her own business.
The individual has a life insurance policy with a large cash value.
The individual has a home equity line of credit.
Student Workbook for HS 300
21
96.
97.
98.
99.
100.
Tests used in determining whether a financial advisor’s activities are
subject to the provisions of the Investment Advisers Act of 1940 include
all the following EXCEPT
(A) a compensation test
(B) a business standard
(C) whether advice or analysis about securities is provided
(D) whether commentary on general market conditions is provided
Registered investment advisers must comply with all the following
responsibilities imposed by the SEC EXCEPT
(A) use the initials RIA after their names on all business cards and
letterheads
(B) obtain a client’s consent prior to transferring the account to another
adviser
(C) refrain from charging performance fees unless the adviser manages
$750,000 of the client’s assets or reasonably believes the client’s net
worth is at least $1.5 million
(D) deliver a written brochure to every current and prospective client
All the following acts affect the securities industry EXCEPT
(A) Investment Company Act of 1940
(B) Trust Indenture Act of 1939
(C) Sarbanes-Oxley Act of 2002
(D) McCarran-Ferguson Act of 1945
Financial advisors who are subject to SEC regulation have all the following
responsibilities EXCEPT
(A) record keeping
(B) recording conversations
(C) delivering brochures
(D) assigning contracts
All the following statements concerning advisor awareness of values are
correct EXCEPT
(A)
(B)
(C)
(D)
Advisors should seek to impose their values on clients.
Advisors should be aware of their own value systems.
Advisors should be aware that values can change.
Advisors should be aware that values are deeply internalized.
Student Workbook for HS 300
22
ANSWER KEY FOR SAMPLE EXAMINATION
Question Number
Correct Answer
Chapter
1
D
1
2
A
2
3
D
3
4
B
6
5
D
6
6
B
9
7
A
8
8
C
6
9
B
6
10
D
6
11
A
6
12
D
6
13
A
6
14
C
1
15
B
1
16
B
4
17
C
5
18
C
11
19
B
6
20
C
6
21
D
7
22
A
7
23
D
7
24
B
7
25
D
7
26
C
1
27
A
4
28
D
11
Student Workbook for HS 300
23
Question Number
Correct Answer
Chapter
29
C
2
30
D
3
31
C
7
32
D
7
33
D
7
34
B
7
35
A
11
36
A
1
37
C
1
38
B
4
39
B
9
40
A
5
41
D
2
42
D
11
43
A
3
44
B
4
45
C
5
46
B
8
47
C
9
48
C
10
49
A
2
50
D
4
51
C
10
52
B
2
53
D
11
54
A
5
55
D
8
56
B
10
57
C
3
58
A
4
Student Workbook for HS 300
24
Question Number
Correct Answer
Chapter
59
C
10
60
D
2
61
A
3
62
B
4
63
D
11
64
A
3
65
B
5
66
A
10
67
A
10
68
C
10
69
D
8
70
A
9
71
D
1
72
A
2
73
C
4
74
B
5
75
D
8
76
C
8
77
D
9
78
A
1
79
D
2
80
C
11
81
D
5
82
A
8
83
C
9
84
B
2
85
A
3
86
C
5
87
B
5
88
A
8
Student Workbook for HS 300
25
Question Number
Correct Answer
Chapter
89
C
9
90
D
1
91
B
3
92
C
4
93
B
8
94
A
11
95
B
8
96
D
9
97
D
10
98
D
10
99
B
9
100
A
2
Student Workbook for HS 300
26
Answers to Sample Examination Questions
1. The answer is (D).
(A), (B), and (C) are incorrect because in these steps of the financial planning
process the advisor does not identify the strengths and weaknesses in the client’s
present financial condition. This is done in step 3 of the process.
2. The answer is (A).
Open-ended questions allow the client to select a response from his or her full
repertory. (B) is incorrect because closed-ended questions limit the client to a
specific and narrow response, either a yes or no. (C) is incorrect because leading
questions are a type of closed-ended, manipulative question. More often than not
they lead the client toward a conclusion that the counselor (not the client) has
already formulated. (D) is incorrect because either/or questions limit the client
to only two options. Clients might prefer both options, neither one, or a third or
fourth option. Opening up questions allows the client to respond freely from his
or her own frame of reference.
3. The answer is (D).
(A), (B), and (C) are incorrect because ethics is not public relations; it is not
about creating a good image; and it is not a luxury to indulge in after meeting the
bottom line. Ethics is about how people conduct business every hour of every
day. Ethical behavior is being honest with ourselves and others.
4. The answer is (B ).
HP-10BII keystrokes:
, C ALL; 100, +/–, PV; 9, I/YR; 4, N; FV. Answer
displayed: 141.16.
(A), (C), and (D) are incorrect calculations.
5. The answer is (D).
HP-10BII keystrokes:
, C ALL; 8000, +/–, PV; 10000, FV; 3, I/YR; N. Answer
displayed: 7.55.
(A), (B), and (C) are incorrect calculations.
6. The answer is (B).
(A), (C), and (D) are incorrect because while these acts pertain to the securities
industry, they do not set the limit for assets under management.
7. The answer is (A).
(B), (C), and (D) are incorrect because these assets are not liquid assets.
Emergency fund assets must be able to be quickly converted to cash without loss
of value in order to meet the client’s needs. A money market mutual fund is a
liquid asset that can quickly be converted to cash without loss of value.
8. The answer is (C).
Student Workbook for HS 300
27
9.
10.
11.
12.
13.
14.
15.
16.
17.
HP-10BII keystrokes:
, C ALL;
, BEG/END (if BEGIN displayed); 1000,
PMT; 9, I/YR; 5, N; PV. Answer displayed: –3,889.65.
(A), (B), and (D) are incorrect calculations.
The answer is (B).
HP-10BII keystrokes:
, C ALL; 10000, FV; 9, I/YR; 4, N; PV. Answer
displayed: –7,084.25.
(A), (C), and (D) are incorrect calculations.
The answer is (D).
HP-10BII keystrokes:
, C ALL;
, BEG/END (if BEGIN not displayed);
25000, FV; 4000, +/–, PMT; 5, N; I/YR. Answer displayed: 7.53.
(A), (B), and (C) are incorrect calculations.
The answer is (A).
HP-10BII keystrokes:
, C ALL; , BEG/END (if BEGIN displayed); 10000,
PMT; 6, I/YR; 4, N; PV. Answer displayed: –34,651.06.
(B), (C), and (D) are incorrect calculations.
The answer is (D).
HP-10BII keystrokes:
, C ALL;
, BEG/END (if BEGIN not displayed);
1000, +/–, PMT; 9, I/YR; 4, N; FV. Answer displayed: 4,984.71.
(A), (B), and (C) are incorrect calculations.
The answer is (A).
HP-10BII keystrokes:
, C ALL;
, BEG/END (if BEGIN displayed); 1000,
FV; 875, +/–, PV; 60, PMT; 10, N; I/YR. Answer displayed: 7.85.
(B), (C), and (D) are incorrect calculations.
The answer is (C).
Building a retirement fund was the top reason why people begin financial
planning. This was selected by 82 percent of the survey participants.
(A) is incorrect because accumulating capital was selected by 31 percent of those
surveyed. (B) is incorrect because purchasing or renovating a home was selected
by 41 percent of those surveyed. (D) is incorrect because generating current
income was selected by 25 percent of those surveyed.
The answer is (B).
(A), (C), and (D) are incorrect because in these steps of the financial planning
process the advisor does not review changes in the client’s circumstances and the
financial environment.
The answer is (B).
With this approach to measuring financial risk tolerance the client indicates the
products that he or she prefers as investments.
(A), (C), and (D) are incorrect because while they are approaches to measuring a
client’s risk tolerance, they are not the most direct approach.
The answer is (C).
Student Workbook for HS 300
28
18.
19.
20.
21.
22.
23.
24.
25.
26.
(A) is incorrect because it is the debt service ratio. (B) is incorrect because it is
the solvency ratio. (D) is incorrect because it is the savings ratio.
The answer is (C).
To test Pam for front loading: $100,000 – ($30,000 + $15,000) = $55,000. (A) is
incorrect because $0 results from applying the debt service ratio. (B) is incorrect
because $45,000 results from applying the solvency ratio. (D) is incorrect
because $90,000 results from applying the savings ratio.
The answer is (B).
HP-10BII keystrokes:
, C ALL;
, BEG/END (if BEGIN not displayed);
100000, FV; 8, I/YR; 5, N; PMT. Answer displayed: –15,783.01.
(A), (C), and (D) are incorrect calculations.
The answer is (C).
HP-10BII keystrokes:
, C ALL;
, BEG/END (if BEGIN displayed); 1000,
+/–, PMT; 4, I/YR; 7, N; FV. Answer displayed: 7,898.29.
(A), (B), and (D) are incorrect calculations.
The answer is (D).
HP-10BII keystrokes:
, C ALL; , DISP, 4; 5,
, NOM%; 365,
, P/YR;
, EFF%. Answer displayed: 5.1267. Then reset for one payment period per
year: 1,
, P/YR, C.
(A), (B), and (C) are incorrect calculations.
The answer is (A).
HP-10BII keystrokes:
, C ALL; 2000, +/–, CFj; 3000, CFj; 1000, +/–, CFj;
0, CFj, 2,
, Nj; 3000, CFj, 2,
, Nj; 6, I/YR;
, NPV. Answer displayed:
4,296.85.
(B), (C), and (D) are incorrect calculations.
The answer is (D).
HP-10BII keystrokes:
, C ALL; 12000, FV; 5, ÷, 2, =, I/YR; 6, x, 2, =, N;
PV. Answer displayed: –8,922.67, becomes –8,923 when rounded to the nearest
dollar.
(A), (B), and (C) are incorrect calculations.
The answer is (B).
HP-10BII keystrokes:
, C ALL; 0, CFj; 1000, CFj; 3000, CFj; 2000, CFj; 0,
CFj; 2000, CFj, 2,
, Nj; 5, I/YR;
, NPV. Answer displayed: 8,460.63.
(A), (C), and (D) are incorrect calculations.
The answer is (D).
HP-10BII Part 1 keystrokes:
, C ALL; 0, CFj; 1000, CFj, 2,
, Nj; 0, CFj;
2000, CFj, 2,
, Nj; 6, I/YR;
, NPV. Answer displayed: 4,912.10. Part 2
keystrokes: +/–, PV; 5, N; FV. Answer displayed: 6,573.49.
(A), (B), and (C) are incorrect calculations.
The answer is (C).
Student Workbook for HS 300
29
27.
28.
29.
30.
31.
32.
33.
(A) is incorrect because describing the client’s present situation based on personal
and financial data gathered from the client takes place in step 2 of the financial
planning process. (B) is incorrect because specifying the client’s stated goals
indicating the priority of each one and the time frame for achieving it takes place
in step 2 of the financial planning process. (D) is incorrect because identifying
problems that the client would encounter in attempting to accomplish stated goals
takes place in step 3 of the financial planning process.
The answer is (A).
(B), (C), and (D) are incorrect because financial risk tolerance increases with the
degree of formal education as specified in (A).
The answer is (B).
Borrowing for the purchase of a home accounts for 70.7 percent of total family
debt. (A) is incorrect because education accounts for 3.1 percent of total family
debt. (C) is incorrect because investments account for 2.8 percent of total family
debt. (D) is incorrect because the purchase of vehicles accounts for 7.8 percent
of total family debt.
The answer is (C).
(A) is incorrect because a nondirective interview takes more time than a directive
interview. (B) is incorrect because the client usually controls the pacing of a
nondirective interview through the depth of his or her responses. (D) is incorrect
because the advisor directs and controls both the pace and the content to be
covered in a directive interview.
The answer is (D).
(A) is incorrect because defining the scope of the engagement is related to step
1 (establishing and defining the advisor-client relationship) of the financial
planning process. (B) is incorrect because defining monitoring responsibilities
is related to step 6 (monitoring the plan) of the financial planning process. (C) is
incorrect because presenting the financial planning recommendations is related
to step 4 (developing and presenting the plan) of the financial planning process.
The answer is (C).
HP-10BII keystrokes:
, C ALL;
, BEG/END (if BEGIN not displayed);
200000, PMT; 1.09, ÷ 1.20, –, 1, x, 100, =, I/YR; 6, N; PV. Answer displayed:
–1,546,699.49.
(A), (B), and (D) are incorrect calculations.
The answer is (D).
HP-10BII keystrokes:
, C ALL; 10000, +/–, CFj; 5000, +/–, CFj; 0, CFj;
10000, CFj; 20000, CFj;
, IRR/YR. Answer displayed: 23.12.
(A), (B), and (C) are incorrect calculations.
The answer is (D).
Student Workbook for HS 300
30
34.
35.
36.
37.
38.
39.
HP-10BII keystrokes:
, C ALL; 8000, +/–, PV; 8, ÷, 52, =, I/YR; 3, x, 52, = N;
FV. Answer displayed: 10,168.12, becomes 10,168 when rounded to the nearest
dollar.
(A), (B), and (C) are incorrect calculations.
The answer is (B).
HP-10BII keystrokes:
, C ALL;
, BEG/END (if BEGIN displayed); 2,
, P/YR; 3,
, xP/YR; 25, PMT; 1000, FV; 8, I/YR; PV. Answer displayed:
–921.37. Then reset for one payment period per year: 1,
, P/YR, C.
(A), (C), and (D) are incorrect calculations.
The answer is (D).
All the answers are examples of special needs trusts, but only answer (D) is
applicable to a pooled trust. (A), (B), and (C) do not apply to pooled trusts.
The answer is (A).
(B), (C), and (D) are incorrect because these financial advisors are practicing
single-purpose financial planning, which is solving a single financial problem
with a single financial product or service.
The answer is (C).
(A) is incorrect because as the client moves up the pyramid, the focus of the
plan shifts from income protection needs to wealth accumulation goals. (B)
is incorrect because the middle part of the pyramid is the wealth accumulation
component of the financial plan. It involves growing money through various
types of investments. (D) is incorrect because the foundation of the pyramid
is focused on protecting the client against an unexpected occurrence that could
cause financial hardship. It is built with emergency savings, insurance coverages,
and a properly drawn will.
The answer is (B).
Availability bias refers to the fact that events that are dramatic and vivid or that
receive heavy media attention are easily available to one’s mind and, therefore,
tend to be overestimated. (A) is incorrect because it refers to the tendency not
to appreciate the true level of danger in the situation. (C) is incorrect because it
refers to the tendency to believe after the fact that an event was more predictable
than it usually was. (D) is incorrect because it refers to the tendency some people
have to seek information that supports their beliefs.
The answer is (B).
(A) is incorrect because while acting in the client’s best interest is important, it
may not be enough. You must be able to prove in court that you have done what
a prudent man would have done in that situation. (C) is incorrect because acting
on a best efforts basis may simply not be enough if a prudent man would have
done more. (D) is incorrect because it is never appropriate to act on self-interest
unless that happens to coincide with the client’s interest.
Student Workbook for HS 300
31
40. The answer is (A).
(B) is incorrect because it partially describes cash flow analysis, not cash flow
planning. (C) is incorrect because the difference between income and expenses
is net cash flow, not cash flow planning. (D) is incorrect because the difference
between total assets and total liabilities is net worth, not cash flow planning.
41. The answer is (D).
I is incorrect because the explanatory type of leading response is meant to be
a relatively neutral description of the way things are. A reassuring response is
intended to make the client feel better.
II is incorrect because the goal of the interpretive efforts including an interpretive
type of leading response is self-interpretation by the client in order to increase the
client’s ability to act effectively.
42. The answer is (A). A health care power of attorney allows the attorney in fact to
expedite Medicare filing for a disabled individual.
II is incorrect because the codes call on advisors to look out for the best interests
of clients.
43. The answer is (A).
II is incorrect because the codes C ALL on advisors to look out for the best
interests of clients.
44. The answer is (B).
I is incorrect because a stable employment history is more indicative of risk
averters. The willingness to make frequent voluntary job changes is considered
an indicator of a willingness to take financial risks.
45. The answer is (C).
Both I and II are correct.
46. The answer is (B).
I is incorrect because the Parent Loans for Undergraduate Students (PLUS loans)
are available for all family incomes and levels; they are not need based.
47. The answer is (C).
Both I and II are correct.
48. The answer is (C).
Both I and II are correct.
49. The answer is (A).
II is incorrect because the subject matter discussed in an interview is specific to
the overall purpose of the interview, and digressions from the subject are usually
not encouraged.
50. The answer is (D).
I is incorrect because people tend to perceive less risk in things that are familiar
to them. II is incorrect because people tend to underestimate the risk involved in
activities under their control.
51. The answer is (C).
Student Workbook for HS 300
32
52.
53.
54.
55.
56.
57.
58.
59.
60.
61.
62.
Both I and II are correct.
The answer is (B).
I is incorrect because once a financial advisor recognizes that a couple is
having marital problems, he or she should encourage the couple to focus on
the problems. Otherwise, if the problems are ignored, whatever decisions are
reached will be less valid than those that would have been reached after a full
airing of the problems.
The answer is (D).
Frank and Jim are both men and cannot file under either married status under
current federal income tax rules.
The answer is (A).
II is incorrect because if a client has a negative net cash flow, the alternative
courses of action available to correct this problem involve either an increase in
income or a reduction in expenses. Investing additional resources will not rectify
the problem but will aggravate it.
The answer is (D).
I is incorrect because the deductibility of even a limited amount of interest
on education loans is not available to all taxpayers. It is phased out for high
taxpayers. II is incorrect because the deductibility of even a limited amount of
higher education expenses is not available to all taxpayers. It is phased out for
high income taxpayers.
The answer is (B).
I is incorrect because fiscal policy is used to manage the deficit.
The answer is (C).
Both I and II are correct.
The answer is (A).
II is incorrect because thrill seekers tend to be fast decision makers.
The answer is (C).
Both I and II are correct.
The answer is (D).
I is incorrect because nonverbal behaviors refer to those aspects of
communication other than words themselves. They are nonlinguistic signs that
convey a wealth of information.
II is incorrect because the active listener responds not only to the verbal message
but also to the nonverbal message communicated by the sender’s body, facial
expression, and/or tone of voice.
The answer is (A).
II is incorrect because the CFP® Board’s Practice Standards are not designed to
be a basis legal liability.
The answer is (B).
Student Workbook for HS
33
63.
64.
65.
66.
67.
68.
69.
70.
71.
72.
73.
I is incorrect because monetary risk takers list their primary long-term goal as
success. Monetary risk averters stated their long-term goal as happiness.
The answer is (D).
Christine cannot deduct child support payments. She only needs to add $50,000
of alimony payment to her income tax return.
The answer is (A).
II is incorrect because financial advisors are expected to place the interests of
clients above their own.
The answer is (B).
I is incorrect because net worth reveals little about the nature of assets or
liabilities; it is simply a residual—the difference between total assets and total
liabilities.
The answer is (A).
II is incorrect because regulating the financial services industry does not reduce
the profit margins of firms.
The answer is (A).
II is incorrect because changing federal income tax rates is not a monetary policy
tool but a fiscal policy tool.
The answer is (C).
Both I and II are correct.
The answer is (D).
I is incorrect because CollegeSure CDs pay an interest rate that is linked to the
inflation rate in college costs as measured by the College Board’s cost index for
500 independent colleges; it is not linked to tuition costs for state schools.
II is incorrect because baccalaureate bonds do not guarantee that they will keep
pace with the increases in college tuition costs for a state school; they typically
are poor inflation hedges.
The answer is (A). II is incorrect because the question is not one of the tests used
to determine whether the activities of a financial services professionals are subject
to the Investment Advisers Act of 1940.
The answer is (D).
Reviewing the plan to see that it is performing satisfactorily is a financial advisor
activity that is considered part of step 6 (monitor the plan) of the financial
planning process.
The answer is (A).
While the advisor may ask questions in counseling, they are not his or her primary
method of communication. The advisor may paraphrase what the client has said,
reflect a feeling, share feedback or perceptions, clarify, summarize, interpret,
provide information, and confront.
The answer is (C).
Student Workbook for HS 300
34
74.
75.
76.
77.
78.
79.
80.
81.
82.
83.
Relative risk tolerance increases with wealth but only if housing is either excluded
from the definition of wealth or classified as a riskless asset.
The answer is (B).
Trusts with an equity interest are classified as investment assets for financial
position statement purposes.
The answer is (D).
The data contained in the FAFSA is used as the basis for estimating the expected
parent contribution and the expected student contribution, which combined
equal the expected family contribution. There is no expected contribution from
grandparents.
The answer is (C).
An open-end lease is riskier for the consumer than a closed-end lease. With an
open-end lease, if the market value is less than the estimated residual value, the
consumer is obligated to make up the difference. In the case of a closed-end lease,
if the market value at the end of the lease is less than the projected residual value,
the dealer absorbs the loss.
The answer is (D).
Insurance agents do not have a fiduciary duty to their clients in all instances.
Many agents perform a dual role that can lead to conflicts of interest if they are
not careful. This dual role finds them as agents for their companies while they
are simultaneously serving their clients in an agent-like capacity.
The answer is (A).
Closely held business planning is not one of the top three financial planning areas
of specialization engaged in by advisors.
The answer is (D).
Most sociological research claims that approximately two-thirds of an
individual’s total message is communicated via nonverbal channels.
The answer is (C).
Unmarried partners are not required to pay federal estate tax when transferring
assets to one another at death. Unmarried partners might owe taxes based on the
size of their estate, but are not required to pay any more than single individuals.
The answer is (D).
Household maintenance costs are considered to be a fixed expense for cash flow
statement purposes.
The answer is (A).
The rate of increase in the consumer price index is not useful for estimating the
cost of funding a college education. What is needed is the education inflation
rate.
The answer is (C).
Student Workbook for HS 300
35
84.
85.
86.
87.
88.
89.
90.
91.
92.
93.
All
but
seven
states
require
individual
investment
adviser
representatives—including those who are associated with SEC-registered
firms—to pass examinations designed to test minimum competency.
The answer is (B).
In any kind of planned and purposeful communication setting, the first element
that needs to be attended to is structuring. It serves to determine both the format
and the subject matter of the interaction that is to follow. Proper structuring
will not only alleviate most of the client’s anxieties about the financial planning
process, it will also help to build rapport with the client.
The answer is (A).
Offering advice only in those areas in which the CFP® Board designee has
competence relates to the Principle of Competence.
The answer is (C).
Cash flow implementation is not one of the recognized components of cash flow
management. Cash flow analysis, cash flow planning, and budgeting are its three
basic components.
The answer is (B).
Property insurance coverages should be analyzed to see if they provide adequate
protection against potential property losses. They are not a factor in the client’s
liquidity position.
The answer is (A).
Section 529 prepaid tuition plans have no income restrictions nor do they require
a demonstration of financial need.
The answer is (C).
A broker or dealer who receives special compensation for advisory services is
not exempt from regulation under the Investment Advisers Act of 1940.
The answer is (D).
Postretirement is not one of the four phases of the financial life cycle that a
career-minded person will pass through on route to the retirement phase. Besides
the early career, career development, peak accumulation, and retirement phases,
the other phase is preretirement.
The answer is (B).
The Practice Standard of determining a client’s personal and financial goals,
needs, and priorities is related to step 2 (determining goals and gathering data)
of the financial planning process.
The answer is (C).
Assessing the client’s risk tolerance is a cooperative venture. The advisor should
use the information he or she collects to start a dialogue with the client about risk
tolerance.
The answer is (B).
Student Workbook for HS
36
By definition, a personal loan is an unsecured debt. The creditor must rely entirely
on the debtor’s promise to repay since there is no collateral.
94. The answer is (A).
95. The answer is (B).
An emergency fund that covers 6 months or even a year’s living expenses may
be inadequate if the client is self-employed or works in an industry not known
for job security.
96. The answer is (D).
Providing commentary on general market conditions will not subject a financial
advisor to the provisions of the Investment Advisers Act of 1940.
97. The answer is (A).
The SEC has taken the position that a registered investment adviser must not use
the initials RIA after his or her name.
98. The answer is (D).
The McCarran-Ferguson Act of 1945 states that regulation of insurance will
remain with the states provided the states actually regulate the insurance
companies operating within their borders.
99. The answer is (B).
Financial advisors who are subject to SEC regulation are not required to record
conversations with their clients.
100.The answer is (A).
Advisors who are aware of their own value systems have a better chance of
avoiding the imposition of their values on the clients. This quality is of vital
importance since the advisor wants to help the client make decisions that stem
from the client’s own value systems rather than from the advisor’s.
Student Workbook for HS 300